Sie sind auf Seite 1von 140

SCHOOL OF MINING ENGINEERING

UNIVERSITY OF NEW SOUTH WALES


Australia

Underground Mine Environment Course


MODULE 1.0 VENTILATION AND MINE SERVICES

SP
TP

SP

TP

1.1 FLUID FLOW, FRICTION AND SHOCK LOSSES

A Minerals Tertiary Education Council Initiative


MINERALS
COUNCIL
Of Australia

File UNSW-UME-(M1-1_FluidFlow)(R1) Page 1 of 40


Revision date December 2003
School Of Mining Engineering Underground Mine Environment
 2003 University Of New South Wales Module 1.1 Fluid Flow, Friction And Shock Losses

Contents Page

1.0 INTRODUCTION……………………………………………………….………………………….. 3

2.0 FUNDAMENTAL UNITS AND RELATIONSHIPS…………….……………………………….. 4

2.1Geometry, Quantity And Velocity……………………………………………………………… 4


2.2Temperature………………………………………………………….…………………………… 5
2.3 Mass And Density………………………………………………….…………………………… 5
2.4 Pressure……………………………………………………………..…………………………… 7
2.4.1 Absolute, Gauge And Relative Pressure…………………..…………………………… 8
2.4.2 Pressure In A Fluid Column……………………………………………………………… 9
2.4.2.1 Natural Ventilation Pressure And Buoyancy……….……………………………11
2.5 Compressible Flow ……………………………………………………………………………. 13
2.6 Static Velocity And Total Pressure……………………………….…………………………… 15

3.0 PRESSURE LOSS DUE TO FRICTION, DISCONTINUITIES AND OBSTRUCTIONS…… 17

3.1 Frictional Losses On Rough Surfaces…………………………………………………………19


3.1.1 Friction Factor and Surface Roughness…………………………………………………21
3.1.2 Resistance Of Leakage Paths……………………………...…………………………… 24
3.2 Shock Losses Due To Circuit Discontinuities…………………………………………………25
3.3 Pressure Losses Due To Airway Obstructions…………..……………………………………29
3.4 Equivalent Length And Effective Airway Resistance…………………………………………32

4.0 BERNOULLI’S EQUATION…………………………………………..…………………………… 33

4.1 Application To Trailing Hose Surveys…………………………………………………………35


4.2 Application To Traverse Barometer Surveys………………….………………………………36
4.3 Application To Orifice Plates And Regulators…………………………………………………37

5.0 REFERENCES………………………………………………………….………………………….. 39

5.1 References Underpinning This Module………………………….…………………………… 39


5.2 References Supplied With Course Notes………………………...…………………………. 39
5.3 Further References………………………………………………………………………………39
5.4 Acknowledgement………………………………………………….……………………………39

APPENDIX A – Frictional Loss Spreadsheet…………………………….……………………………40

File UNSW-UME-(M1-1_FluidFlow)(R1) Page 2 of 40


School Of Mining Engineering Underground Mine Environment
 2003 University Of New South Wales Module 1.1 Fluid Flow, Friction And Shock Losses

1.0 INTRODUCTION
The analysis of fluid flow is fundamental to mine ventilation design, monitoring and practice. The
purpose of this module is to develop an understanding and application of the laws, relationships and
equations required to undertake analysis of fluid flow in ventilation and gas reticulation systems
associated with the underground mine environment.

Unlike the chaotic behaviour of weather systems on surface, gas mixtures and liquids behave in a
relatively predictable manner when confined to ventilation circuits, ducted systems or pipe ranges.
Subject to appropriate assumptions and limitations, it is possible to describe most flow process
encountered to within an acceptable tolerance and within that of standard measurement techniques
employed.

The thermodynamic state of chemically stable gases (e.g. air or methane) and liquids (e.g. water) is
defined by their temperature and pressure. This leads to consideration of fluid properties such as
mass, density, volume and flow rate. In addition, the geometry of the system in which a fluid is flowing
(depth, cross sectional area, discontinuities etc) will determine changes in pressure that occur. It is the
analysis and prediction of circuit pressure differentials for defined flow rates that provides the means
for designing and monitoring the performance of ventilation and gas reticulation systems.

For the purposes of this module, incompressible flow is assumed unless otherwise stated. Corrections
are made for actual density (e.g. in Atkinson’s friction equation) but a thorough analysis of the
thermodynamic behaviour of gases and moist air in the mine environment is beyond the scope of this
module. Potential errors resulting from this assumption are quantified in Section 2.5.

These module notes summarise key aspects of fluid flow, formulae and calculation methods to a
standard required for this course. In this respect, they are not exhaustive but are intended to provide
sufficient detail for the training of industry practitioners and graduate education. It is essential that
candidates read and understand underpinning references for this module. Further references are
provided for candidates requiring more detailed information.

File UNSW-UME-(M1-1_FluidFlow)(R1) Page 3 of 40


School Of Mining Engineering Underground Mine Environment
 2003 University Of New South Wales Module 1.1 Fluid Flow, Friction And Shock Losses

2.0 FUNDAMENTAL UNITS AND RELATIONSHIPS


Fluids are defined as substances that cannot support shear stresses without moving and are classified
as being liquids or gases.

A liquid has intermolecular forces which bind it together so that it possesses the property of volume
but with no definite shape. The nature of intermolecular bonds in liquids is such that changing applied
pressure results in very small changes of volume. For practical purposes, liquids can be considered
incompressible.

A gas, or mixture of gases, comprises molecules that are in constant motion and free to move or
disperse in any direction. Gases do not posses the property of essentially constant volume, which
changes with temperature and or pressure. In an ideal gas, molecules move freely and independently
of each other. In a real gas, inter molecular forces alter ideal gas behaviour to some degree.

Temperature and pressure define the thermodynamic state of a liquid or gaseous fluid. From these
values and the physical properties of the fluid, other defining characteristic such as density, or
viscosity, can be determined. In addition, the dynamic state of a fluid is defined by directional velocity,
which determines further terms, such as velocity pressure.

2.1 Geometry, Quantity And Velocity


The values of length m, circumference (perimeter) m, area m2 and volume m3 of airways, ducts or
pipes are obtained for various shapes using standard geometrical relationships, refer Figure 2.1 In
mine applications the base unit normally used is the meter although mm may be appropriate in some
cases.

Figure 2.1 Geometry Of Airways And Ducts

3
Length m Perimeter m Volume m

Height m
Volume
3
Area m
2 m Area m
2
Diameter m

3
Quantity m /s
3
Quantity m /s Circumference m Velocity m/s Length m
Velocity m/s
Width m

Cross sectional area Circumference


Cylindrical A = π.D2 m2 C = π.D m
4
2
Rectangular A = w x h m C = 2.( w + h) m

Volume V=AxL m3 Surface = C x L m2


area

Fluid velocity is expressed in units of m/s and is calculated from volumetric flow rate (quantity) m3/s
and cross sectional area, m2. Flow rate or quantity can be expressed either in m3/s or l/s where
1.0m3/s = 1000l/s. This relationship is summarised in Equation 2.1.

Equation 2.1 Calculation Of Quantity From Velocity and Area

Quantity = Velocity x Area Q = V x A m3/s

In mine workings, air velocities would normally range 0.5 to 1.5 m/s in development headings under
auxiliary ventilation, up to 12 m/s in main intake and return airways and up to 24 m/s in exhaust shafts.

File UNSW-UME-(M1-1_FluidFlow)(R1) Page 4 of 40


School Of Mining Engineering Underground Mine Environment
 2003 University Of New South Wales Module 1.1 Fluid Flow, Friction And Shock Losses

As a guide, dust pick up from surfaces becomes problematic above ≈ 5.0 m/s and pedestrian access
is difficult above 10 to 12 m/s. To avoid suspension of water droplets in exhaust shafts, velocities
between 7.0 and 12.0 m/s are normally avoided.

Velocities in ducted ventilation systems may be up to 35 m/s, but are normally in the range 20 to 25
m/s for acceptable pressure losses.

Worked Example – Geometry , Quantity And Velocity

What is the cross sectional area and perimeter of a rectangular airway 5.0 m wide x 3.2 m high ?

Area = width x height = 5.0 x 3.2 = 16.0 m2

Perimeter = 2 x (width + height) = 2 x (5.0 + 3.2) = 16.4 m

What is the velocity in the airway for a quantity of 35.0 m3/s ?

Velocity = Quantity = 35.0 / 16.0 = 2.19 m/s


Area

2.2 Temperature
The SI unit of temperature is Celcius t OC based on the boiling point (100OC) and freezing point (0OC)
of water at standard atmospheric pressure. This scale is most convenient for temperatures
encountered in general mine conditions, however, when considering the behaviour of gases, it is
necessary to consider their absolute temperature. At absolute zero, gas molecules occupy the
minimum space possible and are considered to be at rest. Without movement of molecules no
pressure can be applied to a confining physical boundary. That is, absolute zero pressure occurs at
absolute zero temperature. By experimentation, absolute zero is found to occur at –273.15OC.

The Kelvin scale OK is based on 0OK = absolute zero = - 273.15OC with the value of one OK being
exactly the same as one OC.

Equation 2.2 Calculation Of Degrees Kelvin From Degrees Celsius

O O
Temperature in kelvin T K = t C + 273.15

Worked Example – Kelvin Temperature Scale

What is the boiling point of water in degrees Kelvin ?

Boiling point of water = 100 OC = 100 + 273.15 = 373.15 OK

2.3 Mass And Density


The SI unit of mass is the kilogram (kg) which can be related to force by force = mass x acceleration
where acceleration is that due to gravity. For a unit mass under the influence of gravity (9.81m/s2 )
1.0kg therefore represents a force of 9.81 N.

The density of a solid, liquid or gaseous substance is the mass of the substance (kg) per unit volume
(m3) or ρ kg/m3.
Equation 2.3 Calculation Of Density From Mass And Volume

Density ρ = mass kg/m3


volume

File UNSW-UME-(M1-1_FluidFlow)(R1) Page 5 of 40


School Of Mining Engineering Underground Mine Environment
 2003 University Of New South Wales Module 1.1 Fluid Flow, Friction And Shock Losses

For example, 1.0m3 of water has a mass of 1000kg at 4OC and 1.0m3 of air has a mass of
approximately 1.2kg. For constant mass, the change in volume of a liquid with pressure or
temperature is small and therefore the density of liquids can be considered essentially constant.
However, the volume of a constant mass of gas changes significantly with pressure and temperature
and therefore so does it’s density.

It is important to note that two methods for calculating the behaviour of gas mixtures can be used, the
first is approximate and the second more accurate but somewhat more complex. The first method
assumes gases to behave as an ideal incompressible fluid for which the effect of changing volume
and density are recognised as being small and ignored. The second method considers the true
thermodynamic behaviour of air or gas mixtures (Hemp 1982) where account is made for changing
volume and density in addition to heat and mass transfer processes that may occur.

The assumption that air or gas mixtures behave as ideal, incompressible fluids, greatly simplifies
analysis and is acceptable where changes in pressure and temperature do not result in a significant
change in air density or an approximate result is acceptable. This issue is discussed further in Section
2.5.1.

Relative density (RD) is the ratio of a gas density to that of air at the same pressure and temperature.
A gas with a relative density of 1.0 has the same density as air, is lighter for RD <1.0 and heavier for
RD> 1.0

Specific Volume (ν) is defined as the volume per unit mass (m3/kg) that is, the volume occupied by
1kg of a gas and is simply the inverse of density.

Worked Example – Mass, Volume And Density

What is the mass of 1.6 m3 of coal (kg) if the density is 1.39 t/m3

Mass = Density x volume = 1.39 x 1.6 = 2.224 t = 2,224 kg

What vertical force is exerted by this coal (N/m2) if spread evenly over an area of 3 m2 ?

Force per unit area = mass x acceleration (gravity) = 2,224 x 9.81 = 7,272 N/m2
area 3

The density of a gas mixture is 1.46 kg/m3. What volume is occupied by 3.5 kg of this gas at the same
density ?

Volume = mass = 3.5 / 1.46 = 2.397 m3


density

What is the specific volume of the gas ?

Specific volume ν = 1 = 1 / 1.46 = 0.685 m3/kg


density

What is the relative density of the gas compared to air (density 1.2 kg/m3) ?

Relative density RD = density of gas = 1.46 / 1.2 = 1.217 ie heavier


density of air

File UNSW-UME-(M1-1_FluidFlow)(R1) Page 6 of 40


School Of Mining Engineering Underground Mine Environment
 2003 University Of New South Wales Module 1.1 Fluid Flow, Friction And Shock Losses

2.4 Pressure
The pressure exerted by a fluid on a confining physical boundary results from the force exerted by
molecules colliding with the boundary surface. The force exerted depends on the number of molecules
impacting per unit surface area and their kinetic energy, refer Figure 2.2.

Figure 2.2 Effect Of Changing Volume Or Temperature On Pressure.

Change in fluid temperature

Kinetic energy of molecules depends on


temperature therefore influences the Same mass of fluid in smaller volume
impact force per unit area ie pressure
increases or decreases. Increased number of molecules impacting
surface per unit area therefore pressure
increases.
The effect of temperature and pressure on gas volume and density is discussed further in Section 2.5.

Pressure is defined as the force exerted per m2 and is, expressed in N/m2 or Pascals Pa. Note that the
dimensions of Newtons (N) are kg m/s2 and that the force exerted on a containing boundary is
directional but pressure acts equally in all directions.

Equation 2.4 Calculation Of Quantity From Velocity and Area

Force = mass x acceleration N

Pressure = Force N/m2 or Pa


Area

Several other units of pressure are used in different engineering disciplines. For example, the bar (1
bar = 0.1MPa = 100 kPa) is used extensively by meteorologists and many barometers are gradated in
millibars. Meteorologists also report atmospheric or barometric pressure in hectopascals (HPa) where
1HPa = 100Pa.

Because atmospheric temperature and pressure are variable, the scientific community has nominated
a standard temperature and pressure (STP) for the purpose of reporting gas properties. These are:

Standard Pressure = 1 atmosphere = 101.325 kPa


Standard Temperature = 0OC = 273.15OK
Normal temperature =20OC = 293.15OK

Worked Example –Pressure And Force

The pressure differential across a 3.0 m x 5.0 m bulkhead is 900 Pa. What is the effective load
on the bulkhead in kg ?
Pressure = Force per unit area therefore 900 Pa = 900 N/m2

Force = mass x acceleration and therefore mass = force / acceleration

Mass per unit area = 900 / 9.81 = 91.7 kg/m2 (Note gravity may vary 9.79 to 9.81 m/s2 )

Area of bulkhead = height x width = 3.0 x 5.0 = 15.0 m2

Total effective load on bulkhead = 91.7 x 15 = 1,376 kg

File UNSW-UME-(M1-1_FluidFlow)(R1) Page 7 of 40


School Of Mining Engineering Underground Mine Environment
 2003 University Of New South Wales Module 1.1 Fluid Flow, Friction And Shock Losses

2.4.1 Absolute, Gauge and Relative Pressure


Pressures are reported as absolute when relative to absolute zero, gauge when reported relative to
an external datum (normally atmosphere) or relative when measured between two points in a system,
refer Figure 2.3

Figure 2.3 Absolute, Gauge And Relative Pressure

System pressure higher


than atmospheric pressure
Positive gauge pressure
Relative
pressure Absolute pressure Atmospheric pressure
greater than
atmospheric pressure Negative gauge pressure
System pressure lower
than atmospheric pressure
Absolute pressure less
than atmospheric
pressure

Absolute zero pressure

Worked Example – Units Of Pressure

If atmospheric pressure is 101,300 Pa. What is the pressure in HPa ?

1 HPa = 100 Pa. Therefore pressure = 101,300/100 = 1,013 HPa

The pressure inside a sealed balloon (A) is 1,500 Pa greater then atmospheric pressure. What is the
gauge pressure and absolute pressure in the balloon ?

Gauge pressure = pressure relative to atmosphere = + 1,500 Pa

Absolute pressure = atmosphere + gauge = 101,300 + 1,500 = 102,800 Pa

Another balloon (B) has a gauge pressure of + 1100 Pa. What is the relative pressure of balloon A to B.

Relative pressure = Gauge A – Gauge B = 1,500 – 1,100 = + 400 Pa

File UNSW-UME-(M1-1_FluidFlow)(R1) Page 8 of 40


School Of Mining Engineering Underground Mine Environment
 2003 University Of New South Wales Module 1.1 Fluid Flow, Friction And Shock Losses

2.4.2 Pressure In A Fluid Column


The pressure at a point in a stationary column of fluid, with respect to the pressure applied to the
surface datum, is calculated from the fluid density and column height, as follows;

Equation 2.5 Absolute And Gauge Pressures In Fluid Columns

Absolute pressure P2 = P + ρ.g.h Pa P

Gauge pressure P2 = ρ.g.h Pa


h

P2
To confirm this relationship, consider the dimensions involved;

The dimensions of ρ.g.h are kg . m . m = kg . m2 = kg m = N = Pa


m3 s2 m3 s2 m2 s2 m2

It is important to recognise that the change in pressure due to the height of a fluid column applies to
both gases and liquids while stationary or in motion. For example, assuming density remains the
same, the difference in pressure from surface to the bottom of an intake shaft due to the air column is
the same with or without air movement.

Worked Example –Pressure Due To Fluid Column

What is the gauge pressure at the bottom of a 1.0 m column of air, water and mercury relative to the
surface ?

Pressure change = ρ.g.h Pa

For air = 1.2 x 9.81 x 1.0 = 11.78 Pa

For water = 1,000 x 9.81 x.1.0 = 9,810 Pa

For mercury = 13,600 x 9.81 x 1.0 = 133,416 Pa

Surface atmospheric pressure is 101,300 Pa. What is the pressure in units of mm Hg ?

Pressure = 101,300 = ρ.g.h Therefore h = 101,300 = 0.7593 m = 759.3 mmHg


13,600 x 9.81

What is the absolute pressure at the bottom of a 250 m deep shaft due to the column of air at a density of
1.18 kg/m3 if the surface pressure is 101,300 Pa?

Absolute pressure = P + ρ.g.h = 101,300 + 1.18 x 9.81 x 250 = 101,300 + 2,894 = 104,194 Pa

Pressure differentials are commonly measured with manometers comprising a transparent tube
containing a fluid of known density, for example mercury. The coloured manometer fluid contained in
many lower pressure manometers has a density less than 1000 kg/m3 (about 800 kg/m3) but is
selected to have a well defined meniscus.

The measuring limb can be vertical or inclined in order to magnify fluid movement for a given pressure
differential, refer Figure 2.4.

File UNSW-UME-(M1-1_FluidFlow)(R1) Page 9 of 40


School Of Mining Engineering Underground Mine Environment
 2003 University Of New South Wales Module 1.1 Fluid Flow, Friction And Shock Losses

Figure 2.4 Use Of Inclined Manometer Limb To Magnify Change


P system P atmosphere
L inclined
P atmosphere
L vertical

h2 h1

For the same change in fluid height (h2 – h1), the movement of fluid on the inclined scale is greater.
Calculation of gauge and absolute pressure from manometer fluid levels is described in Figure 2.5.

Figure 2.5 Calculation Of Pressure Using A Manometer


Atmospheric pressure P

A Manometer level 1

Pipe at higher
pressure than
atmosphere
Pressure to be
C determined in pipe
hm
Fluid inside pipe hp

B Manometer level 2

Manometer fluid

The absolute pressure indicated by the column of manometer fluid A to B = P + ρm.g.hm Pa


The pressure difference due to the column of fluid C to B = ρp.g.hp Pa
The absolute pressure at C = P + ρm.g.hm - ρp.g.hp Pa

Worked Example –Manometer

For the example in Figure 2.5. What is the absolute and gauge pressure in the pipe if atmospheric
pressure is 101,250 Pa, and

a) The manometer contains mercury and the pipe contains water


ρm = 13,600 kg/m3 hm = 200 mm ρp = 1,000 kg/m3 hp = 120 mm

Pressure due to manometer fluid = ρm.g.hm = 13,600 x 9.81 x 0.2 = 26,683 Pa

Pressure due to pipe fluid in manometer = ρp.g.hp = 1,000 x 9.81 x 0.12 = 1,177 Pa

Gauge pressure in pipe = 26,683 – 1,177 = 25,506 Pa

Absolute pressure in pipe = 101,250 + 26,683 – 1,177 = 126,756 Pa

b) The manometer contains water and the pipe contains air.


ρm = 1,000 kg/m3 hm = 200 mm ρp = 1,2 kg/m3 hp = 120 mm

Pressure due to manometer fluid = ρm.g.hm = 1,000 x 9.81 x 0.2 = 1,962 Pa

Pressure due to pipe fluid in manometer = ρp.g.hp = 1.2 x 9.81 x 0.12 = 1.4 Pa

Gauge pressure in pipe = 1,962 – 1.4 = 1,961.6 Pa

Absolute pressure in pipe = 101,250 + 1,962 – 1.4 = 103,211 Pa

File UNSW-UME-(M1-1_FluidFlow)(R1) Page 10 of 40


School Of Mining Engineering Underground Mine Environment
 2003 University Of New South Wales Module 1.1 Fluid Flow, Friction And Shock Losses

These calculations demonstrate the following points;

• When the pressure of liquids (high density) are being measured, the effect of liquid density on
observed manometer differentials should be accounted for.

• When the pressure of gases (low density) is being measured, the effect of the gas column in the
manometer limb can normally be considered negligible.

• Inclining the measuring limb of the manometer will improve the accuracy of measurement.

• The density of the manometer fluid determines the column length required for a particular
pressure. The length of manometer limbs can be kept manageable by using higher density fluids ,
for example mercury.

2.4.2.1 Natural Ventilation Pressure And Buoyancy


The height of a column and fluid density determines the pressure developed. An important
consequence of this fact in mine ventilation networks occurs when columns of gas or air with different
densities are connected, refer Figure 2.6.

Figure 2.6 Natural Ventilation And Buoyancy Effect

Exhaust
shaft Goaf Intake
drainage shaft

Gas mixture
Less dense of variable More dense Depth of
warmer air ρ2 density ρ2 cooler air ρ1 workings h

Pressure of
workings

Heating of air within a mine normally results in exhaust air being hotter, and therefore less dense, than
intake air. A pressure differential then exists due to the differences in air density in the closed loop
from surface – down the intake shaft – through the workings and returning to surface via the exhaust
shaft. The magnitude of the pressure developed depends on the height of columns (depth of workings)
and the difference in average intake and exhaust density, as follows;

Equation 2.6 Natural Ventilation Pressure

Natural ventilation pressure = ( ρ1 - ρ2 ) .g.h Pa

Natural ventilation pressures become more significant with increasing depth and vary with surface
climatic (intake) conditions. In some locations, subject to very high mid summer temperatures, the
natural ventilation pressure can be negative with respect to direction of ventilation flow. This may
occur at the onset of hotter surface conditions when the body of the mine is at a lower temperature
and cools the intake air.

A second application of this effect, for which column density must be taken into account, is drainage of
seam gas from coal mine goafs through vertical bore holes. In this instance, seam gas may comprise
pure methane or, more commonly, a variable mixture of methane, oxygen, nitrogen and carbon
dioxide. The density of the gas mixture in the bore hole may therefore be significantly greater or less
than that of air. This is often termed the “buoyancy” effect although it is based on the same principles
as natural ventilation.

File UNSW-UME-(M1-1_FluidFlow)(R1) Page 11 of 40


School Of Mining Engineering Underground Mine Environment
 2003 University Of New South Wales Module 1.1 Fluid Flow, Friction And Shock Losses

Worked Example – Natural Ventilation Pressure and Buoyancy

The depth of mine workings is 230 m. The average density of intake air is 1.19 kg/m3 and the average
density of exhaust air is 1.16 kg/m3 .

What is the natural ventilation pressure in this mine ?

Intake column pressure = ρ.g.h = 1.19 x 9.81 x 230 = 2,684 Pa


Exhaust column pressure = ρ.g.h = 1.16 x 9.81 x 230 = 2,617 Pa

Difference = natural ventilation pressure = 2,684 – 2,617 = 67 Pa

What would the natural ventilation pressure be in a mine four times deeper with the same average intake
and exhaust densities ?

Column pressures are directly proportional to depth :. NVP = 67 x 4 = 268 Pa

What would the buoyancy effect be in a goaf drainage hole if the gas has a density of 0.74 kg/m3 ?

Intake column pressure = ρ.g.h = 1.19 x 9.81 x 230 = 2,684 Pa

Drain column pressure = ρ.g.h = 0.74 x 9.81 x 230 = 1,670 Pa

Difference = buoyancy pressure = 2,684 – 1,670 = 1,015 Pa

If the ventilation pressure of the mine’s workings are –1200 Pa with respect to surface, what would
happen in an open goaf drainage hole ?

Net pressure applied to hole = +1,015 – 1200 = - 185 Pa

In this case, although methane is lighter than air, the hole would be expected to downcast in to the mine and a
fan would be required for drainage.

At what depth of workings would a goaf hole be expected to free flow to surface due to buoyancy ?

Free flow starts when buoyancy pressure exceeds ventilation pressure (1200 Pa).

(ρ1 - ρ 2).g.h = 1200 Pa therefore h = 1200 = 272 m


(1.19-0.74) x 9.81

File UNSW-UME-(M1-1_FluidFlow)(R1) Page 12 of 40


School Of Mining Engineering Underground Mine Environment
 2003 University Of New South Wales Module 1.1 Fluid Flow, Friction And Shock Losses

2.5 Compressible Flow


In ventilation circuits or gas drainage systems, only the mass flow rate of dry gas really remains
constant. Therefore calculation techniques, for example network simulation programs, that work with
quantity rather than mass flow rates are not strictly correct. In many situations, for example frictional
losses in semi horizontal airways, the errors involved are small enough to ignore. In other situations,
for example deep shafts or gas drainage systems, the effect of pressure is significant and the change
in volumetric flow rate must be taken into account.

In the module Mine Gases And Gas Laws, the universal gas law is developed to enable gas density
to be calculated if the pressure and temperature are known. Consequently, if a change in pressure
occurs due to frictional losses or changes in elevation, or temperature changes due to heat transfer,
then the volumetric flow must also change. For a given mass of air or gas mixture, there is of course
an associated error in density.

A complete analysis of the thermodynamic behaviour of mine ventilation (Hemp 1982) recognises
these additional issues and bases calculation on mass flow rates rather than quantity. In addition,
moist air does not behave as a perfect gas and the universal gas law must also be corrected
accordingly. Analysis of this type is not within the scope of this module, however, acceptable results
can be obtained by accounting for the magnitude of errors involved.

For example, consider 1.0m3 of dry air at 101.325kPa and 20OC and the range of pressure-
temperature combinations that may apply in the underground mine environment, refer Figure 2.7

Figure 2.7 Variation Of Air Volume With Absolute Pressure

1500 m deep
30
shaft
20 250 m deep
coal m ine
Error In Volume %

10
30C
0
20C
-10 Intake to 3 kPa
0C exhaust fan
-20
-30 -20 kPa gas
drainage
-40
80 90 100 110 120
Pressure Pa (abs)

The highest positive pressures applied to mine ventilation air occur at the bottom of deep intake shafts
(+17.4kPa at 1500 m) and the lowest would occur at the intake to surface exhaust fans (- 4.0 to
5.0kPa). In addition, temperatures can range typically from around 0OC to 35 OC although more
extreme values may be encountered under some climatic conditions or in close proximity to heat
sources. For these conditions, air density would range 1.12 to 1.38 kg/m3.

This indicates that variations in volume of circa ± 5% volume would be expected in shallow mine
ventilation circuits but variations in volume up to approximately 20% may occur in deeper mine
workings. For example, at 1500m depth of workings, approximately 20 % less air volume enters the
workings compared to that entering intake shafts on surface. Clearly this effect must be taken into
account when considering underground ventilation requirements, surface fan capacity and shaft
diameters.

Coal mine gas drainage systems probably involve the lowest gauge pressures (-20 to –40kPa) and
would involve errors of - 25 to -50% volume. That is, the volumetric flow rate in a gas reticulation pipe
may be 25 to 50% greater than that reported at discharge from surface pumps. This obviously has a
profound effect on system design.

File UNSW-UME-(M1-1_FluidFlow)(R1) Page 13 of 40


School Of Mining Engineering Underground Mine Environment
 2003 University Of New South Wales Module 1.1 Fluid Flow, Friction And Shock Losses

Without involving a detailed thermodynamic analysis, the practical approach is to account for the
average density in various parts of the system for, example intake shafts, workings and exhaust
shafts, and then increase detail for systems that incur more significant density changes, for example
gas drainage systems. It is also normally acceptable to assume that air density varies linearly with
depth in deep shafts.

Worked Example – Accounting For Compressible Flow

The frictional loss in a 2000 m long airway is 410 Pa for 60 m3/s and the end of the airway is 50 m higher
than the start. The barometric pressure at the start is 102,100 Pa, density is 1.24 kg/m3 and temperature
remains constant.

What is the pressure, density and volumetric flow rate at the end of the airway ?

Pressure due to column = pgh = 1.2 x 9.81 x 50 = -589 Pa (assuming density is constant)

Pressure at end of airway = 102,100 -410 – 589 = 101,101 Pa

Density at end of airway = p1. P1/P2 = 1.2 x 102,100 / 101,101 = 1.212 kg/m3

Volumetric flow rate = 60 x 1.212 / 1.2 = 60.6 m3/s or 1% higher.

Average density of airway = (1.2 + 1.21) / 2 = 1.205 kg/m3

Pressure due to column = pgh = 1.205 x 9.81 x 50 = -591 Pa (re calculated at average density)

The calculation could then continue to re calculate the average density between start and end of the airway and
hence correct the column pressure to iterate to a final result. However, the change in volume of 1.0 % could not
be measured in practice and the incremental change in pressure is also small.

A mine needs 220 m3/s to enter the workings at the bottom of a 1200 m deep shaft. The shaft resistance is
0.005 Ns2/m8 per 100 m. Estimate the amount of air required to enter the top of the shaft and loss within
the shaft ?

Surface conditions are 101.3 kPa and a density of 1.2 kg/m3 .

Initial guess

Intake column pressure = ρ.g.h = 1.2 x 9.81 x 1200 = 14,126 Pa

Intake frictional loss = 1200 x 0.005 x 1.2 x 2202 = 2,904 Pa


100 1.2

Initial shaft bottom pressure = 101,300 + 14,126 – 2,904 = 112,522 Pa

Air quantity required at surface for 220 m3/s at shaft bottom = 220 x 112,522 = 244.4 m3/s
101,300

Using a surface quantity of 244.4 m3/s, pressures are recalculated in terms of average shaft density and quantity
as follows.

Bottom Average Average Recalulated


Depth pgh Pfric Pressure Density Q Density Quantity Pfric
m Pa Pa Pa kg/m3 m3/s kg/m3 m3/s Pa
1200 14909 3414 112795 1.336171 219.47 1.26809 231.92 3410.3
1200 14928 3410 112818 1.336437 219.42 1.26822 231.90 3410.0
1200 14929 3410 112819 1.336459 219.42 1.26823 231.90 3410.0
1200 14930 3410 112820 1.336461 219.42 1.26823 231.90 3410.0

The surface quantity of 244.4 m3/s would provide 219.4 m3/s to the workings. The pressure at the bottom of the
shaft would be 112,820 Pa, the density would be 1.336 kg/m3 and the frictional loss would be 3,410 Pa.

File UNSW-UME-(M1-1_FluidFlow)(R1) Page 14 of 40


School Of Mining Engineering Underground Mine Environment
 2003 University Of New South Wales Module 1.1 Fluid Flow, Friction And Shock Losses

2.6 Static, Velocity And Total Pressure


Section 2.4.2 described pressure in static fluid columns and that due to differential densities in mine
ventilation circuits. This section is concerned with the analysis of pressure in circuits involving fluid
movement together with pressure losses due to friction and pressure gains due to circuit fans. Fluids
in motion are described by two pressure terms, namely static and velocity, refer Figure 2.8

Figure 2.8 Static, Velocity And Total Pressure


Static pressure
Velocity pressure
Total pressure
Static pressure

Static pressure – or bursting pressure acts with the same magnitude in all directions and is not
dependant on flow. Static pressure is measured at right angles to the direction of flow (side tube).

Velocity pressure – or dynamic pressure is a result of the movement of the fluid. The magnitude of
velocity pressure is dependent on velocity and fluid density. Velocity pressure is normally measured
on the axis of flow direction (facing tube).

Equation 2.7 Velocity Pressure

Velocity pressure VP = ρ V2 Pa
2

To confirm this relationship, consider the dimensions involved;

The dimensions of.ρ.V2 are kg . m 2


= kg . m2 = kg m = N = Pa
2 m3 s m3 s2 m2 s2 m2

Total pressure – the total or facing tube, pressure at any point is given by the sum of static and
velocity pressures. It does not represent a third independent pressure term but indicates that unless
pressure (energy) is added to or removed from the system (friction or fans), the total pressure will
remain unchanged. In gauge and absolute pressure terms the total pressure is given by;

Equation 2.8 Total Gauge And Absolute Pressure

Total pressure TP = SP + VP Pa (gauge)


Total pressure TP = P + VP + g. ρ.h Pa (abs)

In this case, P, is the absolute static pressure at a point in the system. It is important to note that when
measuring gauge pressures, the ρ.g.h component is not included.

The connection of facing and side tubes for measurement of total, static and velocity pressures are
shown in Figure 2.9. Pressure measurements of this type are normally undertaken with a pitot tube
which comprises a central facing tube within a second outer casing. The outer casing connects side
(static pressure) holes to the measuring port through the annulus between the inner and outer tubes.

Figure 2.9 Static, Velocity And Total Pressure Measurement

VP & SP TP
TP TP
SP
SP

SP TP VP

Static (side) tube Total (facing) tube Velocity (TP-SP)

File UNSW-UME-(M1-1_FluidFlow)(R1) Page 15 of 40


School Of Mining Engineering Underground Mine Environment
 2003 University Of New South Wales Module 1.1 Fluid Flow, Friction And Shock Losses

Worked Example – Total, Static And Velocity Pressure

The static pressure in a 760 mm duct is - 2300 Pa and the quantity is 11 m3/s. What is the velocity and
total pressure (ρ = 1.18 kg/m3)?

Area = π . D2 /4 = π . 0.762 / 4 = 0.454 m2

Velocity = Quantity / Area = 11.0 / 0.454 = 24.2 m/s

Velocity pressure = 0.5.ρ .V2 = 0.5 x 1.18 x 24.22 = 347 Pa

Total pressure = SP + VP = -2300 + 347 = -1953 Pa

The duct cross sectional area increases to 0.6 m2 without any frictional losses. What is the new velocity
and static pressure ?

Velocity = Quantity / Area = 11.0 / 0.6 = 18.3 m/s

Velocity pressure = 0.5.ρ .V2 = 0.5 x 1.18 x 18.32 = 198 Pa

Total pressure = constant = -1953 Pa

Static pressure = total - velocity = -1953 - 198 = -2151 Pa

Note that change in static pressure = change in velocity pressure

For the purposes of this analysis it is assumed that gauge static and total pressures only change when
frictional losses or fans are included. This will be dealt with in more detail in chapter 4.0 Bernoulli’s
Equation, but consider the following worked example.

Worked Example – Total, Static And Velocity Pressure With Friction

The absolute barometric (static) pressure on surface at the top of a downcast shaft is 99,500 Pa. The shaft
is 4.0 m diameter, 400 m deep and the air quantity is 250 m3/s at an average density of (ρ = 1.18 kg/m3).
The frictional loss in the shaft is 340 Pa. Ignore entry shock losses etc.

What is the velocity and absolute static pressure at the top and bottom of the shaft ?

The cross sectional area of the shaft is

Area = π . D2 /4 = π . 4.02 / 4 = 12.57 m2

Velocity = Quantity / Area = 250 / 12.57 = 9.94 m/s

Velocity pressure VP = 0.5.ρ .V2 = 0.5 x 1.18 x 9.942 = 58 Pa

Column pressure CP = ρ.g.h = 1.18 x 9.81 x 400 = 4,630 Pa

Frictional loss FP = 340 Pa

As air enters the shaft the velocity pressure increases by 58 Pa therefore the static pressure must decrease by
the same amount.

SPtop = -58 Pa gauge or 99,500 – 58 = 99,442 Pa absolute.

For this example, incompressible flow is assumed, therefore the velocity pressure at the bottom of the shaft is 58
Pa.

The static pressure at the bottom of the shaft is

SPbot = SPtop + CP – FP = 99,442 + 4,630 – 340 = 103,732 Pa absolute.

File UNSW-UME-(M1-1_FluidFlow)(R1) Page 16 of 40


School Of Mining Engineering Underground Mine Environment
 2003 University Of New South Wales Module 1.1 Fluid Flow, Friction And Shock Losses

3.0 PRESSURE LOSS DUE TO FRICTION, DISCONTINUITIES & OBSTRUCTIONS


This section deals with the pressure loss term due to flow over rough surfaces (frictional loss) or flow
through and around discontinuities, for example bends.

When a fluid begins to move, individual molecules or layers start to move over each other in an
approximately linear fashion with retardation dependent on internal cohesion between molecules. This
is termed the viscosity of the fluid. As the velocity of the fluid increases, molecules or layers move in
an increasingly random fashion as the magnitude of inertial forces increase and the flow becomes
turbulent, refer Figure 3.1.

Figure 3.1 Laminar And Turbulent Flow

Low velocity High velocity


laminar flow turbulent flow

Surface
Surface

In addition, when a fluid moves over discontinuities, flow separation can occur dependent on velocity
and geometry of the discontinuity, refer Figure 3.2.

Figure 3.2 Total Head Loss Due To Boundary Layer And Flow Separation

Laminar Flow

Turbulent
Boundary Layer

Transition

Turbulent Flow

There are essentially two sources of pressure loss.

Boundary Layer – A pressure loss occurs due to the effect of adhesive forces between the fluid and
solid surface together with shear forces, determined by fluid viscosity, between adjacent layers of fluid
molecules. This losses occur in a relatively thin (≈ 5mm) “boundary layer” which will vary in size
depending on surface roughness and degree of turbulence. Properties of the boundary layer also
influence other processes such as heat and moisture transfer.

Flow Separation - According to Bernoulli’s equation, a decrease in velocity results in an increase in


static pressure. Therefore, when a fluid flows through a section of increasing area it is flowing against
an adverse pressure gradient. The boundary layer can flow against this pressure gradient for a
certain distance after which it detaches from the surface leaving a “dead fluid” region.

File UNSW-UME-(M1-1_FluidFlow)(R1) Page 17 of 40


School Of Mining Engineering Underground Mine Environment
 2003 University Of New South Wales Module 1.1 Fluid Flow, Friction And Shock Losses

The limits of laminar and turbulent flow can be determined from the dimensionless Reynolds number
which represents the ratio of inertial to viscous forces, as follows.

Equation 3.1 Reynolds Number

Reynolds Number Re = ρ . L . V
µ

Where L = a characteristic dimension, for example diameter.

The Reynolds number at which flow changes from a laminar to turbulent regime, depends on the initial
state of the fluid, the geometry of the system and roughness of the boundary surface. Generally, the
upper limit of laminar flow is between Reynolds numbers of 2,000 and 3,000. The pressure loss due to
friction is directly proportional to velocity in the region of laminar flow and proportional to the square of
velocity in turbulent regions. Between regions of total laminar or fully developed turbulent flow, a
region of transition exits in which the relationship between velocity and frictional loss is described by P
= vn , where n = 1.0 to 2.0.

In circular ducts or airways, diameter is the characteristic dimension, L, required for calculation of
Reynolds number. In non circular ducts or airways, an equivalent or “hydraulic” diameter is used. This
is the diameter of a circle having the same ratio of area to circumference as the shape being
considered, as follows;

Area of a circle A = π.D2 m2 and circumference C = π.D m


4
For a non circular profile with area A’ and perimeter P’, the hydraulic diameter is calculated as follows

Ratio of A’ to P’ A’ = A = π.D2 = D , this is then re arranged for D.


P’ C π.D 4 4

Equation 3.2 Hydraulic Diameter

Hydraulic diameter Dh = 4 A m
C

Worked Example – Reynolds Number

Calculate the velocity and quantity of air required to create turbulent flow (Reynolds number = 2500) in a
3.0 m high x 5.0 m wide development profile ? Assume the density of air = 1.16 kg/m3 and viscosity of air
= 18 x 10-6 kg/ms.

Reynolds Number Re =2500 = ρ . L . V Therefore velocity V = 2,500 x µ


µ ρ.L

Cross sectional area = height x width = 3.0 x 5.0 = 15.0 m2

Perimeter = 2 x (height + width) = 2 x (3.0 + 5.0) = 16.0 m

Characteristic dimension L = hydraulic diameter = 4.A = 4 x 15 = 3.75 m


C 16

Velocity for which Reynolds number = 2500 V = 2,500 x 18 x 10-6 = 0.01034 m/s
1.16 x 3.75

Quantity = velocity x area = 0.01034 x 15.0 = 0.155 m3/s

This demonstrates that in nearly all mining applications, turbulent flow can be assumed and the
“square law” applies. The exceptions may be in very large airways, for example open long hole
stopes or coal mine goafs, or when very low velocities exist, for example leakage through seals.

File UNSW-UME-(M1-1_FluidFlow)(R1) Page 18 of 40


School Of Mining Engineering Underground Mine Environment
 2003 University Of New South Wales Module 1.1 Fluid Flow, Friction And Shock Losses

3.1 Frictional Loss On Rough Surfaces


As gas or liquids flow over rough surfaces, there is a loss of pressure due to the interaction of the
boundary layer with the surface. This loss results in a reduction of static pressure, and in airways of
constant cross sectional area, an equal reduction of total pressure. As described above, in most
applications, flow is assumed to be turbulent and the frictional pressure loss is proportional to the
square of velocity.

Atkinson’s equation is the mathematical representation of the relationship between frictional pressure
loss occurring on rough surfaces and velocity.

Equation 3.3 Atkinson’s Equation In Terms Of Velocity

Pressure loss p = k.C.L. ρ .V2 Pa


due to friction A 1.2

but, V = Q/A, therefore;

Equation 3.4 Atkinson’s Equation In Terms Of Quantity

Pressure loss p = k.C.L. ρ .Q2 Pa


due to friction A3 1.2

Frictional pressure loss is described as being proportional to the “rubbing” surface area
(perimeter/circumference x length = C.L), roughness factor of the surface k, square of velocity and
inversely proportional to cross-sectional area. Density is incorporated as a ratio of actual fluid density
to standard density of 1.2 kg/m3, that is, frictional loss increases with increased density.

Terms, other than quantity, are commonly grouped to represent the airway resistance R Ns2/m8, (units
sometimes known as a Gaul) written as;

Equation 3.5 Resistance And Quantity Relationship

Pressure loss p = R . Q2 Pa
due to friction

The pressure loss due to friction can then be calculated for any airway shape if the flow rate or velocity
together with K factor of the surface, is known, refer Figure 3.3. The density correction should be used
for accurate results or if the density is significantly different to the standard 1.2 kg/m3, for example gas
mixtures other than air.

Figure 3.3 Airway Shape And Surface Roughness

Cylindrical e.g. smooth Rectangular e.g. coal


auxiliary ventilation duct mine development with Semi rectangular e.g. rough
or raise bore holes uniform low roughness blasted metalliferous mine
height from continuous development or arched sections.
miners

File UNSW-UME-(M1-1_FluidFlow)(R1) Page 19 of 40


School Of Mining Engineering Underground Mine Environment
 2003 University Of New South Wales Module 1.1 Fluid Flow, Friction And Shock Losses

The friction factor K, has units of Ns2/m4, and is dependent principally on surface roughness, but also
to some degree on the size of the airway. Surface roughness may vary considerably around the
perimeter of a mine airway (floor compared to roof) or be essentially uniform, such as in a ventilation
duct. The K factor used therefore represents the average for the airway perimeter in question.

For airways of similar cross sectional area, values of K can be obtained from tables or charts however,
it is always preferable to obtain site specific values by direct measurement. The value of k will range
from circa 0.003 Ns2/m4 for smooth ducts to circa 0.013 Ns2/m4 for rough surfaces. If the k factor is
used to represent airways containing obstructions, for example conveyor belts, the value may be
considerably higher but it must be recognised that this is an effective k factor for the specific airway
and not just for it’s surface. It is not therefore appropriate to apply a k factor obtained for an airway
with significant fill, for example a conveyor belt, to a larger airway with the same degree of fill.

Worked Example – Airway Resistance

The friction factor k for a 3.0 x 5.0 m airway profile is 0.012 Ns2/m4 . If the air density is 1.17 kg/m3, what
is the resistance R Ns2/m8 of a 100 m length of the airway ?

Cross sectional area = height x width = 3.0 x 5.0 = 15.0 m2

Perimeter = 2 x (height + width) = 2 x (3.0 + 5.0) = 16.0 m

Resistance per 100 m R100 = p = k.C.L. p = 0.012 x 16 x 100 x 1.17 = 0.005547 Ns2/m8
A3 1.2 15.03 1.2

What is the frictional pressure loss when a ventilation rate of 45 m3/s flows through 450 m of the airway ?

Resistance of 450 m = R100 x Length = 0.005547 x 450 = 0.025 Ns2/m8


100 100

Frictional pressure loss P = R . Q2 = 0.025 x 452 = 51 Pa

When considering appropriate dimensions of airways or ducts, it is important to recognise the


profound importance of diameter (hydraulic diameter for non circular profiles) on resistance and
consequential frictional pressure loss.

For a given length and k factor, the resistance R ∝ C or R ∝ π.D therefore R ∝ D =1


A3 (π.D2)3 D6 D5

that is, resistance is inversely proportional to the fifth power of diameter.

Worked Example – Effect Of Diameter On Resistance

The friction factor k for a 610 mm and 750 mm duct is 0.0033 Ns2/m4 . If air density is 1.17 kg/m3, what is
the frictional pressure loss for 10 m3/s through 100 m of each duct ?

Cross sectional area A = π.D2 A = π. x 0.6102 = 0.292 m2 A = π. x 0.7502 = 0.442 m2


4 4 4

Circumference C = π.D C = π..0.610 = 1.916 m C = π..0.750 = 2.356 m

Resistance R = k.C.L . p R = 0.0033 x 1.916 x 100 x 1.17 = 24.7 Ns2/m8 for 610 mm duct
A3 1.2 0.292 3 1.2

R = 0.0033 x 2.356 x 100 x 1.17 = 8.8 Ns2/m8 for the 750 mm duct
0.442 3 1.2

Frictional pressure loss P = R.Q2 P = 24.7x* 102 = 2470 Pa for 610 mm duct

P = 8.8 x 102 = 880 Pa for 750 mm duct

File UNSW-UME-(M1-1_FluidFlow)(R1) Page 20 of 40


School Of Mining Engineering Underground Mine Environment
 2003 University Of New South Wales Module 1.1 Fluid Flow, Friction And Shock Losses

3.1.1 Friction Factor And Surface Roughness


Airway friction factors can be obtained from charts, tables or measured by pressure-quantity surveys.
Those obtained from surveys are site specific and most appropriate for “tuning” simulation models or
other design purposes. Those obtained from charts or tables are based on “average” conditions, and
in particular the roughness height of the airway surface.

In addition to tabulated values possibly not being appropriate for the location in question, it is also
difficult to “visualise” k factors reported in units of Ns2/m4 . The following section develops a method of
calculating k factors, in terms of the surface roughness height and airway dimensions. This method
can then be used to reconcile measured values with observed conditions and a sensitivity analysis to
be undertaken. For example effect of raise bore diameter on frictional losses.

The method is derived as follows;

Using Atkinson’s equation. p = K . C . L . ρ . V2


A 1.2

From the velocity pressure VP = ρ.v2 the term ρ.V2 can be written ρ.V2 . 2
2 1. 2 2 1.2

and hydraulic diameter Dh = 4. A re arranging C = 4


C A Dh

by substitution, Atkinson equation can be then rewritten as follows;

p = k . L . 4 . ρ . V2 . 2
Dh 2 1.2

Which reduces to p = 6.67 . K . L ρ.V2


Dh 2

Assigning λ = 6.67 k, Atkinson’s equation reduces to the Darcy Weisbach equation for frictional loss;
as follows;

Equation 3.6 Darcy Wesibach Equation

Pressure loss p = λ. L ρ.v2 Pa


Dh 2

From test work and further dimensional analysis of the equation it can be shown that the
dimensionless friction coefficient λ is a function of Reynolds number (degree of turbulence) and the
ratio of surface roughness height to duct diameter (Chasteau 1982).

The relationship between λ, Reynolds number and surface roughness height ( e mm) is described by
the following empirical formula;

1 = 1.74 - 2 Log10(2ε + 18.7 )


√λ Re √λ

Where the dimensionless relative roughness ε is given by ε = e/D or e/Dh. Note that e and Dh must
be in the same units ie mm or m

But for turbulent flow (large Re), in aerodynamically rough ducts, the term 18.7 is very small.
Re√λ

File UNSW-UME-(M1-1_FluidFlow)(R1) Page 21 of 40


School Of Mining Engineering Underground Mine Environment
 2003 University Of New South Wales Module 1.1 Fluid Flow, Friction And Shock Losses

The equation then reduces to;

Equation 3.7 Calculation Of λ From Surface Roughness

1 = 1.74 - 2 . Log10(2ε)
√λ

but , it has been shown that k = λ / 6.67. The equation relating k factor, roughness height (e mm) and
hydraulic diameter (Dh m) for turbulent flow regimes is then;

Equation 3.8 Calculation Of k From Surface Roughness

k= 1 2 Ns2/m8
1.74 - 2. Log10 2.e 6.67
Dh . 1000

The importance of this relationship is that, within an acceptable tolerance, Atkinson’s friction factor can
be calculated for any roughness height and airway profile in the flow regimes normally encountered.
Figure 3.4 shows k factor plotted against Reynolds number for various ratios of e/D.

Figure 3.4 Friction Factor K Ns2/m4 Versus Reynolds Number With Wall Roughness

Lim it of square law Mining


0.016
applications
0.014
Friction Factor K Ns2/m4

Turbulent

0.012
e/D
0.010 Transitional 0.05

0.008 0.025
0.006 0.01
0.004 0.004
Lam inar
0.001
0.002 0.0001
Smooth
0.000
1.0E+03 1.0E+04 1.0E+05 1.0E+06 1.0E+07
Reynolds Number
For most mining applications, the friction factor K is essentially constant for a given ratio of roughness
height to diameter and the square law applies.

Some approximate roughness heights mm are as follows;

Fibreglass ducts 0.2 to 2 mm


Raisebore holes 10 to 25 mm
Coal mine development 50 to 150 mm
Blasted development 150 to 350 mm
Longwall face 500 to 1000 mm

This equation is included in the module spreadsheet for calculation of airway resistance, refer
Appendix A.

File UNSW-UME-(M1-1_FluidFlow)(R1) Page 22 of 40


School Of Mining Engineering Underground Mine Environment
 2003 University Of New South Wales Module 1.1 Fluid Flow, Friction And Shock Losses

Worked Example – Roughness Height In Airways

Using Figure 4.4, determine the K factor for a 610 mm diameter fibre glass duct (rough = 0.3 mm) ?

For the duct ratio e/D = 0.0003 / 0.610 = 4.918 x 10-4 From figure 4.4 k = 0.0025 Ns2/m4

Calculate the K factor for a 3.0 x 5.0 m development profile (rough = 125 mm) ?

For the development

Cross sectional area = height x width = 3.0 x 5.0 = 15.0 m2

Perimeter = 2 x (height + width) = 2 x (3.0 + 5.0) = 16 m

Hydraulic diameter = 4.A = 4 x 15 = 3.75 m


C 16

Ratio e/D = 0.125 / 3.75 = 0.0333

1 = 1.74 - 2 Log10(2ε ) = 1.74 – 2 Log10 ( 2 x 0.0333) = 1.74 - -( 2.361) = 4.1


√λ

Therefore λ = 1/ (4.1)2 = 0.0595 but K = λ / 6.67

Therefore, K = 0.0595/6.67 = 0.00892 Ns2/m4

Worked Example – Roughness Height In Pipes

Calculate the K factor for a 105 mm ID steel pipe with a surface roughness of 0.4 mm ?

Diameter = 0.105 m

Ratio e/D = 0.0004 / 0.105 = 0.00381

1 = 1.74 - 2 Log10(2ε ) = 1.74 – 2 Log10 ( 2 x 0.00381) = 1.74 - -( 4.236) = 5.976


√λ

Therefore λ = 1/ (5.976)2 = 0.0280 but K = λ / 6.67

Therefore, K = 0.0280/6.67 = 0.00419 Ns2/m4

What is the head loss (m per 100 m) for a water flow rate of 30 l/s ?

Cross sectional area A = π.D2 A = π. x 0.1052 = 0.00866 m2


4 4

Circumference C = π.D C = π x 0.105 = 0.33 m

Resistance R = k.C.L . p R = 0.00419 x 0.00866 x 100 x 1000 = 177.7 x 106 Ns2/m8


A3 1.2 0.33 3 1.2

Pressure loss per 100 m at 30 l/s P = R.Q2 = 177.7 x 106 x 0.032 = 159,930 Pa

Head loss = Pressure/ p.g = 159,930 /(1000 x 9.81) = 16.3 m per 100 m

File UNSW-UME-(M1-1_FluidFlow)(R1) Page 23 of 40


School Of Mining Engineering Underground Mine Environment
 2003 University Of New South Wales Module 1.1 Fluid Flow, Friction And Shock Losses

3.1.2 Resistance Of Leakage Paths


The direction of leakage through ventilation devices will always be from higher absolute pressure side
to lower absolute pressure side. When measured by a manometer across a seal or stopping this will
be the observed gauge pressure.

If density effects (natural ventilation pressure) in the mine are small compared to frictional losses,
standard network gauge pressures can be used. For example, leakage will always be from intake to
return airways at pit bottom of a coal mine. However, this is not necessarily the case in multi level
mines where natural ventilation pressures may be of the same order of magnitude as internal frictional
losses.

The magnitude of leakage will depend on the applied pressure differential, perimeter stopping/seal
resistance and pressure quantity relationship P = R.Qn , where n can range 1.0 to 2.0. Typically the
value of n will be less than 2 for high resistance - low quantity leakage paths where flow regimes tends
towards laminar rather than turbulent conditions (Hemp 1998).

Some typical design leakage resistances are shown in Table 3.1, however these can be highly
variable in practice and should be confirmed by site surveys.

Table 3.1 Some Typical Leakage Path Resistances

Leak Res. Leakage flow Leakage flow


2 8
Item Ns /m m3/s per 500 Pa m3/s per 1000 Pa
Double doors 400 1.12 1.58
Conveyor belt seal 50 3.16 4.47
Overcast 1,000 0.71 1.00
Temporary stopping (poor) 200 1.58 2.24
Temporary stopping (good) 2,000 0.50 0.71
Permanent stopping (poor) 1,000 0.71 1.00
Permanent stopping (good) 10,000 0.22 0.32
Brick seal 20,000 0.16 0.22
Thick concrete seal 80,000++ 0.08 0.11

Worked Example – Leakage Through Seals

A coal mine goaf perimeter involves 6 seals with a leakage resistance of 20,000 Ns2/m8 and
resistance relationship P = R.Q1.7. If barometric pressure falls suddenly by 200 Pa, what rate of
leakage could be expected into the adjacent workings ?
1/1.7 0.59
Leakage rate per seal = P = 200 = 0.066 m3/s = 66 l/s per seal
R 20,000

Total leakage into the workings = 6 x 66 = 396 l/s.

File UNSW-UME-(M1-1_FluidFlow)(R1) Page 24 of 40


School Of Mining Engineering Underground Mine Environment
 2003 University Of New South Wales Module 1.1 Fluid Flow, Friction And Shock Losses

3.2. Shock Losses Due To Circuit Discontinuities

In addition to frictional pressure loss, resulting from airflow over rough surfaces, the degree of flow
separation occurring at discontinuities leads to the concept of shock loss and shock loss factors.

The static pressure loss due to flow separation depends on air velocity, relative magnitude of the
change in velocity and degree to which flow separation occurs, refer Figure 3.5.

Figure 3.5 Shock Losses Occurring At Discontinuities


Inlet Losses Discontinuities Outlet losses
Flow Surface
separation friction

Accelaration

Decelaration

It is normally immediately apparent which shapes will incur the highest losses, for example bellmouth
inlet compared with flanged inlet or gradual rather than abrupt expansion/contraction. The static
pressure loss is calculated as a fraction of the velocity pressure at the discontinuity by applying a
shock loss factor X, as follows;

Equation 3.9 Shock Loss Factor And Velocity Pressure

Pressure loss p = X. 1 . p . V2 Pa
due to shock 2

Shock loss factors have been determined for most discontinuities encountered by empirical methods,
scale models or, more recently, by application of computational fluid dynamic programs. Shock losses
can therefore be calculated for any fluid of known density and velocity.

In practice, shock loss factors are obtained from design charts specific to particular component
geometry. When using these charts it is important to note to which velocity pressure (inlet or outlet) the
shock loss factor is applied.

It is also the case that the shock loss factor may be greater than 1.0 for some components. This
occurs, for example in bends, where the shock loss factor accounts for all turning, expansion and
contraction losses that exist i.e a combined overall shock loss factor from inlet to outlet. Shock loss
factors for some common components are shown in Figure 3.6.

When considering static and velocity pressures in a system of varying cross sectional area, it is also
important to account for changes in static pressure resulting from increased or reduced velocity. This
effect is superimposed on any frictional or shock loss that may occur.

File UNSW-UME-(M1-1_FluidFlow)(R1) Page 25 of 40


School Of Mining Engineering Underground Mine Environment
 2003 University Of New South Wales Module 1.1 Fluid Flow, Friction And Shock Losses

Figure 3.6 Some Common Shock Loss Factors

Figure 3.6a Inlet loss


Sharp edge Flanged

Square X = 1.25 X = 0.5


Circular X = 0.90 X = 0.7
Figure 3.6c Bend of uniform cross
Figure 3.6b Abrupt expansion or sectional area
contraction Note X can be greater than 1.0

A1 A2 A1 A2
α

Expansion A2/A1 Contraction A1/A2

1.0 3
Shock Factor X (Small area velocity)

0.9
0.8
0.7
2
Shock Factor X

0.6
0.5
0.4
0.3 1
0.2 90 deg
0.1 mitre bend
0.0 0
1 2 3 4 5 6 0 20 40 60 80 100 120
Ratio Large Area / Small Area Angle deg

Figure 3.6d Junction and intersections Figure 3.6e Outlet loss with diffusers
Same cross sectional area
V1 V2 D2
D1 α

V3

4.5 0.45 1.0 α


1 to 3 1 to 2
4.0 0.40 0.9 50
Shock Factor X Inlet 1 to branch 3

Shock Factor X Inlet 1 to outlet 2

Shock Factor X (D1velocity)

3.5 0.35 0.8 40


3.0 0.30 0.7
0.6 30
2.5 0.25
0.5 20
2.0 0.20
0.4
1.5 0.15
0.3 Outlet shock = VP 7
1.0 0.10 0.2
0.5 0.05 0.1
0.0 0.00 0.0
0.0 0.2 0.4 0.6 0.8 1.0 1.2 1 1.2 1.4 1.6 1.8 2
Ratio Branch velocity V3 / Inlet velocity V1 Ratio Large Diameter D1 / Small Diameter D2

File UNSW-UME-(M1-1_FluidFlow)(R1) Page 26 of 40


School Of Mining Engineering Underground Mine Environment
 2003 University Of New South Wales Module 1.1 Fluid Flow, Friction And Shock Losses

Worked Example – Shock Losses

8.0 m3/s of air in a 610 mm duct flows through a 45O bend. The density of air is 1.16 kg/m3 and the
observed static pressure at A is –1250 Pa gauge.

What is the velocity, static and total pressure at points A and B ?

A
Area of 610 mm duct = 0.292 m2 α
Velocity of air in duct = Q/A = 8.0 / 0.292 = 27.4 m/s B

Velocity pressure = VP = ρ.v2 = 1.16 x 27.42 = 435 Pa


2 2

At point A, static pressure = -1250 Pa and velocity pressure = 435 Pa

Total pressure = static pressure + velocity pressure = -1250 + 435 = -815 Pa

Shock loss factor X for a 45 deg bend = 0.4 from Figure 4.6c

Shock loss pressure due to bend = X. p.v2 = 0.4 x 435 = 174 Pa


2

Velocity pressure at B is the same as no change in quantity or cross sectional area has occurred VP = 435 Pa

Static pressure at B has reduced due to shock loss by 174 Pa therefore SP = -1250 – 174 = -1424 Pa

Total pressure at B = SP + VP = -1424 + 435 = -989 Pa

Note that change in total pressure = -989 - - (815) = -174 Pa = shock loss

9.0 m3/s of air flows through an expansion from 610 mm to 900 mm diameter. Air density is 1.16 kg/m3 ,
the static pressure at A is +930 Pa and the loss through the expansion is 45 Pa. What is the velocity,
static and total pressure at points A and B ?

Area of 610 mm duct = 0.292 m2


Area of 900 mm duct = 0.636 m2
A B
Velocity of air in duct = Q/A At A = 9.0 / 0.292 = 30.8 m/s
At B = 9.0 / 0.636 = 14.2 m/s

Velocity pressure = VP = p.v2 At A = 1.16 x 30.82 = 550 Pa


2 2

At B = 1.16 x 14.22 = 117 Pa


2

Total pressure at A = SP + VP = 930 + 550 = + 1480 Pa

Static pressure at B has reduced due to shock loss by 45 Pa but static pressure at B has also increased due to a
reduction in velocity pressure of 550- 117= 433 Pa

Static pressure at B = 930 – 45 + 433 = 1318 Pa

Total pressure at B = SP + VP = 1318 + 117 = 1435 Pa

Note change in total pressure = 1435 - 1480 = -45 Pa = shock loss

File UNSW-UME-(M1-1_FluidFlow)(R1) Page 27 of 40


School Of Mining Engineering Underground Mine Environment
 2003 University Of New South Wales Module 1.1 Fluid Flow, Friction And Shock Losses

Worked Example – Duct Resistance And Shock Losses

Calculate the static, velocity and total (gauge) pressures at key points in the following duct circuit for an
3 3
air quantity of 7 m /s and a density of 1.17 kg/m ?
40 m 3
2 m /s Duct G
50 m
50 m G 400 mm diam
2 4
Atmosphere K = 0.0040 Ns /m
3
7 m /s 40 m
A B C D
E α = 50O 30 m
F 3
5 m /s
Duct A to B F’
750 mm diam Duct C to D to H
2
K = 0.0035 Ns /m
4 H Atmosphere
610 mm diam through fan
2 4
K = 0.0040 Ns /m

The first step is to calculate the frictional loss and velocity pressure in each component because these terms are
independent. R = k.C.L . p 2
A
3
1.2 2 VP = p.v
A and C from D P = R.Q V = Q/A
2
Diameter Area Circum K Factor Length Resistance Quantity Frictional Velocity Velocity
Duct mm m2 m Ns2/m4 m Ns2/m8 m3/s Loss Pa m/s Pressure Pa
A to B 750 0.442 2.356 0.0035 40 9.3 7 457 15.8 147
C to D 610 0.292 1.916 0.004 50 29.9 7 1467 24.0 336
E to F 610 0.292 1.916 0.004 50 29.9 7 1467 24.0 336
F to G 400 0.126 1.257 0.004 40 247.0 2 988 15.9 148
F' to H 610 0.292 1.916 0.004 30 29.9 5 749 17.1 171

The next step is to calculate the shock loss factor and pressure drop due to discontinuities.

2
P =X p.v
Factor Velocity Shock 2
X Pressure Pa Loss Pa
Atmos to A Sharp edge 0.9 147 132
B to C Area ratio 1.51 0.1 336 34
D to E Angle 50.00 0.4 336 134
F to G Velocity ratio 0.66 2.25 336 755
F to F' Velocity ratio 0.66 0.12 336 40
H to atmos Straight outlet 1 171 171
Then calculate the static and total pressure at each point due to losses and change in velocity that occur.
VP SP TP
Atmosphere 0 0 0
The results can be plotted in order to identify principle losses
Change 147 -132 Inlet
and relationships. In this case the frictional loss from C to F is
A 147 -279 -132
highest. A fan with a total pressure of 4635 Pa and a static
Change 0 -457 Friction
pressure of 4464 Pa would be required.
B 147 -736 -589
Change 189 -34 Contraction VP SP TP
1000
C 336 -958 -623
Change 0 -1467 Friction
D 336 -2426 -2090 0
Change 0 -134 Bend
E 336 -2560 -2224
Gauge Pressure Pa

Change 0 -1467 Friction -1000


F 336 -4027 -3691
Change -187 -755 Intersection
-2000
G 148 -4595 -4447

Change -164 -40 Intersection -3000


F' 171 -3903 -3732
Change 0 -749
H 171 -4652 -4480 -4000
Change 4652 Fan required
Atmosphere 171 0 171
-5000

File UNSW-UME-(M1-1_FluidFlow)(R1) Page 28 of 40


School Of Mining Engineering Underground Mine Environment
 2003 University Of New South Wales Module 1.1 Fluid Flow, Friction And Shock Losses

3.3 Pressure Losses Due To Airway Obstructions

Ducted systems, underground travel roads and exhaust shafts are generally free of significant
obstructions. Pressure losses are then calculated from surface friction and shock factors. In other
airways, such as intake shafts with cages or those with mine conveyor belts, a significant pressure
loss can also be associated with the degree and type of “fill” in place. In this instance, an effective K
factor could be applied to the airway from direct measurement but it may be necessary to predict
losses at the design stage of a project. This section describes methods of calculating pressure losses
due to airway obstructions.

Airway obstructions comprise longitudinal objects, such as pipes or guides, and transverse objects,
such as shaft buntons or conveyor belt hangers, refer Figure 3.7.

Figure 3.7 Pressure Loss Due To Obstructions In An Airway

Frontal area of
transverse Area and perimeter of
obstructions Ao longitudinal obstructions

Velocity v
Surface
roughness

Airway area A
and perimeter P

Losses arise from flow separation around obstructions, dependent on their size and shape, in a similar
manner to shock losses associated with discontinuities. The pressure loss due to a transverse
obstruction is determined by it’s drag coefficient Cd, frontal area Ao and velocity pressure impacting
the frontal surface, as follows;

Equation 3.10 Pressure Loss Due To Drag Coefficient Of Obstruction

Pressure loss p =Cd . 1 . p.V2 . Ao Pa


due to obstruction 2 A

Equation 3.10, indicates that the pressure loss due to a transverse obstruction is dependent on it’s
shape (Cd), it’s size (Ao) and the size of the airway in which it is placed (A).

The ratio Ao/A is known as the coefficient of fill Cf. Using the relationship V = Q/A, the equation can
then be rewritten to express the pressure loss in terms of quantity, as follows;

Equation 3.11 Pressure Loss Due To Drag Coefficient Of Obstruction

Pressure loss p =Cd . Cf . 1 . p. Q2 Pa


due to obstruction 2 A2

Drag coefficients for common shapes of transverse obstructions are shown in Figure 3.8.

Figure 3.8 Drag Coefficients For Common Structural Members in Shafts And Airways
For infinite length

Stream
I Beam Square Rectangle Circular Plate Lined Flanges
Drag Coeff Cd 2.60 2.00 1.50 1.20 1.95 0.80 1.00

File UNSW-UME-(M1-1_FluidFlow)(R1) Page 29 of 40


School Of Mining Engineering Underground Mine Environment
 2003 University Of New South Wales Module 1.1 Fluid Flow, Friction And Shock Losses

It has been found by experimentation that drag losses due to longitudinal objects are small, and in
some cases actually reduce total shaft losses by providing a straightening effect on flow. Longitudinal
obstructions are accounted for by subtracting their total cross sectional area from the area of the
airway and including their perimeter in calculation of surface frictional loss.

The calculation of pressure loss due to transverse obstructions of given geometry, requires that the
base case drag coefficients (infinite length) are corrected to account for the effects of geometry. These
are obtained from the ratios of height to width CHW and spacing to width CSW, refer Figure 3.9. The
corrected drag coefficient to be used in equation 3.10 or 3.11, is then given by;

Equation 3.12 Correction Of Drag Coefficients

Corrected drag coefficient Cd’ = Cd x CHW x CSW

Figure 3.9 Factors Affecting Base Case Drag Coefficients


(From Chasteau & Gillard 1965, Greuer 1961, Howes 1981)
Figure 3.9 A Correction for Height
To Width Ratio
1.00

Cd increases with L 0.95


length for a given width

Correction Factor C HW
0.90

0.85

W 0.80

Cd reduces with H 0.75


increased H/W ratio
until surface friction Cd reduces with closer
0.70
becomes significant. spacing due to mutual
S interference
1.0 1.5 2.0 2.5 3.0 3.5 4.0 4.5 5.0
Height To Width Ratio H/W

Figure 3.9 B Correction for Spacing


To Width Ratio

1.00
0.90
0.80
Correction Factor C SW

0.70
Direction of
airflow 0.60
0.50
The drag coefficient for a combination of structural members in 0.40
an airway is obtained by summing the corrected drag 0.30
coefficients for each member and applying a weighted average 0.20
to the free area of the airway.
0.10
0.00
0 10 20 30 40 50 60
Spacing To Width Ratio S/W

Methods for the analysis of losses in shafts, including those due to moving conveyances, are covered
in more detail in the references supplied with the notes, McPherson,1988.

File UNSW-UME-(M1-1_FluidFlow)(R1) Page 30 of 40


School Of Mining Engineering Underground Mine Environment
 2003 University Of New South Wales Module 1.1 Fluid Flow, Friction And Shock Losses

Worked Example – Pressure Loss Due To Obstructions

A 2.5 km long coal mine gate road is to be supported with concrete filled pipes during longwall retreat.
The pipes are 800 mm diameter and are located at 4.0 m spacing for 500 m out bye of the face line. The
3 3
airway is 2.5 m high and 5.4 m wide, the airflow rate is 45 m /s and the air density s 1.2 kg/m .

Calculate the effect of installing this support on pressure loss ?


Face Line

Base drag coefficient for cylinder = 1.2 4.0 m 500 m


spacing
Width of cylinder = diameter = 0.8 m
5.4 m
Length of cylinder = height = 2.5 m

Frontal area = width x length = 0.8 x 2.5 = 2.0 m2 0.8 m 3.0 m


diam 1500 m
Ratio spacing/width = 4.0/0.8 = 5.0

Correction factor CSW = 0.20 3


45 m /s

For cylinders, the ratio of height to width = 1.0 therefore CHW =1.0

Corrected drag coefficient = Cd x CHW x CSW = 1.2 x 1.0 x 0.2 = 0.24

Cross sectional area of airway = height x width = 2.5 x 5.4 = 13.5 m2

Coefficient of fill Cf = Ao/A = 2.0/13.5 = 0.148

Pressure loss per pipe p =Cd . Cf . 1 . p. Q2 = 0.24 x 0.148 x 1.2 x 452 = 0.234 Pa
2 A2 2 x 13.52

Number of pipes = distance/spacing = 500 /4.0 = 125

Pressure loss due to 500 m of pipe supports = 125 x 0.234 = 30 Pa

Perimeter of airway = 2.(height + width) = 2 x (2.5 + 5.4) = 15.8 m

Resistance per 100 m R100 = p = k.C.L. p = 0.009 x 15.8 x 100 x 1.2 = 0.00578 Ns2/m8
A3 1.2 13.53 1.2

Pressure loss in 1500 m of airway with no support = 0.00578 x 1500 x 452 = 175 Pa
100

Pressure loss in 500 m due to airway resistance alone = 0.00578 x 500 x 452 = 59 Pa
100
Pressure loss in 500 m with support = 30 + 59 = 89 Pa

In this example, the effect of installing support is to increase the pressure loss in the 500 m section of airway.
However, the pressure loss of 30 Pa is numerically small due to the low airway velocity and significant mutual
interference between support.

File UNSW-UME-(M1-1_FluidFlow)(R1) Page 31 of 40


School Of Mining Engineering Underground Mine Environment
 2003 University Of New South Wales Module 1.1 Fluid Flow, Friction And Shock Losses

3.4 Equivalent Length And Effective Airway Resistance


It is sometimes convenient, for the purposes of calculation or entry of data into computer simulation
programs, to include shock losses directly into the calculation of airway resistance. If the resistance of
100 m of straight airway is known (R100) shock losses can be included as an ”equivalent length”. The
equivalent length of a shock loss component is defined as being the length of airway that would result
in the same frictional pressure loss. For example, if R100 = 0.02 Ns2/m8 and the equivalent length of a
bend is 25 m, the resistance of a 200 m long airway = 0.02 x (200+25)/100 = 0.045 Ns2/m8.

An alternative method is to combine shock loss with the resistance as a separate term. This maintains
the actual airway length but requires that an artificial resistance is used.

The derivation of both methods is shown below;

Effective Resistance Equivalent Length


Shock loss p = X. p . v2 Pa Equating Atkinson’s equation in terms of velocity
2 with the shock pressure loss;

and, velocity v = Quantity = Q m/s p = K.C.L . ρ . v2 = X. ρ. v2


Area A A 1.2 2
by substitution Therefore K.C.L = X
1.2 A 2
Shock loss Ps = X . 1 . p . Q2 Pa
2 A2

Equation 4.13 Shock Loss As An Effective Resistance Equation 4.14 Shock Loss As An Equivalent Length

Reff = R + X . 1 . p Ns2/m8 Leq = X 1.2 A = 0.6 X.A m


2. A2 2.K.C K.C

Worked Example – Equivalent Length And Effective Resistance

The K factor for a 3.0 m x 5.0 m airway profile is 0.012 Ns2/m4 . A single bend has a shock loss factor of
X=0.3 and the air density is 1.2 kg/m3. The air quantity is 160 m3/s.

Calculate the pressure loss for 100 m of airway using the equivalent length method ?

Cross sectional area = height x width = 3.0 x 5.0 = 15.0 m2

Perimeter = 2 x (height + width) = 2 x (3+5) = 16.0 m

Equivalent length of bend = 0.6 . X.A = 0.6 x 0.3 x 15 = 14.06 m


K.C 0.012 x 16

Resistance per 100 m R100 = p = k.C.L. p = 0.012 x 16 x 114.06 x 1.2 = 0.00649 Ns2/m8
A3 1.2 15.03 1.2

Pressure loss = R . Q2 = 0.00649 x 1602 = 166 Pa

Calculate the pressure loss for 100 m of airway using the effective resistance method ?

Resistance per 100 m R100 = p = k.C.L. p = 0.012 x 16 x 100 x 1.2 = 0.00569 Ns2/m8
A3 1.2 15.03 1.2

Effective resistance Reff = R + X 1. p = 0.00569 + 0.3 x 1.2 = 0.00649 Ns2/m8 ie the same as above
2. A2 2 x 152

File UNSW-UME-(M1-1_FluidFlow)(R1) Page 32 of 40


School Of Mining Engineering Underground Mine Environment
 2003 University Of New South Wales Module 1.1 Fluid Flow, Friction And Shock Losses

4.0 BERNOULLI’S EQUATION


In the absence of fans or frictional losses, there is assumed to be no change in total pressure which
remains constant between any two points in a flow system. Consequently, if the velocity pressure
increases or decreases due to a change in cross sectional area, and the fluid is incompressible, there
must be an equivalent and opposite change in static pressure.

Bernoulli’s equation is a statement of this “conservation” of total pressure when ideal incompressible
fluid behaviour is assumed and there is no pressure loss (friction) or pressure gain (fans).

Individual terms of Bernoulli’s equation are required in order to predict the behaviour of ventilation
circuits or measure their performance. All pressure terms can be directly or indirectly measured and it
has been shown that velocity pressure can be calculated from the velocity and density of a fluid . That
is, the velocity pressure can be accurately predicted knowing quantity and dimensions of the circuit
airways or ducts. Pressure losses due to friction and pressure gains due to circuit fans are then
required in order to calculate changes in static pressure.

Equation 4.1 Bernoulli’s Equation P1 VP1 h1


Total pressure at 1 = Total pressure at 2 = constant
P1 + VP1 + g. ρ.h1 = P2 + VP2 + g. ρ.h2 = constant
or, rearranging P2 VP2 h2

P2 = P1 + (VP1 – VP2) + g. ρ. (h1 - h2 )

Equation 4.1 simply states that the pressure at point 2 is given by the pressure at point 1 plus/minus
change in velocity pressure plus/minus the change in pressure due to a change in elevation. Note that,
by convention, elevation is measured positive upwards.

In reality, changes in total pressure occur due to frictional losses and circuit fans. If a fan is placed
between point 1 and 2, the pressure at point 2 will increase. If frictional losses are included, the
pressure at point 2 will reduce. The statement of conservation of pressure can then be written as
follows;

Equation 4.2 Bernoulli’s Equation With Friction And Fan Pressure Terms
Pfan P1 VP1 h1
Total pressure in = Total pressure out Pfric
P1 + VP1 + g. ρ.h1 + Pfan = P2 + VP2 + g. ρ.h2 + Pfric

or, rearranging
P2 VP2 h2
P2 = P1 + (VP1 – VP2) + g. ρ. (h1 - h2 ) +Pfan - Pfric

The following worked example applies Bernoulli’s equation to the most common flow configurations
encountered. For this example, the start pressure P1 is 101,000Pa and, when applied, the change in
velocity pressure is 45Pa, the frictional loss is 225Pa, the change in pressure due to elevation is
+2,010 Pa and the fan pressure is 2,600Pa.

File UNSW-UME-(M1-1_FluidFlow)(R1) Page 33 of 40


School Of Mining Engineering Underground Mine Environment
 2003 University Of New South Wales Module 1.1 Fluid Flow, Friction And Shock Losses

Worked Example – Bernoulli’s Equation

Application to inclined airway, no flow

P2 = P1 + ρ (V12 – V22) + ρg (h1 – h2) - Pfric + Pfan


2
h1 V1 Pfric
V1= V2 Pfan = 0 Pfric = 0
P1
P2 = P1 + ρg (h1 – h2) V2
h2
P2 = 101,000 + 2,010 = 103,010 Pa P2

Application to horizontal airway, constant cross section with no fan

P2 = P1 + ρ (V12 – V22) + ρg (h1 – h2) - Pfric + Pfan


2
Pfric
V1= V2 h1 = h2 Pfan = 0
V1 V2
h1 h2
P2 = P1 - Pfric P1 P2

P2 = 101,000 – 225 = 100,775 Pa

Application to inclined airway, constant cross section with no fan

P2 = P1 + ρ (V12 – V22) + ρg (h1 – h2) - Pfric + Pfan


2

V1= V2 Pfan = 0 h1 V1 Pfric


P1
P2 = P1 + ρg (h1 – h2) - Pfric
V2
h2
P2 = 101,000 + 2,010 – 225 = 102,785 Pa P2
Nb elevation h is measured positive upwards.

Application to horizontal airway, constant cross section with fan

P2 = P1 + ρ (V12 – V22) + ρg (h1 – h2) - Pfric + Pfan


2
Pfan Pfric
V1= V2 h1 = h2
V1 V2
h1 h2
P2 = P1 - Pfric + Pfan P1 P2
P2 = 101,000 – 225 + 2,600 = 103,375 Pa

File UNSW-UME-(M1-1_FluidFlow)(R1) Page 34 of 40


School Of Mining Engineering Underground Mine Environment
 2003 University Of New South Wales Module 1.1 Fluid Flow, Friction And Shock Losses

Application to horizontal airway, changing cross section with no fan

P2 = P1 + ρ (V12 – V22) + ρg (h1 – h2) - Pfric + Pfan


2
Pfric
h1 = h2 Pfan = 0
V1 V2
h1 h2
P2 = P1 + ρ (V12 – V22) - Pfric P1 P2
2

P2 = 101,000 – 45 - 225 = 100,730 Pa

Note that, in this case, the velocity pressure has increased by 45 Pa therefore the static pressure at
point 2 reduces by 45 Pa in addition to frictional losses.

4.1 Application To Trailing Hose Survey


Trailing hose surveys are used to directly measure differential pressures between two points that are
at some distance (100 to 300 m) from each other or across ventilation devices. The usual application
of this technique is for measuring frictional losses in airways where a significant distance between
points is required to obtain a measurable pressure differential. Consider an inclined airway of constant
cross sectional area, the tube is run between two points and a manometer is connected to one end.
There is therefore no flow within the tube.

P2 = P1 + ρ (V12 – V22) + ρg (h1 – h2) - Pfric + Pfan


2 Pfric
h1 V1
For the airway V1= V2 and Pfan = 0 P1
P2 = P1 + ρg (h1 – h2) - Pfric V2
Pfric P1tube h2
P2
For the tube V1= V2= 0 Pfric = 0 Pfan = 0
P2tube
P2tube = P1tube + ρg (h1 – h2)

P2tube = P2 because the tube is open and protected from influence of velocity. Therefore,

P1tube + ρg (h1 – h2) = P1 + ρg (h1 – h2) - Pfric

The static head pressures are the same and cancel. The manometer measures the difference
between P1and P1tube.

Manometer pressure differential = P1 - P1tube = Pfric ie the frictional pressure loss.

Note that the change in static pressure occurs in both the airway and the tube. Providing the tube is
left to stabilise to exactly the same density as that in the airway, this change cancels out and for
airways of essentially constant cross sectional area, the observed pressure differential is the frictional
pressure loss.

File UNSW-UME-(M1-1_FluidFlow)(R1) Page 35 of 40


School Of Mining Engineering Underground Mine Environment
 2003 University Of New South Wales Module 1.1 Fluid Flow, Friction And Shock Losses

4.2 Application To Traverse Barometer Surveys


A barometer measures the absolute static pressure at a given point and therefore is influenced by all
terms of the Bernoulli equation when moved between locations. To calculate the frictional pressure
loss between two points, the observed pressures must be corrected for changes in elevation and
changes in barometric pressure during the traverse. This last correction is normally obtained by
recording atmospheric pressure at a stationary point while the traverse barometer is being moved.

For constant density;


h1 V1 Pfric
P2 = P1 + ρ (V12 – V22) + ρg (h1 – h2) - Pfric + Pfan P1
2 Station 1 V2
h2
Assuming V1= V2 and Pfan = 0 P2

P2 = P1 + ρg (h1 – h2) - Pfric Station 2

P2 and P1 are measured by the barometer separately at known times and are then corrected for any
change in barometric pressure due to surface climatic conditions P2* and P1*

To account for changing density, the correction applied for a change in surface barometer (Pc) is

i.e the change in surface pressure ∆Pc is corrected by the ratio of


P* = P - P ∆Pc absolute pressure and if the surface control barometer increases, the
Pc underground pressure P is reduced.

Pfric = P1* - P2*+ ρg (h1 – h2) When the density at station 1 differs from station 2, for example in a
shaft, then the average density must be used and the equation
becomes;

Pfric = P1’ - P2’+ g (ρ1 + ρ2) ∆h Note that there will be a ≈11 Pa error in pressure reading per 1.0 m
2 error in elevation. Density is assumed to vary linearly between station
1 and station 2.

Worked Example – Trailing Hose And Traverse Barometer Surveys

The manometer attached to a trailing hose in a mine airway reads 15 mm H2O. What is the frictional loss
between the ends of the hose ?

Frictional loss = observed pressure = ρm.g.hm = 1,000 x 9.81 x 0.015 = 147 Pa

A traverse barometer reads 101,250 Pa on surface and 103,890 Pa at the bottom of a 250 m deep access
drift. What is the frictional loss in the drift if air density on surface is 1.18 kg/m3 and air density at the
bottom of the shaft is 1.22 kg/m3 .The surface barometric pressure increased by 65 Pa between the two
readings ?

Correcting second reading for change barometric pressure P2 = 103.890 – 65 x 103,890 = 103,823 Pa
101,250

Actual change in absolute pressure = 103,823 – 101,250 = 2,573 Pa

Change in pressure due to air column = ρ.g.h = (1.18 +1.22) x 9.81 x 250 = 2,943 Pa
2

That is, without frictional losses, the pressure at the bottom of the shaft would be 2,943 Pa greater than on
surface. But, due to frictional losses, it is in fact only 2,573 Pa greater.

Frictional loss in shaft = P2 – P1 – average column pressure = 103,823 – 101,250 – 2,943 = -370 Pa

File UNSW-UME-(M1-1_FluidFlow)(R1) Page 36 of 40


School Of Mining Engineering Underground Mine Environment
 2003 University Of New South Wales Module 1.1 Fluid Flow, Friction And Shock Losses

4.3 Application To Orifice Plates And Regulators


The change in static pressure with change in velocity pressure is used to measure flow rates in orifice
plates, refer Figure 4.1

Figure 4.1 Flow Through An Orifice Plate


D1 D1/2

A1 A2
V1 A3
V2 Q = A1.V1 = A2.V2
P1 V3 D1 D2
P2

Vena contracta

P1 + ρV12 + ρ g h1 = P2 + ρV22 + ρ g h2 = constant for any airway section assuming no losses occur.
2 2

assume negligible change in elevation and that density is constant,

P1 + ρV12 = P2 + ρV22 therefore the observed differential pressure ∆P = ρ (V22 - V12) Pa


2 2 2

This simply states that, in the absence of friction or shock losses, the observed change in static
pressure equals the change in velocity pressure. Therefore, flow rate can be calculated from the
observed change in static pressure, cross sectional area and fluid density.

Two corrections are required to account for actual flow regime and loses in an orifice plate;

The coefficient of velocity Cv is defined as the ratio between the actual average velocity of the jet at
the vena contracta and the theoretical velocity which would have existed had there been no frictional
loss at the orifice. Cv has a value of approximately 0.97 for orifice plates.

The coefficient of contraction Cc is defined as the ratio between the area of the jet at the vena
contracta and the area of the orifice or A3/A2.

The total coefficient of discharge Cd ranges 0.6 to 0.61 for orifice plates with area ratios from 0.1 to
0.6.

By substitution and rearrangement of the equations, flow rate through the orifice is calculated from
observed static pressure differential using;

Equation 4.3 Orifice Plate Equation

Q = A2.Cd 2 ∆P m3/s, where m = A2 = D22


ρ(1 – m2) A1 D12

File UNSW-UME-(M1-1_FluidFlow)(R1) Page 37 of 40


School Of Mining Engineering Underground Mine Environment
 2003 University Of New South Wales Module 1.1 Fluid Flow, Friction And Shock Losses

This equation can also be applied to mine regulators in order to estimate the size of opening required
for a specified pressure reduction at a given ventilation rate.

Equation 4.4 Orifice Plate Equation Applied To Regulators

A2 = Q . ρ(1 – m2) m2
Cd 2 ∆P

For practical applications, and recognising the tolerance of measurements being undertaken, Le Roux
(1990) combines the Cd , (1- m2)0.5 and (1/2)0.5 terms by assuming Cd = 0.6 and m = 0.1 (10 %). The
equation then reduces to;

Equation 4.5 Approximate Equation For Regulator Areas

A2 ≈ 1.2 Q. ρ m2 Q ≈ A2 ∆P m3/s
∆P 1.2 ρ

Note that, when measuring the actual pressure drop across regulators, ends of the trailing hose must
be placed so as to avoid the effect of vena contracta. Unlike ventilation doors or stoppings, the return
side tube should be placed some 3 to 4 diameters away so that velocity regain is complete. The
effective coefficient of discharge is best determined by site measurement for the type of regulator
used.

Worked Example – Orifice Plates And Regulators

A 50 mm orifice plate (Cd = 0.605) is being used to measure flow rates in a 100 mm pipe from a gas
drainage hole. The manometer reads 16 mm Hg and the density of the gas mixture is 0.92 kg/m3

What is the flow rate in the pipe (l/s)?

m = D22/D12 = 502/1002 = 0.25

Area of orifice = π D22 = π.0.052 = 0.001963 m2


4 4

Observed pressure differential = ρ g h = 13,600 x 9.81 x 0.016 = 2,134 Pa

Gas flow Q = A2.Cd 2 ∆P = 0.001963 x 0.605 2 x 2,134 = 0.083 m3/s = 83 l/s


ρ(1 – m2) 2
0.92(1 – 0.25 )

A regulator is to be placed in a return airway with a plan quantity of 60 m3/s and a required pressure
differential of 940 Pa. If air density is 1.16 kg/m3, estimate the opening required in the regulator ?

Regulator area A2 ≈ 1.2 Q. ρ = 1.2 x 60 1.16 = 2.5 m2


∆P 940

File UNSW-UME-(M1-1_FluidFlow)(R1) Page 38 of 40


School Of Mining Engineering Underground Mine Environment
 2003 University Of New South Wales Module 1.1 Fluid Flow, Friction And Shock Losses

5.0 REFERENCES

5.1 References Underpinning This Module


Le Roux W.L. 1990 “Le Roux’s Notes On Mine Environmental Control Ch.1 Properties Of Air”
Ch.3 Airflow
Published by Mine Ventilation Society Of South Africa

5.2 References Contained In The Course Notes


Daly, B. 1978 “Airflow And Pressure In Ducted Systems”
In Woods Practical Guide To Fan Engineering

“Various Friction Factors And Shock Losses”


Source Unknown

McPherson, M.J 1988 “An Analysis Of The Resistance And Airflow Characteristics Of Mine Shafts”
4th International Mine Ventilation Congress, Brisbane

Prosser, B.S and Wallace, K.G “Practical Values Of Friction Factors”


MVS Engineering Web Site

5.3 Further References


Bromilow J.G 1961 “The Estimation And The Reduction Of The Aerodynamic Resistance Of Mine
Shafts”. Journal Of The Mine Ventilation Society Of South Africa, Nov 1961 pp 201-222

Chasteau V.A.L 1982 “Environmental Engineering In South African Mines –Ch.1 Fundamentals Of
Fluid Flow”. Published by Mine Ventilation Society Of South Africa.1982

Chasteu V.A.L and Gillard D.1965 “The Prediction Of The Resistance To Airflow Of Mine Shafts
Equipped With Internal Structures” Journal Of The Mine Ventilation Society Of South Africa, Oct 1965
pp 133-145

Deglon P. & Hemp R. 1992 “An Evaluation Of Parameters To Be Used In Colliery Ventilation
Planning” 5th Int Mine Vent Cong, Johannesburg, 1992.

Greuer R.E.A 1961 “The Determination Of Mine Shaft Resistance To Airflow By The Mine Ventilation
Research Centre At Essen”. Journal Of The Mine Ventilation Society Of South Africa, May 1965 pp
61-69

Hemp R. 1982 “Environmental Engineering In South African Mines –Ch.2 Thermodynamic Aspects Of
Mine Airflow”. Published by Mine Ventilation Society Of South Africa.1982

Hemp R. 1998 ”The Effect Of Changes In Barometric Pressure On Mines In The Highveld Of South
Africa” Journal Of The Mine Ventilation Society Of South Africa April 1998

5.4 Acknowledgement
Howes M J. 1981.”Environmental Engineering Course Notes”
Significant assistance was gained for the content and format of this module from Environmental
Engineering course notes written and compiled by Dr M.J. Howes.

File UNSW-UME-(M1-1_FluidFlow)(R1) Page 39 of 40


School Of Mining Engineering Underground Mine Environment
 2003 University Of New South Wales Module 1.1 Fluid Flow, Friction And Shock Losses

APPENDIX A FRICTIONAL LOSS SPREAD SHEET

The following spreadsheet combines surface roughness, shock losses and parallel airways to enable
calculations of pressure loss with flow rate to be undertaken for any parallel airway configuration.

A B C D E F G H I J K L M N O P Q
2 Length Height Width Area Perim Density Rough k Resis(1) X No R(total) Q V DP
3 m Diam m2 m kg/m3 mm Ns2/m4 Ns2/m8 Ns2/m8 m3/s m/s Pa
4 Shaft 100 5 2 10.0 14.0 1.2 50 0.00693 0.00970 1.0 1 0.01570 300.0 30.0 1413

=IF(E4>0,E4*D4,PI()*D4^2/4) =N4*O4^2
=IF(E4>0,2*(E4+D4),PI()*D4)
=O4/F4/M4
=(1/(1.74-2*LOG10(2*I4/(1000*4*F4/G4))))^2/6.67 =(K4+L4*0.5*H4/F4^2)/M4^2

=J4*G4*C4/F4^3*(H4/1.2)

Cell Comment
B:4 Description of airway
C:4 Enter airway length, m
D:4 Enter height or diameter, m
E:4 Enter width m or 0 if D:4 = airway diameter
F:4 Calculates area from height and width or diameter, m2
G:4 Calculates perimeter from height and width or diameter, m2
H:4 Enter density, kg/m3 . Note that this can apply to liquids as well as gases
I:4 Enter average roughness height of airway, mm
J:4 Calculates k factor using equation 3.8, Ns2/m4
K:4 Calculates resistance due to surface roughness in single airway, Ns2/m8
L:4 Enter shock loss factor for airway
M:4 Enter number of parallel airways
N:4 Calculates total effective resistance, Ns2/m8
O:4 Enter quantity, m3/s
P:4 Calculates velocity in airway(s), m/s
Q:4 Calculates pressure loss, Pa

The following examples can be used to test the spreadsheet.

Length Height Width Area Perim Density Rough k Resis(1) X No R(total) Q V DP


m Diam m2 m kg/m3 mm Ns2/m4 Ns2/m8 Ns2/m8 m3/s m/s Pa
Shaft 100 6 28.3 18.8 1.2 50 0.00534 0.00045 1.0 1 0.00120 300.0 10.6 108
Airway 100 3 5 15.0 16.0 1.2 200 0.01105 0.00524 2.0 2 0.00264 200.0 6.7 106
Duct 100 0.6 0.3 1.9 1.2 0.5 0.00282 23.48510 1.5 1 34.7430 10.0 35.4 3474

File UNSW-UME-(M1-1_FluidFlow)(R1) Page 40 of 40


© 2003 UNSW Underground Mine Environment Module 1.1 Fluid Flow Friction & Shock Losses Photographs © 2003 UNSW Underground Mine Environment Module 1.1 Fluid Flow Friction & Shock Losses Photographs

Ventilation And Mine Services Various Airway Geometry And Roughness (1)

Fluid Flow, Friction And Shock Loss


Photographs
SP
TP

SP

TP
6 m Horadiam Shotcreted shaft 5.0 m x 4.0 m Coal Mine Development

Content
– Various Airway Geometry And Roughness
– Shock Losses
– Airway Obstructions

700 mm Hammered Hole Low Density Shaft Furnishings

© 2003 UNSW Underground Mine Environment Module 1.1 Fluid Flow Friction & Shock Losses Photographs © 2003 UNSW Underground Mine Environment Module 1.1 Fluid Flow Friction & Shock Losses Photographs

Various Airway Geometry And Roughness (2) Various Airway Geometry And Roughness (3)

Large Open Coal Mine Goaf Arched Timber Support


Timbered Airway Various Losses In A Conveyor Drift Split

Rough Profile For Blasted Development Variation In Roughness Due To Rib Failure Blasted Exhaust Raise Hand held raise

© 2003 UNSW Underground Mine Environment Module 1.1 Fluid Flow Friction & Shock Losses Photographs © 2003 UNSW Underground Mine Environment Module 1.1 Fluid Flow Friction & Shock Losses Photographs

Various Airway Geometry And Roughness (4) Flow And Gas Concentration Profiles In Goaves (CFD)

Can also be applied to spread of


smoke during mine fires
Long hole stope pile Shrink Stope

1
© 2003 UNSW Underground Mine Environment Module 1.1 Fluid Flow Friction & Shock Losses Photographs © 2003 UNSW Underground Mine Environment Module 1.1 Fluid Flow Friction & Shock Losses Photographs

Critical Velocity In Upcast Airways Shock Losses (1)


Water droplets carried up
raise by air velocity

Surface Fan Elbow

Bell mouth Inlet

Ring used to
trap water
Hoses from ring to return flowing down
Various Duct Profiles
pressure for improved removal side walls
Escape Way And Fan Elbow

© 2003 UNSW Underground Mine Environment Module 1.1 Fluid Flow Friction & Shock Losses Photographs © 2003 UNSW Underground Mine Environment Module 1.1 Fluid Flow Friction & Shock Losses Photographs

Airway Obstructions (1) Airway Obstructions (2)


Timber Support (Cribs) Steel/concrete Support (Cans)

Low Density Buntons


Heavily Timbered Shaft

Grating Removed From Inlet To Surface Fans

Coal Mine Longwall Supports LAST SLIDE RETURN TO START >>>>

2
SCHOOL OF MINING ENGINEERING
UNIVERSITY OF NEW SOUTH WALES
Australia

Underground Mine Environment Course


MODULE 1.0 VENTILATION AND MINE SERVICES

1.2 FANS AND AUXILIARY VENTILATION

A Minerals Tertiary Education Council Initiative


MINERALS
COUNCIL
Of Australia

File UNSW-UME-(M1-2_Fans)(R1) Page 1 of 42


Revision date December 2003
School Of Mining Engineering Underground Mine Environment
 2003 University Of New South Wales Module 1.2 Fans And Auxiliary Ventilation

Contents Page

1.0 INTRODUCTION……………………………………………………………………………….. 3

2.0 TYPES OF FANS AND FAN CHARACTERISTICS……………………………………….. 4

2.1 Centrifugal Fans…………………………………………………………………………… 4


2.2 Axial Fans…………………………………………………………………………………... 6
2.3 Mixed Flow Fans…………………………………………………………………………… 7
2.4 Actual Fan Characteristic Curves………………………………………………………... 8
2.4.1 Fan Power And Efficiency………………………………………………………….. 9
2.4.1.1 Compressible Flow In Fans………………………………………………… 10
2.4.1.2 Temperature Rise Through Fans…………………………………………... 10

3.0 FAN PERFORMANCE AND FAN LAWS…………………………………………...……… 12

3.1 Fan Operating Point. ……………………………………………………………………… 12


3.2 Laws Relating Fan Speed And Gas Density……………………………………………. 14
3.3 Fans In Series And Parallel Combination. ……………………………………………… 15
3.3.1 Fans In Series Combination………………………………………………………… 15
3.3.2 Fans In Parallel Combination………………………………………………………. 15
3.4 Correction For Natural Ventilation……………………………………………………….. 17
3.5 Fan Performance Control Methods……………………………………………………… 18
3.6 Comparison Of Fan Types……………………………….……………………………….. 20
3.7 Fan Noise…………………………………………………………………………………… 23

4.0 FAN PRESSURES…………………………………………………………………………….. 25

4.1 Inlet Bends…………………………………………………………………………..……… 29


4.2 Outlet Diffusers…………………………………………………………………..………… 31

5.0 AUXILIARY VENTILATION…………………………………………………………………... 33

5.1 Jet Fans…………………………………………………………………………………….. 37

6.0 REFERENCES…………………………………………………………………………………. 39

Appendix A. Mathematical Description Of Fan Curves……………………………………. 40

Appendix B. Fan Curve Analysis Spread Sheet…………………………………………….. 42

File UNSW-UME-(M1-2_Fans)(R1) Page 2 of 42


School Of Mining Engineering Underground Mine Environment
 2003 University Of New South Wales Module 1.2 Fans And Auxiliary Ventilation

1.0 INTRODUCTION
Fans have numerous applications in domestic, surface industrial and underground mining
environments. Fans are normally used to cause a circulation of air or other gas mixtures, in open or
ducted circuits to deliver a volume at a pressure determined by system characteristics. Although fans
vary significantly in physical size, power and capacity, the fundamental laws and relationships
describing fan performance are essentially universal.

Daly 1978, defines a fan as “A fan is defined as a rotary, bladed machine maintaining a continuous
flow of air. Continuous because the air flows steadily into, through and out of the fan, a feature
distinguishing it from positive displacement (piston, vane or lobe) machines, which generally produce
a pulsating flow. A fan has a rotating impeller invariably carrying blades of some kind. These blades
exert a force on the air, thereby maintaining the flow and raising the (total) pressure.”

De La Harpe 1982, also distinguishes fans from other compression devices by defining a fan as
having a compression ratio (inlet to outlet absolute pressure ratio) of less then 1.1 : 1. Under normal
conditions this would indicate a pressure rise of less then 10 kPa. However, in practice some special
purpose fans can develop significantly higher pressures.

The purpose of this module is to describe fan types and applications pertinent to the underground
mining environment together with calculation techniques employed to predict requirements, select
appropriate units, and measure performance in various configurations.

Unless otherwise stated, calculations described in this module assume incompressible fluid flow. That
is, density corrections are allowed for but are assumed to remain constant. This is generally
acceptable for practical fan selection and monitoring of system performance when fan pressure
differentials are less than 2.5 kPa. Above this point, it should be recognised that errors will occur due
to the true thermodynamic behaviour of gas mixtures in fans.

These module notes summarise key aspects of fan design, performance, formulae and calculation
methods to a standard required for this course. In this respect, they are not exhaustive but are
intended to provide sufficient detail for the training of industry practitioners and graduate education. It
is essential that candidates read and understand underpinning references for this module. Further
references are provided for candidates requiring more detailed information.

File UNSW-UME-(M1-2_Fans)(R1) Page 3 of 42


School Of Mining Engineering Underground Mine Environment
 2003 University Of New South Wales Module 1.2 Fans And Auxiliary Ventilation

2.0 TYPES OF FANS AND CHARACTERISTIC CURVES


There are three basic fan types:-

1. Centrifugal fans in which air enters the centre of an impeller, orientated at 90 degrees to the
direction of flow, then passes over the blades to be discharged through a collecting scroll or
volute to the outlet side.

2. Axial fans in which air passes through the zone swept by the rotor blades to be discharged
directly to the outlet side;

3. Mixed flow fans combine the features of axial and centrifugal fans. These fans are similar in
design to a single stage axial but with blades located in a position where a reduction in annular
area occurs in order to cause acceleration of the air stream.

2.1 Centrifugal Fans


A centrifugal fan consists of a bladed impeller with a central inlet (suction eye) through which air
enters the central blade chamber. The direction of flow then turns through 90 degrees as it passes
over the blades and discharges into the collection scroll or volute. From the collection scroll, a diffuser
section is normally used to reduce discharge velocity, refer Figure 2.1

Figure 2.1 Centrifugal Fans

Diffuser Diffuser
Electronic
Suction Back speed
Rotation
eye plate control

Collection Air
scroll or Inlet
Volute
Drive Motor
shaft
Blades Impeller
Variable
inlet vanes

Various discharge angles available

Plate blades Aerofoil blades

Principally, impeller diameter, impeller width, blade design and rotational speed determine the
volumetric and pressure capacity of a centrifugal fan.

Blades are constructed either from constant thickness plates or aerofoil sections. High efficiencies can
now be obtained with either blade type, however, aerofoil sections are required when blade stresses
increase.

File UNSW-UME-(M1-2_Fans)(R1) Page 4 of 42


School Of Mining Engineering Underground Mine Environment
 2003 University Of New South Wales Module 1.2 Fans And Auxiliary Ventilation

Most importantly, blades can be angled in a backward, forward or radial orientation with respect to the
direction of rotation. This design feature determines the nature of the fan’s characteristic and
power/efficiency curves, refer Figure 2.2 .

Figure 2.2 Air Movement In A Centrifugal Impeller And Types Of Impeller

Air Movement Radial Bladed Forward Bladed Backward Bladed


Ut Ut Ut
Ur Ua
Blade Ua Ur Ua
Ur

A C θ θ θ
Ub Ub Ub
B

Theoretical Theoretical
Ur – radial component Theoretical characteristic characteristic
Ut – tangential component P characteristic P P
Ub – blade velocity
Ua – absolute resultant velocity
P = ρ.Ui2

Q Q
Q
A centrifugal fan’s action depends on the fact that, since air has mass, it also has inertia, whereby
once set in motion it will continue to move in a straight line unless some forces applies to change the
velocity of that motion.

Consider a particle of air at point A in Figure 2.2. It is acted upon by the blade which tends to drive it in
a similar path towards B, but owing to its inertia, the resultant direction tends towards C in a straight
line. The combined or resultant effect of these two movements is that the particle of air actually flows
in a spiral until it is finally clear of the impeller.

It is the conversion of the tangential velocity component (Ut) of the resultant absolute velocity (Ua) in
the volute of the fan that provides useful static pressure. The pressure developed, P = ρ.Ut2 , depends
on blade shape and peripheral speed and therefore also on impeller diameter and rotational speed.

In radial bladed fans the radial velocity component Ur is at 90 degrees to the circumference and
therefore has no effect on the tangential velocity component when quantity increases. The theoretical
pressure developed is therefore of zero slope or constant with changing quantity.

If the blade is curved either backward or forward to the direction of rotation an additional velocity
component (Ur cot(θ)) is imparted to the air flow.

When the blades are forward curved, this component is positive and is added to the velocity ie the
tangential component is greater than the peripheral blade speed (Ub). The pressure developed
therefore increases with Ur and quantity.

When the blades are backward curved, this component is negative and is subtracted from the velocity
i.e the tangential component is less than the peripheral blade speed (Ub). The pressure developed
therefore decreases with Ur and quantity.

File UNSW-UME-(M1-2_Fans)(R1) Page 5 of 42


School Of Mining Engineering Underground Mine Environment
 2003 University Of New South Wales Module 1.2 Fans And Auxiliary Ventilation

2.2 Axial Fans


An axial fan consists of one or more bladed rotors located on the drive shaft of a motor in a circular
casing, refer Figure 2.3.

Figure 2.3 Axial Fans


Casing Blade pitch Impeller solidity
Rotor

Tail fairing

Tip
clearance Hub Straighteners
Spinner Blades
guide vanes

Contra rotating two stage fan Blade Types

Motor 2 Motor 1
Outlet Inlet
silencer silencer

Flat Cambered Aerofoil


Plate Plate Section
Axial fan with external motor
for main coal mine exhaust Bifurcated auxiliary fan
Back draught
Bend dampers

Drive shaft

Rotor(s), hub Motor


Turning and fairing
vanes

The volumetric and pressure capacity of an axial fan is determined principally by rotor diameter, blade
design, number of blades (solidity), blade pitch and rotational speed.

Flat plate, cambered plate and aerofoil blade sections are employed, although flat plates are only used
for low pressure – low efficiency applications. Cambered plates are approximately 2.0 % less efficient
than aerofoil blades and are suitable for auxiliary fans where the additional fabrication costs of aerofoil
blades are not warranted. Main surface and larger underground booster fans would use aerofoil
blades.

File UNSW-UME-(M1-2_Fans)(R1) Page 6 of 42


School Of Mining Engineering Underground Mine Environment
 2003 University Of New South Wales Module 1.2 Fans And Auxiliary Ventilation

The pressure developed by an axial fan is again dependent on the tangential component (Ut) of the
resultant absolute velocity (Ua) of air leaving the blade, refer Figure 2.4.

Figure 2.4 Air Movement On An Axial Blade

Increased airflow
Rotation
Ur Us Theoretical
characteristic
Ui P
Ut
Ua
Ub
Q

Ui – inlet velocity Ut – tangential component Us – swirl component


Ub – blade velocity Ua – absolute resultant velocity Ur – radial component

Air enters the area swept by blades of the rotor at a velocity Ui and leaves with a resultant absolute
velocity Ua. The resultant velocity is a function of blade angle, tip speed (Ub) and quantity/velocity of
air flowing over the blade surface (Ur).

As the quantity of air increases, so does the blade radial velocity Ur resulting in a reduction of the
tangential component for constant blade tip speed. This is similar in derivation to that of a backward
bladed centrifugal fan and the pressure developed decreases with increased quantity.

The function of guide vanes or straighteners placed after the rotor is to remove the swirl component
(Us) and thus increase static pressure gained. Pre rotators can also be used to impart a pre swirl to
the air stream before entering the rotor. This has the effect of reducing the swirl component leaving
the blade and hence increases static pressure.

2.3 Mixed Flow Fans


Mixed flow fans comprise an centrifugal type impeller orientated in an axial direction to airflow, refer
Figure 2.5. The fans can be designed so that more pressure conversion occurs on the guide vanes
and in the diffuser than on the impeller blades.

Figure 2.5 Mixed Flow Fans

Guide
Blades vanes

Flow regimes on the impeller and method of construction make the fan suitable for arduous
applications where erosion resistance of rotating parts is important.

File UNSW-UME-(M1-2_Fans)(R1) Page 7 of 42


School Of Mining Engineering Underground Mine Environment
 2003 University Of New South Wales Module 1.2 Fans And Auxiliary Ventilation

2.4 Actual Fan Characteristic Curves


The actual shape of fan characteristic pressure/quantity curves are many and varied, but they do have
a more or less common trend. These differ from the theoretical centrifugal or axial fan characteristic
curves due to frictional, shock and conversion losses. These losses generally increase with the square
of velocity and therefore increase with square of air quantity through the fan, refer Figure 2.6 and 2.7.

Figure 2.6 Actual And Theoretical Figure 2.7 Actual Centrifugal Fan
Fan Characteristic Curves Characteristic Curves

Stall Theoretical Frictional Backward


zone curve losses Radial

Stall Forward
point
P P
Shock
losses Radial
Actual
curve Power Forward

Backward

Q Open circuit Q
capacity

The normal operating section of the P/Q curve is approximately parabolic in shape but there is a point
in the duty cycle of a fan beyond which the performance of the fan becomes unstable. This point is at
the top end of the pressure range of the fan and if the fan is called upon to operate beyond this point a
condition known as “fan stall” or “surge” may arise due to partial recirculation of air at the blade tips.

In this upper region the fan is capable of producing two or more quantities for a given pressure. In
practice this change occurs very rapidly, and if allowed to continue may cause damage to the fan,
particularly axial fans. This is important in axial fans where recirculation of air around blade tips,
dependent on tip clearance, will result in bending stresses and the risk of catastrophic blade failure.

The effective power transmitted by fan impeller and guide vanes to the air or gas flowing through it is
given by the product of fan pressure and quantity, as follows;

Equation 2.1 Air Power

Air power AP = Pressure x Quantity = P x Q kW


1000

To confirm this relationship, consider the units involved;

Power = P . Q = Pa . m3 = N m3 = N m = J = Watts
s m2 s s s

The product of pressure and quantity for both radial and forward bladed centrifugal fans increases with
quantity, resulting in increasing power consumption. The product of pressure and quantity in backward
bladed centrifugal and axial fans reduces with increased quantity and therefore these fans have non
overloading power characteristics. In terms of efficiency, axial and backward bladed centrifugal fans
are similar. Both forward and radial bladed centrifugal fans have lower efficiencies.

File UNSW-UME-(M1-2_Fans)(R1) Page 8 of 42


School Of Mining Engineering Underground Mine Environment
 2003 University Of New South Wales Module 1.2 Fans And Auxiliary Ventilation

2.4.1 Fan Power And Efficiency


Fan efficiency describes the ratio of power transferred to the air, or gas mixture, passing through it to
the power absorbed by the fan at a given duty. Due to the magnitude of values involved these are both
normally expressed in kilowatts with efficiency in percent, as follows;

Equation 2.2 Fan Efficiency

Fan efficiency η = Air power = P. Q x 100 %


Motor power kW 1000. MP

Power losses occur within the fan, drive shaft, gearbox (if fitted), motor, speed controller (if used),
transmission cables and transformer. It is therefore important that the method used by manufacturers
to define fan efficiency is confirmed so that measurement of absorbed power is appropriate. For
example, power curves may exclude gearbox losses and therefore measuring electrical power
consumption at the motor would be inaccurate. It is also important to note if the efficiencies quoted are
in terms of total or static pressures.

The formula required to calculate motor power from measured voltage and current, are as follows;

Equation 2.3 Calculation Of Motor Power

AC motor power MP = 3 x V x I x cos(Φ) kW


1000

DC motor power MP = V x I kW
1000
The power factor cos(Φ) may vary but a value of 0.8 is normally appropriate.

Figure 2.8 Fan Efficiency Contours


Pressure
Manufacturers fan curve(s) may be supplied with power and Fan
efficiency contours plotted together with pressure-quantity efficiency
characteristics, refer Figure 2.8. contours
80 75
Ideally, the fan will be selected so that the operating point lies
in an area of high efficiency. Alternatively, power consumption P 70
can be analysed to calculate the cost of operating at other
points, different fans can be compared or multiple fans used 65
when the duty varies considerably within the life of the fan.
Power

Q
Worked Example - Fan Power And Efficiency

A fan is operating at a duty of 120 m3/s at 1400 Pa. What is the fan efficiency if the A.C. motor
is absorbing 140 amps at 1000 V (power factor =0.8)?

Motor power MP = 3 x V x I x cos(Φ) MP = 3 x 1000 x 140 x 0.8 = 194 kW


1000
Air power AP = P x Q AP = 1400 x 120 = 168 kW
1000 1000

Fan efficiency η = AP x 100 η = 168 x 100 = 86.6 %


MP 194

File UNSW-UME-(M1-2_Fans)(R1) Page 9 of 42


School Of Mining Engineering Underground Mine Environment
 2003 University Of New South Wales Module 1.2 Fans And Auxiliary Ventilation

2.4.1.1 Compressible Flow In Fans


For simplicity, the calculations described in this module assume air to behave as an incompressible
fluid (like water) when passing through a fan and that the process is adiabatic (no heat exchange).
These approximations are acceptable in most applications where the pressure differential across the
fan is less than about 2,500 Pa.

For higher pressure rises, compression and heating of air through the fan should be taken into
account. That is the volumetric flow rate on the discharge (higher pressure) side of the fan will be
lower than on the inlet side but the mass flow rate remains the same. The situation is further
complicated by heat transfer and the psychrometric conditions of the air/water and vapour/water
droplet mixtures involved.

A thorough examination of the true thermodynamic behaviour of moist air in fans is beyond the scope
of this module, further details are provided in the references, McPherson 1971.

To account for losses associated with compression (reduced volume) at the fan outlet, a
compressibility factor K is used. The total efficiency of a fan is then described by;

Equation 2.4 Fan Efficiency With Compression

Fan efficiency η = K P. Q x 100 %


1000. MP

The value of K reduces with increased absolute pressure ratio ( Patmos + Pfan)/Patmos and reduced fan
efficiency. For absolute pressure ratios of 1.05 to 1.3 ( 5 to 30 % atmosphere) and for a fan with circa
80 % efficiency, the value of K ranges 0.99 to 0.92.

2.4.1.2 Temperature Rise Through Fans


The work done by a fan to increase the pressure/energy of air flowing through it overcomes frictional
losses, and therefore, apart from some velocity regain, a fan does no useful thermodynamic work.
Consequently, all electrical power input to the fan is converted to heat energy. Depending on fan
design and location of the motors, heat from motors and drive shaft bearings may or may not be
added to the air passing through the fan.

At constant volume, the thermal capacity of air is approximately 0.712 kJ/kg OC and approximately
1.015 kJ/kg at constant pressure. That is, 0.712 kJ are required to heat 1.0 kg by 1.0 OC. These
values vary with moisture content, pressure and temperature.

The dry bulb temperature rise through a fan is then given by;

Equation 2.5 Temperature Rise Through Fans

O
Temperature rise ∆t = P. Q . 1 C
1000.η (ρ.Q.Cp)

As an approximation and for low (2.5 kPa) pressure differentials, the wet bulb temperature of 10 m3/s
air increases by 1 OC for 40 kW heat input.

File UNSW-UME-(M1-2_Fans)(R1) Page 10 of 42


School Of Mining Engineering Underground Mine Environment
 2003 University Of New South Wales Module 1.2 Fans And Auxiliary Ventilation

Worked Example – Temperature Rise Through Fan

It is emphasised that these calculations are very much an approximation but serve to demonstrate the
orders of magnitude involved.

Estimate the temperature rise through a fan operating at a duty of 120 m3/s at 1400 Pa if the
motor is not in the air stream ?

Air power AP = P x Q AP = 1400 x 120 = 168 kW


1000 1000

Allow for 5 % losses in motor and bearings and a fan efficiency of 79%

Power available for heating = 168 = 227 kW


(0.79 – 0.05)

O
Dry bulb temperature increase ∆t = Power = 227 = 1.55 C
(ρ.Q.Cp) 1.2 x 120 x 1.015

O
Wet bulb increase ∆t = Power . 10 = 227 x 10 = 0.47 C
40 Q 40 X 120

Estimate the temperature rise through an auxiliary fan operating at a duty of 12.0 m3/s at 5,400
Pa if the motor is in the air stream(assume 79 % efficient)?

Air power AP = P x Q AP = 5,400 x 12.0 = 65 kW


1000 1000

Power available for heating = 65 = 82 kW


0.79
O
Dry bulb temperature increase ∆t = Power = 82 = 5.6 C
(ρ.Q.Cp) 1.2 x 12.0 x 1.015

O
Wet bulb increase ∆t = Power . 10 = 82 x 10 = 1.71 C
40 Q 40 X 12.0

This demonstrates that the temperature rise through fans increases with pressure and that the
temperature rise through intake booster fans or force auxiliary fans can have significant impact on
working temperatures.

File UNSW-UME-(M1-2_Fans)(R1) Page 11 of 42


School Of Mining Engineering Underground Mine Environment
 2003 University Of New South Wales Module 1.2 Fans And Auxiliary Ventilation

3.0 FAN PERFORMANCE AND FAN LAWS


This section deals with the fundamental laws and relationships required to predict and or monitor fan
performance in open or ducted circuits.

3.1 Fan Operating Point


When a fan is connected to a ducted or mine ventilation circuit, the pressure developed by the fan
overcomes frictional and shock losses within the circuit. For practical purposes, it is assumed that
these losses are calculated from system resistance and the square of quantity, as follows, refer Figure
3.1;

Equation 3.1 Pressure Loss

Circuit pressure loss Pc = R.Q2 Pa

Figure 3.1 Fan Operating Points


Pressure X = P Q points
R X
A Higher
X resistance
B 3
Pc Lower
Q P X resistance
1
Circuit Fan 4
X
System
resistance 2 X
P = R.Q2

X
Q
The operating point (e.g. point 1 fan A) is given by the intersection of the fan characteristic and circuit
resistance curves. That is, the point where the pressure developed by the fan balances pressure
losses within the circuit. The fan cannot develop the pressure to draw more air through the circuit or, if
less air was in the circuit, the fan pressure would exceed frictional losses and the quantity would
increase.

If a different fan is installed or the duty changed (e.g. speed reduced) then the duty point moves to the
new intersection of fan characteristic and circuit resistance (e.g. point 2 fan B).

The effect of changing circuit resistance will therefore also affect the duty point. For example, if the
circuit resistance increases then the duty points move to 3 and 4 for fans A and B respectively.

These calculations are most easily manipulated using spreadsheets. The fan curve is usually
represented by between four to eight pressure – quantity points selected to describe the curve
adequately. The equation that usually provides the best fit to the operating section of the fan curve is a
least square parabola of the form P = AQ2 + B.Q + C, refer Appendix A. This information is also
required as input parameters to network simulation programs.

File UNSW-UME-(M1-2_Fans)(R1) Page 12 of 42


School Of Mining Engineering Underground Mine Environment
 2003 University Of New South Wales Module 1.2 Fans And Auxiliary Ventilation

Worked Example – Fan Operating Points

A fan’s characteristic curve is described by the following P-Q data points.


Single fan
2500
Single Unit
Quantity Pressure 2000
m3/s Pa
100 2200

Pressure Pa
1500

145 2000
190 1500 1000

215 1000
500
235 500
250 0 0
0 50 100 150 200 250 300
Quantity m 3/s

If the system resistance is 0.034 Ns2/m8 what is the operating point ?

Calculate system pressure loss for quantities from 0 to 250 m3/s and plot on same graph
P=R.Q^2 2500
Single fan Resistance Operating point

Quantity Pressure
m3/s Pa 2000

0 0.0 Pressure Pa
50 85.0 1500

100 340.0
1000
150 765.0
200 1360.0 500
250 2125.0
0
Operating point is 200 m3/s at 1360 Pa 0 50 100 150 200 250 300
Quantity m 3/s

What is the operating point if the system resistance is reduced to 0.020 Ns2/m8 ?

Single fan Resistance


Operating point Low er resistance
R= 0.034 R= 0.020 2500
P=R.Q^2 P=R.Q^2
Quantity Pressure Pressure 2000

m3/s Pa Pa
Pressure Pa

0 0.0 0.0 1500

50 85.0 50.0
1000
100 340.0 200.0
150 765.0 450.0 500
200 1360.0 800.0
250 2125.0 1250.0 0
0 50 100 150 200 250 300
Quantity m 3/s

Operating point is 220 m3/s at 968 Pa

In these examples, the operating point is obtained by estimating the quantity at the point of
intersection and calculating the pressure from known resistance.

File UNSW-UME-(M1-2_Fans)(R1) Page 13 of 42


School Of Mining Engineering Underground Mine Environment
 2003 University Of New South Wales Module 1.2 Fans And Auxiliary Ventilation

3.2 Laws Relating Fan Speed And Density


Fan characteristic curves are normally acquired at the time of purchase or from tenders at specified
speed(s) and air density. Fan designers employ many fan laws describing the relationship between
fan performance, fan geometry, impeller dimensions, rotational speed, blade tip velocity or air density.
In practice, density and speed are normally the only variables that may change during the use of a
particular fan unit. It is sometimes appropriate to change impeller solidity of axial fans or the impeller of
a centrifugal fan for a wider or narrower design. These modifications should be made in consultation
with the fan manufacturer.

The relationship between fan pressure and quantity with speed and density are as follows;

Equation 3.2 Fans Laws – Speed And Density

Speed change 2 3
Q1 = n1 P1 = n1 W1 = n1.
Q2 n2 P2 n2 W2 n2

Air quantity directly Pressure proportional Power proportional


proportional to speed to square of speed to cube of speed.

Density change
P1 = ρ1 . W1 = ρ1.
Q1 = Q2 P2 ρ2 W2 ρ2

No change in quantity Pressure directly Power directly


proportional to proportional to
density density

The quantity passing through a fan is directly proportional to speed and the pressure developed is
proportional to the square of the speed. Consequently the absorbed power is proportional to the cube
of speed. Density does not effect fan quantity but pressure developed and power consumed is directly
related to density.

Worked Example - Fan Laws

A fan is operating at 360 rpm with a duty of 120 m3/s at 1400 Pa, the efficiency is 81 %. What is
the new duty and power consumed if the speed is changed to 515 rpm ?

Power at 360 rpm W1 = (Q1 x P1/1000) W1 = (120 x 1400/1000) = 207.4 kW


(ε/100) (81/100)

New quantity Q2 = Q1 x n2 / n1 Q2 = 120 x 515 / 360 = 171.7 m3/s

New pressure P2 = P1 x (n2 / n1)2 P2 = 1400 x (515 / 360)2 = 2865 Pa

New power W2 = W1 x (n2 / n1)3 W2 = 207.4 x (515 / 360)3 = 607 kW

What is the duty if the density then changes from 1.20 kg/m3 to 1.15 kg/m3 ?

New quantity Q2 = Q1 Q2 = 171.7 m3/s

New pressure P2 = P1 x ρ2 / ρ1 P2 = 2865 x 1.15 / 1.20 = 2745 Pa

New power W 2 = W1 x ρ 2 / ρ 1 W2 = W1 x 1.15 / 1.20 = 582 kW

File UNSW-UME-(M1-2_Fans)(R1) Page 14 of 42


School Of Mining Engineering Underground Mine Environment
 2003 University Of New South Wales Module 1.2 Fans And Auxiliary Ventilation

3.3 Fans In Parallel And Series Combinations


Fans are connected in series and parallel for a number of applications. For main surface or
underground booster fans, meeting the required duty with two or more fans in parallel provides
security in the event of a single unit failure or for maintenance. In addition, increasing the size of a
single fan to accommodate high air volumes may incur disproportionate casing fabrication and
foundation costs.

Although theoretically possible, centrifugal fans are not usually connected in series to boost pressure.
Series configuration is usually employed when axial fans are used in a high pressure auxiliary
ventilation systems.

3.3.1 Fans In Series Combination


Combined
Consider two fans A and B connected in series. Fan Curve

A B
P P = PA + PB
Q Q = QA = QB
Fan
PA PB Pressure

P Fan A
The same quantity passes through both fans but the
pressure developed is given by the sum of that due to
each fan, P = PA + PB. Fan B
PA
The method of calculation is to generate a combined PB
fan curve by summing the fan pressure at each quantity Resistance
point.

The operating point is then given by the intersection of Quantity Q


the system resistance curve and the combined fan
curve.

Series fans do not necessarily have to have the same characteristic curve, but should be similar to
avoid one fan stalling whilst the other operates.

If the fans are different, individual fan operating points are then determined by the pressure developed
for quantity Q.

3.3.2 Fans In Parallel Combination


Consider two fans A and B connected in parallel,
P = PA = PB
A QA
Q = QA + QB Resistance
Fan Fan A
B QB Pressure

P Fan B Combined
P Fan Curve
In parallel combination, the pressure drop across both fans
is the same with the quantity through each unit determined
by it’s characteristic curve. Again, the fans do not
necessarily have to be the same but should be of similar
duty.
QB QA Q

If the fans are different, the quantity through each fan is determined by intersection of pressure P with
each characteristic curve.

File UNSW-UME-(M1-2_Fans)(R1) Page 15 of 42


School Of Mining Engineering Underground Mine Environment
 2003 University Of New South Wales Module 1.2 Fans And Auxiliary Ventilation

When required to calculate the fan characteristic curve for multiple fans operating at a different speed
and or density than the manufacturers curve, it is usually more convenient to correct the single fan
characteristic curve first then combine them in series or parallel.

Worked Example - Fans In Series And Parallel

A fan’s characteristic curve is described by the following P-Q data points.


Single fan
Single Unit 2500

Quantity Pressure
2000
m3/s Pa
100 2200

Pressure Pa
1500
145 2000
190 1500 1000

215 1000
235 500 500

250 0
0
0 50 100 150 200 250 300
Quantity m 3/s

What is the combined characteristic curve for two units in series ?

Single Unit Two in series Single fan Tw o in series

Quantity Pressure Quantity Pressure 5000

m3/s Pa m3/s Pa 4500


4000
100 2200 100 4400
3500
145 2000 145 4000
Pressure Pa

3000
190 1500 190 3000 2500
215 1000 215 2000 2000
235 500 235 1000 1500

250 0 250 0 1000


500
Q2 = Q1 P2 = P1 x 2 0
0 50 100 150 200 250 300
Quantity m 3/s

What is the combined characteristic curve for two units in parallel ?

Single Unit Two in parallel Single fan Tw o in series Tw o in parallel


5000
Quantity Pressure Quantity Pressure 4500
m3/s Pa m3/s Pa 4000 Series
100 2200 200 2200 3500
145 2000 290 2000
Pressure Pa

3000
190 1500 380 1500 2500

215 1000 430 1000 2000


Parallel
235 500 470 500 1500

1000
250 0 500 0
500

Q2 = Q1 x 2 P2 = P1 0
0 100 200 300 400 500 600
Quantity m 3/s

File UNSW-UME-(M1-2_Fans)(R1) Page 16 of 42


School Of Mining Engineering Underground Mine Environment
 2003 University Of New South Wales Module 1.2 Fans And Auxiliary Ventilation

3.4 Correction For Natural Ventilation Pressure


The natural ventilation pressure in a mine ventilation circuit is determined by the depth of workings
and difference between average intake (downcast) and average exhaust (upcast) density. Normally,
heating of air within the mine results in exhaust air being hotter, and therefore less dense, than that in
the intakes. Consequently, natural ventilation pressure normally acts in the direction of airflow and
acts to assist surface fans. It would also be expected that natural ventilation pressure would increase
with lower ventilation rates due to higher exhaust temperatures for similar heat loads. In reality this
relationship will be highly complex but a linear relationship is shown for simplicity, refer Figure 3.2.

Figure 3.2 Effect Of Natural Ventilation Pressure On Fan Operating Point.

Without natural ventilation


P P = R.Q2

NVP With natural ventilation


P = R.Q2 - NVP

Natural ventilation
pressure

NVP Q

The fan characteristic curve is not changed by natural ventilation pressure, it must only be corrected
for exhaust air density. The fan duty is that determined by the system resistance , net of natural
ventilation pressure. The method of calculation is to calculate the system pressure loss (P=R.Q2 ) with
quantity and then subtract natural ventilation pressure. In this respect, natural ventilation is described
as a constant pressure circuit fan.

Note that the effect of natural ventilation pressure or change in resistance on fan operating points
depends on the gradient if the fan curve at the point of intersection.

Worked Example – Fan Operating Point With Natural Ventilation Pressure

Refer to example Worked Example – Fan Operating Points for initial data R = 0.034 Ns2/m8

If natural ventilation pressure is 350 Pa in the direction of flow, what is the new operating point
Single fan Resistance
Operating point Resistance w ith NVP
P=R.Q^2 P=R.Q^2-NVP 2500

Quantity Pressure Pressure


2000
m3/s Pa Pa
0 0.0 -350.0 1500
Pressure Pa

50 85.0 -265.0
1000
100 340.0 -10.0
150 765.0 415.0 500
200 1360.0 1010.0
250 2125.0 1775.0 0
0 50 100 150 200 250 300

3 -500
Operating point is 210 m /s at 1150 Pa Quantity m 3/s

File UNSW-UME-(M1-2_Fans)(R1) Page 17 of 42


School Of Mining Engineering Underground Mine Environment
 2003 University Of New South Wales Module 1.2 Fans And Auxiliary Ventilation

3.5 Fan Performance Control Methods


Most fan applications require the fan duty to be controlled in order to alter the operating point in a
given system. For example, main mine fan(s) may be purchased for a 15 year mine life during which
time the quantity of air required and system resistance could change significantly.

The method selected to control fan performance depends on the application and range of duty
required, and are principally as follows, refer Table 3.1.

Table 3.1 Methods Of Controlling Individual Fan Performance

System dampers Inlet vanes Speed control Blade pitch variation


1. Simple regulation of the 1. Throttle fan inlet but also 1. Fan efficiency remains 1. Only applies to axial
system introduce a pre swirl. essentially constant at fans.
2. Does not change fan 2. More efficient than damping. various speed settings. 2. Adjustment at standstill
characteristic curve 3. Reduced fan efficiency with 2. Higher capital cost. or in motion.
3. Most uneconomical partial vane closure 3. Maintains relatively high
method but simple for efficiency.
ducts.
4. Not suitable for main or
axial fans

Pressure Pressure Pressure Pressure

P1 P1 P1 P1

P2 Power 1
Power 1 1 Power
Power 2 2
P P 3 P 2 P 3
3
2 1
3 2 1 3 2 1 3
Q1
Q Q1 Q2 Q Q1 Q Q1 Q

System dampers – by which the system is regulated to increase resistance and hence reduce
quantity.

Inlet vanes – by which the inlet to the fan is regulated by vanes that introduce a pre swirl for improved
efficiency compared to dampers.

Speed control – by which the rotational speed of the impeller is changed either by controlling the
motor speed directly or installing a gearbox.

Axial blade pitch adjustment – by which the fan duty is controlled by increasing or decreasing blade
pitch between set limits.

For some applications, a combination of two control methods may be employed. For example inlet
guide vanes for fine adjustment together with a two speed motor for coarse adjustment.

In each case, the pressure – quantity duty is determined by the point of intersection between the
system resistance and fan characteristic curve. This in turn allows the power consumption to be
determined from which the additional cost, or cost saving of a proposed duty change can be
calculated.

File UNSW-UME-(M1-2_Fans)(R1) Page 18 of 42


School Of Mining Engineering Underground Mine Environment
 2003 University Of New South Wales Module 1.2 Fans And Auxiliary Ventilation

Worked Example – Fan Duty Control

A fan is operating with a duty of 200 m3/s at 2100 Pa, a speed of 500 rpm and efficiency of 80% .
The quantity is to be changed to 175 m3/s.

What is the existing power consumption ?

Existing air power = P.Q = 2100 x 200/1000 = 420 kW

Existing electrical consumption = 420/ 0.8 = 525 kW

Calculate the fan duty if the change is made by altering speed ?

Quantity ratio = Q1 = n1 therefore n2 = n1 Q2 = 500 x 175 = 438 rpm


Q2 n2 Q1 200
2 2 2
Pressure ratio P1 = n1 therefore P2 = P1 n2 = 2100 x 438 = 1611 Pa
P2 n2 n1 500

Electrical consumption = 1611 x 175 = 352 kW


1000 x 0.8

The fan speed must be reduced to 438 rpm for a duty of 175 m3/s at 1611 Pa. The power consumption
would reduce by 173 kW to 352 kW.

What is the power saving ($/year) if the cost of power is $ 0.05 per kWh ?

Cost of 1.0 kW per year = 0.05 x 24 x 365 = $438 /year

Annual cost of power at existing duty = 525 x 438 = $229,950

Power saving = (525 – 352) x 438 = $ 75,774 / year.

Other methods of duty control require the fan curve to be defined in order to determine the fan
operating point at 175 m3/s. The cost benefit of each method can then be compared.

File UNSW-UME-(M1-2_Fans)(R1) Page 19 of 42


School Of Mining Engineering Underground Mine Environment
 2003 University Of New South Wales Module 1.2 Fans And Auxiliary Ventilation

3.6 Comparison Of Fan Types And Specific Speed


Based on theoretical characteristic curves and fan design laws, a comparison of fan types with respect
to a backward bladed centrifugal fan, is shown in Table 3.2.

Table 3.2 Comparison Of Fan Type Performance At The Same Duty

Axial Backward Radial Forward


Size 0.82 1.00 1.02 0.80
Speed 1.40 1.00 0.70 0.57
Power 1.00 1.00 1.20 1.14

In mining applications, the choice of fan invariably lies between axial or backward bladed centrifugal
designs. Radial bladed centrifugal fans are less efficient and slightly larger than backward bladed fans
but may be suitable for conditions with high erosion rates, for example dust extractors. Forward bladed
centrifugal fans are smaller and operate at lower speeds but do not have a non overloading power
characteristic. Forward bladed fans will incur lower capital cost and would be suitable for constant duty
industrial applications where noise may also be an issue.

The principal benefit of axial fans are the compact nature of their design, involving smaller diameters
and axial flow path through the impeller. Both axial and backward bladed centrifugal fans can operate
with similar efficiencies but, for the same duty, an axial fan will have to rotate approximately 40 %
faster with a consequential increase in noise generated.

For a given fan diameter, the main variable available is rotational speed determined by the maximum
speed of the drive motor and any electrical or mechanical speed variation (normally reduction)
employed.

The synchronous speed of induction motors is determined by the number of poles and supply
frequency (Synchronous speed = 2 x frequency / number of poles). Under load, the actual motor
speed will be less than synchronous speed by ≈1.0% for large motors and up to ≈10% for small
motors, this is known as slip. The synchronous and range of full load speeds for induction motors at
50 Hz supply are as shown in Table 3.3.

Table 3.3 Synchronous And Full Load Speeds Of 50 Hz Induction Motors


Number Of Sychronous Speed Full Load Speed
poles rps rpm Small <1 kW Large > 500 kW
2 50 3000 2700 2985
4 25 1500 1380 1492
6 17 1000 900 995
8 13 750 680 745
10 10 600 550 596
12 8 500 460 496
16 6 375 350 360

Due to practicable strength of blades and impellers, the limiting rotational speed of axial or centrifugal
fans is determined by the tip velocity of the blades. This limit is typically between 100 and 125 m/s for
larger diameter fans and approximately 140 m/s for smaller auxiliary fans. Tip speeds are calculated,
as follows;.

Equation 3.3 Blade Tip Velocity

Tip velocity = π. D . n m/s


60

Where, n is the rotational speed, rpm, and D is the diameter of the impeller.

File UNSW-UME-(M1-2_Fans)(R1) Page 20 of 42


School Of Mining Engineering Underground Mine Environment
 2003 University Of New South Wales Module 1.2 Fans And Auxiliary Ventilation

The fan speed used is then determined by the available motor pole speeds that are less than or equal
to the limiting blade tip velocity, refer Figure 3.3.
Figure 3.3 Limiting Fan Speeds

For example, two pole speeds of 2800 to 2950 rpm 5000


would be suitable for auxiliary fan diameters less
than 1.0 m.
4000 Limited by blade
Larger main mine fans (2.0 to 4.0 m diameter) will tip velocity

Fan Speed rpm


operate with at 6 to 12 pole speeds, typically 480 to
960 rpm. 3000
2P Limited by
synchronous
The speed range for which a fan’s performance is at 2000 speeds
or near peak efficiency lies within a relatively narrow
range. In addition, the range of optimum speeds 4P
varies with fan type and design parameters. A broad 1000
6P
method of selecting appropriate fans for a given duty 8P
10 P
is to use specific speed. 0
0.0 1.0 2.0 3.0 4.0 5.0
Impeller Diameter m

Specific speed is defined as the speed at which a fan would have to operate in order to supply 1.0
m3/s at a pressure of 1 Pa. The relationship between specific speed, actual speed, quantity, pressure
and density is as follows;

Equation 3.4 Specific Speed And Blade Tip Velocity

Specific speed ns = n . Q0.5 rps


0.75
60 (P/ρ)

Note that specific speed is reported in revolutions per second (rps), but fan speed is commonly
reported in revolutions per minute. A plot of fan efficiency against specific speed is shown in Figure
3.4.
Figure 3.4 Specific Speeds For Various Fan Types

95
Centrifugal Mixed Axial
90
Backw ard
85 curved
Efficiency %

80

75
Radial Forw ard
70 Blades curved

65
0.01 0.1 1 10
Specific Speed rps

Centrifugal fans are suitable for specific speeds from approximately 0.03 to 0.30 rps and axial fans
from 0.3 to 2.0 rps. Radial and forward bladed centrifugal fans are less efficient and therefore selected
when other design features, for example erosion resistance or capital cost make them attractive.

Specific speed is inversely proportional to pressure and therefore higher pressure applications result
in lower specific speed suggesting the use of centrifugal instead of axial fans.

File UNSW-UME-(M1-2_Fans)(R1) Page 21 of 42


School Of Mining Engineering Underground Mine Environment
 2003 University Of New South Wales Module 1.2 Fans And Auxiliary Ventilation

A summary of considerations to be made when selecting fan types are shown in Table 3.5.

Table 3.5 Considerations For Fan Selection

Consideration Axial Fans Centrifugal Fans


Size and mass Smaller and lighter than centrifugal fans Larger casing and impeller dimensions.
making them easier to start (less inertia). Larger overall profile due to orientation to flow.
Lower capital and installation costs Starting issues can b overcome with variable
Compact for auxiliary fan applications. speed drives.

Pressure Due to recirculation at blade tips, multi stage Very high pressures can be obtained with narrow
fans required above circa 5.0 kPa impellers.

Stall point More prone to severe stall at higher pitch More robust design reduces risk of catastrophic
angles. failure when subject to stall vibrations.
Blade tips not supported therefore prone to
bending stresses and catastrophic failure.

Efficiency High efficiency maintained over a greater Higher peak efficiency but over a smaller range.
duty range.

Performance control Blade pitch control improves flexibility and Inlet guide vanes reduce efficiency.
maintains efficiency. If speed control is available then there is no
significant difference between fan types.

Reversibility Will operate in reverse with 40 to 60 % flow Individual units do not have reversible
but at significantly lower efficiency. capabilities i.e. backward blades become
forward blades.
Complex duct and door arrangements required.

Speed Higher rotational speed allows direct drive


using cheaper, more readily available
motors (less poles).

Reliability Higher rotational speeds, small tip More robust impeller design and lower rotational
clearances and motors in the air stream speeds make centrifugal fans more reliable.
make axial fans less reliable.

Manufacture More complex rotor and hub assembly with Less complex and easier to fabricate.
fine engineering tolerances.

Maintenance Motor and bearings usually in the air stream Motor and bearings not in the air stream,
making maintenance more problematic. therefore maintenance more straightforward.
Alternative is to use long lay shafts to take
the motor out of the air stream.

Corrosion/ Erosion Axial blades (particularly aluminium alloys) Radial steel plate blades provide greater wear
are particularly susceptible to wear. resistance.

Noise Higher noise levels due to higher rotational Generally quieter than axial fans at the same
speeds for the same duty as a centrifugal duty.
fan.

File UNSW-UME-(M1-2_Fans)(R1) Page 22 of 42


School Of Mining Engineering Underground Mine Environment
 2003 University Of New South Wales Module 1.2 Fans And Auxiliary Ventilation

3.7 Noise
Due to the relatively high speeds of impeller rotation and often high power consumption, fans can be a
significant source of noise both underground and on surface. The four main sources of noise in fans
(Howes 1982) are as follows,;

Vortex noise caused by turbulence in shear zones around the trailing edges of blades or guide vanes.
The noise produced is spread over a wide range of frequencies with the highest in the 1000 Hz
frequency band. This is the most significant noise source.

Blade noise caused by periodic variations in pressure resulting from rotation.

Mechanical vibrations may be due to aerodynamics of the system together with mechanical vibration
resulting from imbalance or misalignment of drive components.

Electrical components such as motors, transformers and variable speed drive controllers can
produce noise that, may not have a very high sound power level, but lies in a relatively narrow
frequency range or is “tonal” . This can be problematic in surface applications in close proximity to
residential homes or offices.

Noise is emitted from fans directly through the inlet and outlet air path of the fan and also by
transmission through the casing (breakout). It is important to note that because the velocity of sound in
air is approximately 344 m/s, the sound emitted from axial fan inlets will be nearly as significant as that
at the discharge.

The sound power level (SWL) of a fan depends on the type of fan, pressure, quantity and power
consumed at the operating duty and is calculated as follows;
Figure 3.5 Sound Power Levels For
Centrifugal Fans
140
Equation 3.5 Fan Sound Power Levels
135 4.0 kPa
Fan SWL = SWLo + SWLs 130
2.0 kPa
Sound Power Level dB

125
120
SWLo ≈ 36 dB for centrifugal fans 115 0.5 kPa
SWLo ≈ 40 dB for axial fans 110
105

SWLs = 10.Log Q + 20.Log Pt dB for all fans 100


95
90
SWL = overall sound power level in octave 1 10 100 1000
bands 31.5 to 8,000 Hz Quantity m3/s

Note the Pt is the fan total pressure and SWL is the sound power and NOT the sound pressure levels.

From the overall sound power level, the sound power level for individual frequency bands can be
estimated by applying correction factors. The corrections to be applied to the sound power level of a
centrifugal and axial fans are shown in Table 3.6.

Table 3.6 Sound Power Level Corrections To Obtain Octave Band Spectra

Mid band freq. Hz 63 125 250 500 1,000 2,000 4,000 8,000
Backward blades -4 -6 -8 -10 -15 -20 -26 -32
Forward blades -3 -6 -10 -14 -18 -22 -27 -32
Axial fan -10 -7 -5 -7 -8 -12 -18 -24

File UNSW-UME-(M1-2_Fans)(R1) Page 23 of 42


School Of Mining Engineering Underground Mine Environment
 2003 University Of New South Wales Module 1.2 Fans And Auxiliary Ventilation

Worked Example – Fan Noise

Calculate the sound power level spectra for an axial and centrifugal fan operating at 200 m3/s
and total pressure of 2.5 kPa ?.

Axial overall SWL = 40+10.Log Q +20.Log P = 40 + 10 Log(200) + 20 Log(2500) = 131 dB

Centrifugal overall SWL = 36+10.Log Q +20.Log P = 36 + 10 Log(200) + 20 Log(2500) = 127 dB

Applying correction for mid band frequencies.

AXIAL FAN Axial Fan Centrifugal Fan


Mid band freq. Hz 63 125 250 500 1,000 2,000 4,000 8,000 130

Overall SWL 131 131 131 131 131 131 131 131 125

Axial fan -10 -7 -5 -7 -8 -12 -18 -24 120

Sound Power Level dB


Corrected 121 124 126 124 123 119 113 107 115

110

BACKWARD BLADED FAN 105

Mid band freq. Hz 63 125 250 500 1,000 2,000 4,000 8,000 100

Overall SWL 131 131 131 131 131 131 131 131 95

Backward blades -4 -6 -8 -10 -15 -20 -26 -32 90


63 125 250 500 1,000 2,000 4,000 8,000
Corrected 127 125 123 121 116 111 105 99 Mid Band Frequency Hz

Generally, a backward bladed centrifugal fan will be quieter than an axial fan for the same pressure
developed.

When required, sound attenuation can be achieved by installing inlet and or outlet silencers, lagging
the casing and or building a sound attenuating enclosure around the fan. Cylindrical and splitter type
silencers can be used, refer Figure 3.6. The effectiveness of the silencer is dependent on length to
width or diameter ratio, splitter spacing, and splitter width. Silencers also result in a pressure loss
which should be quantified and included in system resistance.

Figure 3.6 Cylindrical And Splitter Silencers For Fan Systems

Splitter silencer
Hollow cylindrical silencer

Pod type cylindrical


silencer

File UNSW-UME-(M1-2_Fans)(R1) Page 24 of 42


School Of Mining Engineering Underground Mine Environment
 2003 University Of New South Wales Module 1.2 Fans And Auxiliary Ventilation

4.0 FAN PRESSURES


The description of fan pressure – quantity characteristics in section 2.0, is generally concerned with
fan static pressure and static (frictional) pressure losses in circuits that determine the fan operating
point. If the velocity of air or gas in fan ducts and outlets was very low, then this description would be
adequate. However, for practicable fan, duct and airway diameters, velocities are normally very high
(20 to 40 m/s) in fan ducts and casings and the velocity component must be taken into account..

For example, if a fan discharges air at a velocity of 30 m/s and a density of 1.2 kg/m3 the velocity
pressure is 0.5 ρ v2 = 540 Pa. The fan has developed this velocity pressure in addition to the static
pressure gain and therefore the effective static pressure applied to the system is reduced.

According to fan testing standards, the following definitions of pressure are made, (Burrows 1974);

Fan TOTAL pressure is the algebraic difference between the mean total pressure at the fan outlet
and the mean total pressure at the fan inlet.

Fan VELOCITY pressure is the velocity pressure corresponding to the average velocity at the fan
outlet based on the total outlet area without any deduction for motors, fairings or other bodies.

Fan STATIC pressure is the difference between the fan total pressure and fan velocity pressure. This
represents the work done in compression and is given by the difference in fan total pressure and fan
velocity pressure.

Side tube or static pressure tube is a tube which allows the air to flow without disturbance past one
or more small orifices having their axes at right angles to the direction of flow of the air stream in which
it is placed.

Facing tube or total pressure tube is an open ended tube, the axis of which is coincident with the
direction of the air stream in which it is placed, the open end facing up stream i.e against the direction
of flow.

Fan curves can be provided to represent the static or total pressure rise through the fan. The
difference between the curves is the velocity pressure at fan outlet for the quantity through the fan.

Figure 4.1 Fan Static And Total


Pressure Characteristic Curves

Equation 4.1 Fan Total, Static And Velocity Pressure Total pressure
Fan
Total pressure TP = Static pressure SP + Velocity pressure VP characteristic
curve Duty
2
Fan Velocity pressure VP = 1 . ρ . V OUTLET Pa point
2 P
System
2
P = R.Q

Static pressure

Q
When measuring fan pressures in ducts, underground mine circuits or surface installations it is
important to note the following relationships;

• Static pressure reduces due to frictional or shock losses in the circuit and increases through a fan.

• The velocity pressure changes if there is a change in quantity or cross sectional area otherwise it
is constant. That is, assuming incompressible flow, a change in static pressure does not alter
velocity pressure.

• If the velocity pressure changes, there is a corresponding change in the static pressure. This is
added or subtracted from any frictional or fan pressure component that may also occur.

File UNSW-UME-(M1-2_Fans)(R1) Page 25 of 42


School Of Mining Engineering Underground Mine Environment
 2003 University Of New South Wales Module 1.2 Fans And Auxiliary Ventilation

Derivation Of Fan Pressures With Fan At Inlet

When a fan is located at the intake to a duct or airway, the fan pressures are related as follows, refer
Figure 4.2 A;

For a force fan assuming SPin = 0 VPin VPout


SPin SPout
Fan total pressure = Outlet total – inlet total TPin TPout

TPfan = TPout - TPin

Fan static pressure = fan total pressure – outlet velocity pressure

and, SPfan = TPfan - VPout by definition

therefore SPfan = (SPout + VPout) - (0 + 0) – VPout

SPfan = SPout = duct side tube pressure.

For example, if the fan static pressure is 400 Pa and the duct velocity pressure is 200 Pa, the facing
tube would read TPout = 600 Pa and the side tube would read SPout = 400 Pa.

For a fan located at the inlet to a duct the fan static pressure is measured by the discharge side tube
and fan total pressure by the discharge facing tube.

Derivation Of Fan Pressures With Fan Located In A Duct

When a fan is located in the middle of a duct then, assuming no change in velocity pressure, the fan
total pressure is given by the difference in inlet and outlet total pressure, refer Figure 4.2B.

Fan total pressure = Outlet total – inlet total VPin VPout


SPin SPout
TPfan = TPout - TPin TPin TPout
TPfan = (SPout + VPout) - (SPin + VPin)

TPfan = SPout - SPin

Fan static pressure = fan total pressure – outlet velocity pressure

SPfan = TPfan – VPout

For example, if the fan static pressure is 400 Pa and the duct velocity pressure is 200 Pa, the
difference between inlet and outlet facing tubes would read TPout - TPin = 600 Pa, that is the fan total
pressure. The difference in side tube readings SPout - SPin would also be 600 Pa.

The fan static pressure then equals 600 – 200 = 400 Pa.

File UNSW-UME-(M1-2_Fans)(R1) Page 26 of 42


School Of Mining Engineering Underground Mine Environment
 2003 University Of New South Wales Module 1.2 Fans And Auxiliary Ventilation

Derivation Of Fan Pressures With Fan At Outlet

If a fan is located at the outlet of a duct or airway, the fan pressures are related as follows, refer Figure
4.2 C;

Fan total pressure = Outlet total – inlet total

TPfan = TPout - TPin

Fan static pressure = fan total pressure – outlet velocity pressure


VPin Improved diffuser
SPfan = TPfan – VPout VPout efficiency but
SPin at increased
TPin fabrication cost
therefore minimising VPout leads to static pressure regain

SPfan = ( TPout - TPin )– VPout

SPfan = (SPout + VPout) - (SPin + VPin) – VPout

For an exhaust fan SPout = 0, therefore

SPfan = (0 + VPout) - (SPin + VPin) – VPout

SPfan = - (SPin + VPin) = inlet duct facing tube pressure

For a fan located at the outlet of a duct the fan static pressure is measured by the intake facing tube
and fan total pressure by the intake side tube tube.

For example, if the fan static pressure is 400 Pa and the duct velocity pressure is 200 Pa, inlet facing
tube would read TPin = - 400 Pa (fan static) and the side tube would read SPin –600 Pa (fan total). The
velocity pressure = TPin - SPin = -400 - -600 = +200 Pa.

File UNSW-UME-(M1-2_Fans)(R1) Page 27 of 42


School Of Mining Engineering Underground Mine Environment
 2003 University Of New South Wales Module 1.2 Fans And Auxiliary Ventilation

Figure 4.2A. Fan At Start Of Duct Figure 4.2B Fan In Middle Of Duct Figure 4.2C Fan At Outlet Of Duct

Friction = 400 Pa VP = 200 Pa Friction = 300 Pa VP = 200 Pa Friction = 100 Pa Friction = 400 Pa VP = 200 Pa

A B C A B C D A B C

300 Pa
400 Pa
600 Pa 600 Pa 400 Pa
-500 Pa

-300 Pa 100 Pa

VP

TP
SP VP
VP VP VP
0 0 0
VP
TP SP SP
TP

VP

1. Fan total pressure = delivery facing gauge 1. Fan total pressure = change in side and facing gauge 1. Fan total pressure = change side and facing gauge
2. Fan static pressure = delivery side gauge 2. Fan static pressure = fan total minus fan velocity 2. Fan static pressure = inlet facing gauge.
3. Facing gauge pressure at duct start A = 0
4. Side gauge pressure at duct start A = - VP
5. Side gauge pressure at duct outlet C = 0
6. Facing tube pressure at duct outlet C = VP

File UNSW-UME-(M1-2_Fans)(R1) Page 28 of 42


School Of Mining Engineering Underground Mine Environment
 2003 University Of New South Wales Module 1.2 Fans And Auxiliary Ventilation

4.1 Inlet Bends


The calculation of frictional and shock losses for circuit components are covered in detail in the Fluid
Flow Friction And Shock Loss module. However, losses associated with main fan inlet bends are
worthy of note.

It is important to recognise that fan characteristic curves may be provided without including inlet bend
losses, but network simulation and some pressure measurement locations focus on shaft collar static
(side tube or piezometer ring) pressures. Consequently, significant losses associated with the inlet
bend may be omitted from a complete analysis of the system. In addition, if there is a change in cross
sectional area between the shaft collar and fan inlet, the change in velocity pressure may also be
significant and influence the interpretation of pressure measurements taken, refer Figure 4.3.

Figure 4.3 Surface Fan Bends

Bifurcated Second fan in


Fan duct static bend parallel
and velocity
pressure

Turning
Inlet box Shaft static vanes
and velocity
pressure

The static pressure loss around the bend is determined by the velocity of air entering the bend and the
bend geometry in terms of a shock loss factor X, as follows;

Equation 4.2 Bend Shock Loss

Shock loss Ps = X. 1 . p . V2 Pa
2

The shock loss factor can range from about 0.1, for a well designed bend with turning vanes and large
inlet diameter, to about 1.1 for a single piece mitre bend without turning vanes. The shock loss is
calculated using the inlet velocity, for example at the shaft collar.

In main mine fan installations it is common for two or more fans to be connected in parallel with fan
inlet diameters being smaller than that of the shaft collar. Therefore, in addition to the static pressure
loss due to inefficiencies of the bend, shock loss also occurs at the contraction together with a change
in static pressure corresponding to the change in velocity pressure.

The calculation method described in the worked example below, is based on the fact that velocity
pressures can be calculated at each point based on cross sectional area and quantity. All static and
total pressures are then calculated from the observed collar pressure.

File UNSW-UME-(M1-2_Fans)(R1) Page 29 of 42


School Of Mining Engineering Underground Mine Environment
 2003 University Of New South Wales Module 1.2 Fans And Auxiliary Ventilation

Worked Example – Fan Inlet Bend


3
210 m /s exhausts from a 4.5 m diameter shaft into a bend with a shock loss factor of 0.75 Two identical
3
fans are located in parallel and the diameter of each fan inlet duct is 2.1 m. The air density is 1.17 kg/m

If the shaft collar static (side tube) pressure is –1850 Pa, what are the total, static and velocity pressures
at each fan inlet duct ?

Shaft area A = π D2/4 = π 4.52/4 = 15.9 m2

Shaft velocity V = Q/A = 210/ 15.9 = 13.2 m/s

Shaft velocity pressure VP = . 1. p . V2 = 1. 1.17. 13.22 = 102 Pa


2 2

Shaft total pressure TP = SP + VP = -1850 + 102 = -1748 Pa

Similarly for each fan

Fan duct area = π 2.12/4 = 3.46 m2

Quantity through each fan = 210/2 = 105 m3/s

Fan duct velocity = 105/3.46 = 30.3 m/s

Fan duct velocity pressure = 1. 1.17. 30.32 = 538 Pa


2

Static pressure change Ps = X. 1. p .V2 = 0.75 x 102 = 76 Pa


due to shock loss in bend 2

Change in velocity pressure through bend = 538 – 102 = 436 Pa

Static pressure in fan ducts = -1850 – 76 – 436 = -2362 Pa


Note velocity pressure increased therefore
static pressure must decrease

Total pressure in fan ducts = SP +VP = -2362 + 538 = -1824 Pa

Check – shock loss in bend must be accounted for by change in total pressure

Change in total pressure = -1824 – (-1748) = 76 Pa

Exhaust fan static pressure = inlet facing tube= -1824 Pa and NOT –2362 Pa

File UNSW-UME-(M1-2_Fans)(R1) Page 30 of 42


School Of Mining Engineering Underground Mine Environment
 2003 University Of New South Wales Module 1.2 Fans And Auxiliary Ventilation

4.2 Outlet Diffusers


The total pressure developed by a fan comprises a static pressure and velocity pressure component.
The static pressure component overcomes the resistance of the system and is the value that should
be compared with calculated frictional and shock losses.

If the discharge velocity pressure is reduced by increasing the cross sectional area of the fan outlet
then, in accordance with Bernoulli’s equation, there will be a corresponding increase in static pressure.
The maximum possible regain, for a given diffuser, is therefore the difference between intake and
discharge velocity pressures.

Worked Example – Outlet Diffusers


3 3
A 2m diameter fan exhausts 90 m /s at a density of 1.2 kg/m . The fan total pressure is 1,900 Pa. What is
the discharge velocity pressure and fan static pressure ?

Fan exhaust area A = π D2/4 = π 2.02/4 = 3.14 m2

Velocity pressures VP = 0.5 .ρ. V2 = 0.5 .ρ. Q2 = 0.5 x 1.2 x 902 = 493 Pa
A2 3.142

Fan static pressure = fan total – discharge velocity = 1900 – 493 = 1407 Pa

What is the effect of reducing the fan outlet velocity pressure to 100 Pa ?

Change in discharge velocity pressure = 493 – 100 = 393 Pa

Fan total pressure is constant therefore change in fan static pressure = change in velocity pressure

Fan static pressure = 1407 + 393 = 1800 Pa.

Note that fan total pressure = fan static + discharge velocity = 1800 + 100 = 1900 Pa constant.

However, because diffusers are not aerodynamically perfect, frictional and shock losses occur.
Diffuser losses are dependent on area ratios, angle of divergence, length ratio and centre cone, if
used. The optimum diffuser size is determined by cost of fabrication, operational issues (auxiliary fans)
and net effect on fan performance, refer Figure 4.4.

Figure 4.4 Optimum Diffuser Geometry


L 1
Shock loss Optimum angle
3.5

7
0.8 3.0
8
θ/2 9
Shock Loss K

D1 D2 0.6
10
2.5

A2/A1
A1 A2 12
θ U1 U2
0.4
15
2.0

VP1 VP2 0.2 20 1.5

θ/2
0 1.0

D2 = D1 + 2 x L x tan(θ/2)
0 1 2 3 4 5 6 7 8
L/De

The efficiency of a diffuser is then given by the actual static pressure increase compared to the
maximum possible i.e difference in velocity pressures.

Equation 4.2 Diffuser Efficiency

Diffuser efficiency η = static pressure increase = P2 – P1 x 100 %


velocity pressure decrease VP2 – VP1
Note that for a diffuser discharging to atmosphere P2 = 0 Pa gauge

File UNSW-UME-(M1-2_Fans)(R1) Page 31 of 42


School Of Mining Engineering Underground Mine Environment
 2003 University Of New South Wales Module 1.2 Fans And Auxiliary Ventilation

Worked Example – Outlet Diffusers


3
A 2m diameter fan outlet is to be fitted with a diffuser. The exhaust quantity is 90 m /s with a density of
3
1.2 kg/m . If a L/De ratio of 4.0 is used, what is the length, out let diameter and area of the diffuser for
optimum design ?

Inlet area A1 = π D2/4 = π 2.02/4 = 3.14 m2

Length of diffuser L = De x L/De = 2.0 x 4.0 = 8.0 m

Optimum angle = 9.0 O from graph Figure 3.4

Outlet diameter D2 = D1 + 2 x L x tan(θ/2) = 2 + 2 x 8.0 x tan(9/2) = 3.26 m

Outlet area A2 = π D2/4 = π 3.262/4 = 8.34 m2

Area ratio AR = A2/A1 = 8.34/3.14 = 2.66 (check against graph)

What is the pressure regain in the diffuser and what is the efficiency ?

Diffuser K factor for L/De = 4.0 is 0.32 from graph

Velocity pressures VP = 0.5 x ρ x V2 = 0.5 x ρ x Q2


A2

VP1 = 0.5 x 1.2 x 902 / 3.142 = 492 Pa

VP2 = 0.5 x 1.2 x 902 / 8.342 = 70 Pa

Diffuser regain = (1 – k) x VP1 = (1 – 0.32) x 492 = 334 Pa

Diffuser efficiency η = static pressure increase x 100 = 334 x 100 = 79 %


velocity pressure decrease (492 – 70)

File UNSW-UME-(M1-2_Fans)(R1) Page 32 of 42


School Of Mining Engineering Underground Mine Environment
 2003 University Of New South Wales Module 1.2 Fans And Auxiliary Ventilation

5.0 AUXILIARY VENTILATION


Auxiliary ventilation is required to manage gas, dust and heat in the immediate vicinity of development
faces in advance of a primary ventilation circuits. Auxiliary ventilation may also be used for ventilation
of other “blind” ends, for example sumps.

The basic design process is the same as for other ventilation systems i.e. what quantity of air is
required to manage contaminants present to design limits. A critical factor for auxiliary ventilation is
also the condition of supply air in terms of gas concentration and temperature. In force configurations,
heating of air through the auxiliary fan must also be taken into account.

The capacity of auxiliary ventilation systems is typically limited by practicable duct diameters in the
available excavation profile, length of duct and limiting burst or collapse pressure rating of the duct
material used. Supplementary controls may be required when contaminant loads cannot be managed
with practicable auxiliary ventilation rates, for example gas drainage or localised refrigeration.

There are principally three types of ducted auxiliary ventilation systems used, refer Figure 5.1. These
are forcing systems, exhausting systems and combinations of the two. An additional variation is the
use of rigid steel or fibre glass ducting that can withstand both positive and negative pressures or lay
flat ducts that can only be used with positive pressures.

Figure 5.1 Some Auxiliary Ventilation Options

Fan &
scrubber

Force Exhaust Force-Exhaust Force Overlap 2 hdg twin exhaust fans

Forcing systems are generally preferred in metalliferous mines because the main contaminant
loading occurs during blasting (no persons present) allowing cheaper layflat ducts to be used in
relatively long (20 to 100 m) lengths and diameters up to circa 2.0 m. Dust suppression is then
required during the loading cycle.

Exhausting systems are generally preferred in coal mines because the development process is
continuous requiring seam gas and dust management at the immediate face position. In addition, pillar
lengths of 60 to 100 m, do not extend ducts beyond around 200 m. Typically, rigid fibreglass ducts are
used in lengths of 2 to 6 m and diameters of 600 to 750 mm.

Force overlap or force – exhaust systems are employed for longer single entry coal mine
development headings where face gas and dust management is required in addition to higher
ventilation rates afforded by larger diameter lay flat ducts.

File UNSW-UME-(M1-2_Fans)(R1) Page 33 of 42


School Of Mining Engineering Underground Mine Environment
 2003 University Of New South Wales Module 1.2 Fans And Auxiliary Ventilation

The resistance of ducts can be calculated from standard surface roughness heights or from design
charts available from suppliers. The roughness height of smooth layflat and fibreglass ducts ranges
1.0 to 2.0 mm with K factors ranging 0.003 to 0.0035 Ns2/m8. Corrugated or spiral wound ducts have
effective roughness heights of 5 to 10 mm with K factors ranging 0.0045 to 0.0060 Ns2/m8 . These
values vary with the quality of installation and should be confirmed by site measurements.

If ducts did not leak, the fan operating point would then be calculated directly from the quantity of air to
be delivered at the face and the frictional losses occurring within the duct. However, all ventilation
ducts leak through seams and joints at a rate dependent on applied pressure differentials and the
leakage resistance of the duct, refer Figure 5.2.

Figure 5.2 Auxiliary Ventilation Duct Leakage.

Leakage Leakage Leakage


through through through
joints seam holes/tears
Q face

High Low
Q fan
pressure pressure
P fan

Q duct Quantity in duct


P duct Q face
Pressure in duct

The rate of leakage increases with differential pressure across the duct wall. Therefore, in both force
and exhaust ducts, the quantity of air delivered by the fan is greater than that delivered to the face. It
should be recognised that in force systems, leakage out of the duct may be considered “useful” where
as in exhaust systems leakage into the duct may result in an unacceptable reduction in airway
velocity.

The performance of an auxiliary duct system is described by two ratios and with it’s effective
resistance, as follows;

Equation 5.1 Ducts With Leakage

Volume increase ratio VIR = Quantity through fan


Quantity delivered to face

Pressure increase ratio PIR = Pressure required for face quantity (with leakage)
Pressure required for face quantity (no leakage)

Effective resistance Reff = Pressure required for face quantity (with leakage)
Quantity through fan 2

In a perfect duct with no leakage, the quantity through the auxiliary fan equals that at the face and VIR
= 1.0. There is no leakage, therefore the pressure increase ratio PIR = 1.0.

When leakage occurs the quantity through the fan is greater than that at the face. The volume
increase ratio VIR is then greater than 1.0 and increases with lower leakage resistance. The pressure
required to deliver the same face quantity also increases due to frictional losses associated with the
higher fan quantity, therefore in leaking ducts PIR is also greater than 1.0.

The resistance of duct leakage paths are given in terms of leakage quantity over a defined length at a
specified pressure, for example m3/s per 1000 m at 100 Pa (Le Roux,1990), or as a resistance over a

File UNSW-UME-(M1-2_Fans)(R1) Page 34 of 42


School Of Mining Engineering Underground Mine Environment
 2003 University Of New South Wales Module 1.2 Fans And Auxiliary Ventilation

defined length, for example Ns2/m8 per 100 m. The values given are an average for the duct length
and do not account for a one off tear or large hole. It should also be recognised that, for a given
material quality, the leakage resistance reduces with increased duct diameter due to the increase in
surface area available for leakage to occur. Some typical values of leakage resistance are shown in
Table 5.1.

Table 5.1 Values Of Duct Leakage Resistance


Leakage Leakage rate Leakage rate
Resistance m3/s m3/s
Installation Standards Ns2/m8 per 100 m per 100 Pa per km per kPa per 100 m
Poor standards or short lengths 2500 2.00 0.6325
Good standard short lengths 5000 1.41 0.4472
Average standards longer lengths 10000 1.00 0.3162
Excellent standards longer lengths 15000 0.82 0.2582

The relationship between frictional pressure losses, duct quantity, leakage rate and fan operating point
requires an iterative solution, or application mathematical techniques described by Vutukuri, 1986. An
alternative is to use the leakage nomogram provided by Le Roux (page 80 based on NCB research).

Figure 5.3 Duct Leakage Nomogram


1.01
1.01
20

1 1.02
30
1.02
40
1.03
2 50
8 1.03 1.04
7 60
3 6 70
80 1.04 1.05
4 5 90
100 1.05 1.06
5
6
760 duct 4
1.06
1.07
1.08
7 3 1.07
8 1.09
1.08

Pressure Increase Ratio PIR


9 1.10
2 1.09
10 200 1.10
1.15
300
20 1 1.15
0.9 1.20
0.8 400
0.7 1.20
30 500
0.6
1.3
40 0.5 600 1.3
Leakage Coefficient m3/s per km at 0.1 kPa

50 610 duct 0.4 700


Duct resistance Ns2/m8 per 100 m

800 1.4
60 900 1.4
70 0.3
1000 1.5 Volume increase ration VIR
80 1.5
90 1.6
0.2 1.6 1.7
100
1.7 1.8
1.8 1.9
1.9 2.0
2000 2.0
200 0.1
0.09
0.08
Length of duct m

3000
300 0.07 3
0.06 3
400 4000 4
0.05
500 0.04 5000 5 4
600 6 5
700 0.03 6000 7
800 7000 8 6
900 8000 9 7
0.02 9000 10
1000
10000

0.01

File UNSW-UME-(M1-2_Fans)(R1) Page 35 of 42


School Of Mining Engineering Underground Mine Environment
 2003 University Of New South Wales Module 1.2 Fans And Auxiliary Ventilation

Worked Example – Leakage From Ducts


3
A 250 m long exhaust duct must circulate 8 m /s to the face area. The following duct sizes are to be
considered.
2 8 3
610 mm R100 = 30.09 Ns /m leakage coeff = 1.00 m /s per 100 Pa per km
2 8 3
760 mm R100 = 9.31 Ns /m leakage coeff = 1.00 m /s per 100 Pa per km

What is the required fan pressure – quantity duty for each duct ?

Resistance of ducts Rtotal = Length x R100 No leak pressure loss = Rtotal x Qface2
100

610 mm duct Rtotal = 250 x 30.09 = 75.2 Ns2/m8 No leak pressure loss = 75.2 x 82 = 4,813 Pa
100

From nomogram VIR = 1.15 Fan quantity = VIR . Qface = 1.15 x 8 = 9.2 m3/s

PIR = 1.13 Fan pressure = PIR . Pnoleak = 1.13 x 4813 = 5,439 Pa

760 mm duct Rtotal = 250 x 9.31 = 23.3 Ns2/m8 No leak pressure loss = 23.3 x 82 = 1491 Pa
100

From nomogram VIR = 1.08 Fan quantity = VIR . Qface = 1.08 x 8 = 8.64 m3/s

PIR = 1.065 Fan pressure = PIR . Pnoleak = 1.065 x 1491 = 1588 Pa

This highlights the profound effect that relatively small increases of duct diameter have on fan pressure and
leakage.

File UNSW-UME-(M1-2_Fans)(R1) Page 36 of 42


School Of Mining Engineering Underground Mine Environment
 2003 University Of New South Wales Module 1.2 Fans And Auxiliary Ventilation

5.1 Jet Fans


A further application of auxiliary fans, that is common in civil tunnels but less common in underground
mines, is as open circuit booster fans, refer Figure 5.4.

Figure 5.4 Jet Fan As An Open Circuit Booster

V1 V2 A2 Q2
A1
Q1
V3

One or more fans are located in the airway and operated without any duct attached. The high velocity
fan jet creates a forward force over the tunnel area dependent on the momentum (mass x velocity) of
the jet stream. This resultant pressure is then given by the force created divided by the cross sectional
area to which it is applied.

Thrust of the jet = ρ . Q2 . V2 = ρ . V22 . A2 Newtons with respect to fan outlet

And the fan static pressure Ps = 0.5 ρ . V22 . V1 2


Pa
V2

The fan operating point is determined by the discharge velocity pressure and will normally only require
the operation of single stage fans.

The induced pressure in the airway ∆P = ρ . V22 . A2 Pa per fan, and for n fans in series, the pressure
developed is; A1

Equation 5.2 Pressure Developed By n Jet Fans

∆P = n . ρ . V22 . A2
A1

Worked Example – Jet Fan Pressure


3 3
A fan with a 900 mm diameter outlet, delivers 23 m /s (density = 1.17 kg/m ) in to a 4.0 x 5.0 airway with a
3
ventilation quantity of 40 m /s. What is the pressure developed by a single fan ?

Area of airway = 4.0 x 5.0 = 20 m2

Fan outlet area = π .D2 = π .0.92 = 0.636 m2


4 4

Fan outlet velocity = Q/A = 23.0 / 0.636 = 36.2 m/s

Pressure developed ∆P = n . ρ . V22 . A2 = 1 x 1.17 x 36.22 x 0.9 = 69 Pa


A1 20

Jet fans in this configuration cannot generate very high differential pressures. They are suitable for
directional control in complex working areas or where bulkheads would be problematic, for example
trucking horizons, and where frictional losses in individual airways are of the same order of magnitude
as the pressure created by the fan(s).

File UNSW-UME-(M1-2_Fans)(R1) Page 37 of 42


School Of Mining Engineering Underground Mine Environment
 2003 University Of New South Wales Module 1.2 Fans And Auxiliary Ventilation

A second application of jet fans is for increasing local air velocity in order to improve the cooling power
of the environment or to scour short stubs. Approximate equations for prediction of jet profile formation
are as follows;

Vx = K.Vo √Ao m/s Vo


Vx
X Ao
Qo Qx
X
Qx = 2.Qo. Vo m3/s
Vx

X = distance from fan outlet (m)


Vx = maximum velocity along centre line at distance X (m/s)
Vo = outlet velocity (m/s)
Ao = outlet area (m2)
Qo = fan delivery (m3/s)
Qx = total entrained airflow at distance X (m3/s)
K = constant (6.5 for circular outlet)

These calculations can be undertaken for site fans or preferably, most fan suppliers will have a range
of portable units designed for the purpose. An example 760 mm “man cooler” fan profile is shown in
Figure 5.5

Figure 5.5 Jet Profile ( 760 mm Diam 5.0 kW Fan 8.8 m3/s)

8m
1.25 m/s

2.50 m/s

Fan 5.0 m/s 3.75 m/s

8m
0 10 20 30 40 50
Distance from fan m

File UNSW-UME-(M1-2_Fans)(R1) Page 38 of 42


School Of Mining Engineering Underground Mine Environment
 2003 University Of New South Wales Module 1.2 Fans And Auxiliary Ventilation

6.0 REFERENCES

6.1 References Underpinning This Module


Le Roux W.L. 1990 “Le Roux’s Notes On Mine Environmental Control Ch.4 Mechanical Ventilation”
Published by Mine Ventilation Society Of South Africa

6.2 References Supplied With Course Notes


Daly, B. 1978 “Fans And Fan Performance”
In “Woods Practical Guide To Fan Engineering”

Wolski, J. and Barry, J 1997 “Analysis Of Multi-Fan Ventilation Duct Line: Resistance, Leakage,
Performance”, 6th International Mine Ventilation Congress, Pittsburgh

6.3 Further References


Burrows J. 1974 “Measurement Of Fan Pressures”
Journal Mine Ventilation Society South Africa, Vol 27 No.4 April 1974

Daly B.B 1978 “Woods Practical Guide To Fan Engineering”


Pub. Univ Press Camb.

De La Harpe J.H 1982 “Environmental Engineering In South African Mines –Ch.7 Basic Fan
Engineering”. Published by Mine Ventilation Society Of South Africa.1982

Howes M.J 1982 “Environmental Engineering In South African Mines –Ch.37 Noise Control”.
Published by Mine Ventilation Society Of South Africa.1982

McPherson 1971 “The Isentropic Compression Of Moist Air In Fans”


Journal Mine Ventilation Society South Africa, June 1971

Rabia H 1988 “Mine Environmental Engineering Ch 3 Fan Engineering”


Pub Entrac Software UK 1988

Vutukuri S and Lama R.D.1986 “Environmental Engineering In Mines – Airflow Analysis In Leaky
Ducts” Published Cambridge University Press. pp 60-67

File UNSW-UME-(M1-2_Fans)(R1) Page 39 of 42


School Of Mining Engineering Underground Mine Environment
 2003 University Of New South Wales Module 1.2 Fans And Auxiliary Ventilation

APPENDIX A MATHEMATICAL DESCRIPTION OF FAN CURVES


For calculation purposes, fan curves are obtained by selecting data points from the manufacturers
curves and fitting a least square parabola of the form, refer Figure A1;

Equation A.1 General Form Of Parabola

Fan pressure P = AQ2 + B.Q + C

Where A,B,C are constants which can be obtained by trendlines in Excel or statistical analysis. It is
important to note that the equation obtained should only be used in the range covered by the data
points from which it is obtained. The shape of a parabola is such that extrapolation outside this range
may provide two quantity solutions for the same pressure.

Figure A.1 Mathematical Description Of Fan Curves

Fitted fan curve


Stall point
P = AQ2 + B.Q + C
System resistance
Pressure
Pa P = R.Q2
Operating point

Actual fan curve

nd
2 solution to
quadratics
Open circuit
capacity

Quantity m3/s

The open circuit capacity of the fan curve is given by the intersection of the curve with the X axis i.e.
the quantity at which pressure is zero. A.Q2 + B.Q + C = 0 .The solution for this quadratic equation is

Equation A.2 Solution For Open Circuit Capacity

Open circuit quantity Q = -B- (B2 - 4.A.C)


2.A

The stall point for the fitted curve only (not necessarily the actual fan curve) is given by the turning
point where the gradient of the pressure – quantity curve is zero i.e. dP/dQ = 0

Differentiating equation A1 with respect to Q dP = 2A.Q + B = 0 , therefore;


dQ

Equation A.3 Solution For Stall Quantity

Stall quantity Q = -B
2.A

For a system resistance curve P = R.Q2 , the intersection point with the fan curve is the pressure
quantity solution for the operating point. At this point;

File UNSW-UME-(M1-2_Fans)(R1) Page 40 of 42


School Of Mining Engineering Underground Mine Environment
 2003 University Of New South Wales Module 1.2 Fans And Auxiliary Ventilation

P = A.Q2 + B.Q + C = R.Q2 Therefore, by rearrangement (A - R).Q2+ B.Q + C = 0 for which the
quadratic solution is;

Equation A.4 Solution For Operating Quantity

Operating quantity Q = -B- (B2 - 4.(A - R).C)


2(A – R)

The pressure can then be found by substituting Q in either resistance or fan curve equations.

The gradient of the resistance curve for any value of Q is dP/dQ = 2RQ Pa per m3/s and the gradient
of the fan curve at any point is 2AQ + B Pa per m3/s. Both of these terms are required for the
calculation of ventilation network solutions by iterative techniques.

To allow for speed and density changes or multiple fans, the basic fan equation constants are
modified as follows;

Equation A.5 Correction Of Fan Curve Equation Constants


2
A’ = A. Ns . ρ1 B’ = B. Ns . S1 . ρ1 C’ = C. Ns . S1 . ρ1
Np2 ρ Np S ρ S ρ

A’ B’ C’ = corrected constants from base characteristic curve


Ns = number in series
Np = number in parallel
Sρ = base curve speed and density
S1 ρ1 = new speed and density.

File UNSW-UME-(M1-2_Fans)(R1) Page 41 of 42


School Of Mining Engineering Underground Mine Environment
 2003 University Of New South Wales Module 1.2 Fans And Auxiliary Ventilation

APPENDIX B FAN CURVE ANALYSIS SPREADSHEET


The following spreadsheet can be used for general solutions of fan curves and system resistance.
=G5*($C$7/$C$3)^2 =J5*($C$8/$C$4) =M5*C$10
=F5*$C$7/$C$3 =I5 =L5*C$9

A B C D E F G H I J K L M N O P
2 Single Unit Base Curve BASE CURVE Speed Correction Density Correction Parallel/Series
3 Speed 300 rpm Quantity Pressure Quantity Pressure Quantity Pressure Quantity Pressure
4 Density 1.2 kg/m3 m3/s Pa m3/s Pa m3/s Pa m3/s Pa
5 100 2200 117 2994 117 2845 233 2845
6 Actual Installation 155 2000 181 2722 181 2586 362 2586
7 Speed 350 rpm 190 1500 222 2042 222 1940 443 1940
8 Density 1.14 kg/m3 212 1000 247 1361 247 1293 495 1293
9 No Parallel 2 235 500 274 681 274 647 548 647
10 No Series 1 250 0 292 0 292 0 583 0
11
12
13 System Resistance Base Curve Actual Curve Resistance
14 Resis 0.012 Ns2/m8
15 NVP 200 Pa 3000
16 Quantity P=RQ2 P - NVP 2750
17 0 0 -200 2500
18 65 50 -150 2250
0 130 202 2 2000
19
1750
Pressure Pa

20 194 454 254


1500
21 259 807 607
1250
22 324 1260 1060
1000
23 389 1815 1615
750
24 454 2470 2270
500
25 519 3226 2270
250
26 583 4083 2270
0
27 -250 0 50 100 150 200 250 300 350 400 450 500 550 600 650
28 -500
29 =B25+MAX(O$5:O$12)/9 Quantity m3/s
30

=C$14*B26^2 =IF(C26-C$15<=MAX(P$5:P$10),C26-C$15,D25)

Cell Comment
C:3 &4 Base case fan curve speed and density
C:7 to 10 Actual installation speed, density, number parallel and series. Note both series and parallel =1
for single fan unit.
F:5 to10 Base case fan curve quantity points
G:5 to 10 Base case fan curve pressure points
I:5 to 10 Quantity corrected for actual speed
J:5 to 10 Pressure corrected for actual speed
L:5 to 10 Quantity corrected for actual density ie no change !!
M:5 to 10 Pressure corrected for actual density
O:5 to 10 Quantity corrected for number of fans in parallel
P:5 to 10 Pressure corrected for number of fans in series
C:14 System resistance
C:15 Natural ventilation pressure
B17:26 Quantity for calculation of system pressure – up to maximum fan quantity
C17:26 Frictional pressure drop
D17:26 System pressure net of natural ventilation pressure- up to maximum fan pressure.

The operating point is estimated from the intersection of system resistance and actual fan curve
graph. Alternatively you can use the mathematical description of fan curves described in Appendix A
to calculate the point directly.

File UNSW-UME-(M1-2_Fans)(R1) Page 42 of 42


© 2003 UNSW Underground Mine Environment Module 1.2 Fans And Auxiliary Ventilation Photographs © 2003 UNSW Underground Mine Environment Module 1.2 Fans And Auxiliary Ventilation Photographs

Ventilation And Mine Services Large Centrifugal Installations


Circular diffusers Rectangular diffusers
Fans And Auxiliary Ventilation Photographs

Eichenburg Bend

Low shock loss bend

Triple Centrifugal Fan Installation Evase

Bend &
Contents Explosion
Motor Doors
– Centrifugal Fans
– Axial Fans
– Bends, control and monitoring
Volute
– Silencers
– Boosters & Auxiliary Ventilation Drive shaft
Back draught Damper Doors (Open)

© 2003 UNSW Underground Mine Environment Module 1.2 Fans And Auxiliary Ventilation Photographs © 2003 UNSW Underground Mine Environment Module 1.2 Fans And Auxiliary Ventilation Photographs

Centrifugal Fans Other Centrifugal Configurations

Large Drop Out Box


Centrifugal Impeller
Mixed Flow Impellers Side Discharge

Upcast &
downcast shaft
Twin Inlet Fan On Force Downcast

Side Discharge

© 2003 UNSW Underground Mine Environment Module 1.2 Fans And Auxiliary Ventilation Photographs © 2003 UNSW Underground Mine Environment Module 1.2 Fans And Auxiliary Ventilation Photographs

Other Centrifugal Configurations Larger Axial Fan Installations

Horizontal Back
draught Dampers
Side Diffusers
intake

Plenum
chamber

Twin 2 Stage Axial Surface Installation

Side Twin Axials About To Be Connected To Downcast Shaft


intake

Application in very gassy coal


mine where ventilation must be
maintained during fan
maintenance or relocation
Hoisting shaft
Very Large Single Stage Axial

1
© 2003 UNSW Underground Mine Environment Module 1.2 Fans And Auxiliary Ventilation Photographs © 2003 UNSW Underground Mine Environment Module 1.2 Fans And Auxiliary Ventilation Photographs

Other Axial Configurations Axial Fans

No Shaft Bend Jet Fan


Axial Fan Impeller

Short Drive Shaft Configuration Axial Fan With Layflat Force Duct Propeller fans in stone mine

© 2003 UNSW Underground Mine Environment Module 1.2 Fans And Auxiliary Ventilation Photographs © 2003 UNSW Underground Mine Environment Module 1.2 Fans And Auxiliary Ventilation Photographs

Bends, Control and Monitoring (1) Bends, Control and Monitoring (2)

Bend With Turning Vanes


Mitre Bend Coal Mine Explosion Doors Variable Inlet Vanes (Closed)
Bend With Turning Vanes Anti Reversing Doors

Eichenburg Bend With Concrete Drop Out

© 2003 UNSW Underground Mine Environment Module 1.2 Fans And Auxiliary Ventilation Photographs © 2003 UNSW Underground Mine Environment Module 1.2 Fans And Auxiliary Ventilation Photographs

Bends, Control and Monitoring (3) Fan Silencers

Cylindrical And Pod Silencers

Splitter Silencers Splitter Silencers

2
© 2003 UNSW Underground Mine Environment Module 1.2 Fans And Auxiliary Ventilation Photographs © 2003 UNSW Underground Mine Environment Module 1.2 Fans And Auxiliary Ventilation Photographs

Underground Boosters And Bulkhead Fans Coal Mine Auxiliary Fans


Centrifugal Auxiliary Fan And Mono railed Duct

Centrifugal Auxiliary Fan And


Rigid 700 mm Duct

Bifurcated Scrubber

© 2003 UNSW Underground Mine Environment Module 1.2 Fans And Auxiliary Ventilation Photographs © 2003 UNSW Underground Mine Environment Module 1.2 Fans And Auxiliary Ventilation Photographs

Metalliferous Mine Auxiliary Fans Jet Fans In Civil Tunnels

2 Stage axial with silencer

Inlet and outlet silencers

Single stage axial fans with silencers


Could also be used in large workshops

Compressed Air Fan

© 2003 UNSW Underground Mine Environment Module 1.2 Fans And Auxiliary Ventilation Photographs © 2003 UNSW Underground Mine Environment Module 1.2 Fans And Auxiliary Ventilation Photographs

Auxiliary Duct (1) Auxiliary Duct (2)

Fibreglass Duct With Spigot Joints Large Diameter Layflat


Example Layflat Connection And Large Diameter Layflat
Suspension
Fibreglass Straight And Ts Spiral Wound Duct Note low clearance and rough
airway profile
Spiral Wound Layflat

Rigid Spiral Wound Duct (Rough = 15 mm) Spiral Wound Duct

3
© 2003 UNSW Underground Mine Environment Module 1.2 Fans And Auxiliary Ventilation Photographs © 2003 UNSW Underground Mine Environment Module 1.2 Fans And Auxiliary Ventilation Photographs

Auxiliary Duct (3) Auxiliary Duct (4)

Layflat Joints Layflat T piece Large rip Stitch repair - leakage

Layflat cassette Hanging layflat at height

Force fan and air mover Inline fan too far from first fan LAST SLIDE RETURN TO START >>>>

4
SCHOOL OF MINING ENGINEERING
UNIVERSITY OF NEW SOUTH WALES
Australia

Underground Mine Environment Course


MODULE 1.0 VENTILATION AND MINE SERVICES

4
M2
A 1 B 2 C 3 D 6 E
M1
5

1.3 VENTILATION NETWORK ANALYSIS

A Minerals Tertiary Education Council Initiative


MINERALS
COUNCIL
Of Australia

File UNSW-UME-(M1-3_Networks)(R1d) Page 1 of 29


Revision date December 2003
School Of Mining Engineering Underground Mine Environment
 2003 University Of New South Wales Module 1.3 Ventilation Network Analysis

Contents Page

1.0 INTRODUCTION…………………………………………………………………………………... 3

2.0 AIRWAYS IN SERIES AND PARALLEL……………………………………………………….. 4

2.1 Airways In Series………………………………………………………………………………. 4


2.2 Airways In Parallel……………………………………………………………………………… 5
2.3 Resolving Networks Into Parallel And Series Airways……………………………………… 6

3.0 VENTILATION NETWORK ANALYSIS…………………………………………..……………. 12

3.1 Terminology And Relationships……………………………………………………………… 12


3.2 Kirchoff’s Laws………………………………………………………………….……………… 13
3.3 Application To Simple Networks……………………………………………………………… 15
3.4 Ventilation Network Analysis By Iterative Solution…………………………….…………… 17
3.4.1 The Hardy Cross Method……………………………………………………………..… 17
3.4.2 Establishing An Initial Ventilation Distribution………………………….……………… 19
3.4.3 Application To Simple And Compound Networks………………………………………21

4.0 COMPUTER SIMULATION PROGRAMS………………………..…………………………….. 24

4.1 Developing And Reconciling A Network………………………………….…………………. 24


4.2 Compressible Flow And Natural Ventilation Pressure………………………………….……26
4.2.1 Effect Of Compression………………………………….………………………………. 26
4.2.2 Natural Ventilation Pressure………………………………….………………………… 26
4.3 Contaminant Distribution………………………………….………………………………….. 27

5.0 REFERENCES………………………………………………………………………………….…. 28

5.1 References Underpinning This Module……………………………………………………… 28


5.2 References Supplied With Course Notes…………………………………………………… 28
5.3 Further References……………………………………………………………………………. 28

Appendix A Modifications To Frictional Loss Spreadsheet…………………………….……. 29

File UNSW-UME-(M1-3_Networks)(R1d) Page 2 of 29


School Of Mining Engineering Underground Mine Environment
 2003 University Of New South Wales Module 1.3 Ventilation Network Analysis

1.0 INTRODUCTION

For the purposes of this module, “ventilation network analysis” means the calculation of resistance,
quantity and pressure in interconnected networks of discrete airways.

These techniques are used to predict the performance of mine ventilation networks, location of
regulators and requirements for surface and or underground fans. They can also be applied to other
network flow situations, such as water or compressed air reticulation systems.

The questions confronting a mine ventilation engineer, for which network analysis is used, are;

1. What configuration of airways and fan duties are required to meet design requirements.

2. How will various design options (number of headings, shafts, additional connections) affect circuit
performance ?

3. How will the circuit perform with time as the mine expands or changes.

4. What physical ventilation devices should be in place (regulators or stoppings) in order to control
the distribution of air within the ventilation circuit

5. Can observed pressure differentials and quantity distribution be reconciled with predictive models?

With the now common use and availability of computers, it is often the case that network simulation
software is used to solve nearly all mine ventilation network problems. Whilst the answers provided
may be correct, it is important for the ventilation engineer to understand the basic principles employed.

In this respect, many problems can be resolved into simple networks that do not require iterative
techniques. Solutions provided by spreadsheets may more clearly demonstrate circuit performance
when multiple scenarios are being considered.

The level of network detail required depends on the answer being sought. For example, it is rarely
necessary to model each and every mine airway in order to determine surface fan performance or the
distribution at main ventilation splits.

Network simulation provides an important tool for the ventilation engineer, but limitations of
incompressible flow programs must be acknowledged together with recognition that the quality of
results are directly dependent on the quality of input data used.

In addition to predicting ventilation flow quantities and direction, some programs have also been
developed to model the distribution of contaminants, for example products of combustion, heat, radon
or other mine gases. These are briefly described in section 4.0 but It is not within the scope of this
module to examine these in more detail.

These module notes summarise key aspects of ventilation network analysis, formulae and calculation
methods to a standard required for this course. In this respect, they are not exhaustive and should
only be used for guidance. It is essential that candidates read and understand underpinning
references for this module. Further references are provided for candidates requiring more detailed
information.

File UNSW-UME-(M1-3_Networks)(R1d) Page 3 of 29


School Of Mining Engineering Underground Mine Environment
 2003 University Of New South Wales Module 1.3 Ventilation Network Analysis

2.0 AIRWAYS IN SERIES AND PARALLEL


The first step in understanding ventilation network analysis is to be able to determine frictional
pressure losses in individual airways and “simple” combinations of airways. In these situations, the
solution is definitive and does not require an iterative solution.

The resistance to airflow for individual airways is calculated from the density corrected Atkinson’s
equation together with allowance for shock or obstruction losses (refer Fluid Flow module). The result
of these calculations is a resistance value R Ns2/m8 for each airway and a relationship between
frictional pressure loss and quantity of air in the airway.

The pressure loss in an airway is normally assumed to follow the square law, where P = R.Q2.
Therefore, if the resistance and quantity of air in an airway is known, the frictional loss between it’s
start and end point can be calculated.

2.1 Airways In Series


For airways in series, the quantity of air in each airway is the same and the total frictional pressure
loss is given by the sum of pressure losses in each airway.

Ptot = P1 + P2 + P3 +…Pn
Ptot Rtot
Ptot = Rtot.Qtot2 = R1.Qtot2 + R2.Qtot2 +….Rn.Qtot2
Qtot is the same for each airway and cancels. R1 R2 Rn
Qtot
Therefore the total resistance, Rtot, is given by the sum of individual airway resistances.

Equation 2.1 Resistance Of Airways In Series

Rtot. = R1 + R2 +….Rn Ns2/m8


For n identical airways
Rtot. = n.Rn Ns2/m8

Worked Example – Airways In Series


2 8 2 8
Airway 1 has a resistance of 0.09 Ns /m and airway 2 has a resistance of 0.08 Ns /m
3
What is the frictional pressure loss for one of each airway in series and an air quantity of 85 m /s?

Total resistance Rtot. = R1 + R2 = 0.09 + 0.08 = 0.170 Ns2/m8

Frictional pressure loss = R. Q2 = 0.170 x 852 = 1228 Pa

3
What is the frictional pressure loss for four of airway 1 in series and an air quantity of 25 m /s?

Total resistance Rtot. = 4 x R1 = 4 x 0.09 = 0.36 Ns2/m8

Frictional pressure loss = R. Q2 = 0.36 x 252 = 225 Pa

File UNSW-UME-(M1-3_Networks)(R1d) Page 4 of 29


School Of Mining Engineering Underground Mine Environment
 2003 University Of New South Wales Module 1.3 Ventilation Network Analysis

2.2 Airways In Parallel


When airways are connected in parallel, the pressure differential between the start and end of each
airway is the same, but the quantity of air in each airway depends on individual airway resistance.

Ptot = P1 = P2 = P3 = …Pn
Ptot Rtot
Ptot = Rtot.Qtot2 2
which also = R1 Q1 = R2 Q2 =
2
Rn Qn2

And Qtot = Q1 + Q2 +…Qn R1Q12

2 Qtot R2Q22 Qtot


From P = R.Q Q = P , therefore by substitution
R
RnQn2
Ptot = P1 + P2 + ….. Pn
Rtot R1 R2 Rn

But, Ptot = P1 = P2 and therefore cancel to provide equation 2.2 for series airways.

Equation 2.2 Resistance Of Airways In Parallel

1 = 1 + 1 + … 1 Ns2/m8
Rtot R1 R2 Rn
For n identical airways
Rtot. = Rn Ns2/m8
n2

As the pressure differential is common to all airways, the quantity of air in each airway can be
calculated as follows;
Ptot = Rn Qn2 = Rtot.Q2tot

Equation 2.3 Quantity Of Air In Parallel Airway

Qn = Rtot.Q2tot m3/s
Rn

Worked Example – Airways In Parallel


2 8 2 8
Airway 1 has a resistance of 0.09 Ns /m and airway 2 has a resistance of 0.08 Ns /m
3
What is the frictional pressure loss for one of each airway in parallel and an air quantity of 85 m /s?

1 = 1 + 1 = 1 + 1 = 3.333 + 3.536 = 6.869 Rtot = 1 / 6.8692 = 0.0212 Ns2/m8


Rtot. R1 R2 0.09 0.08

Frictional pressure loss = R. Q2 = 0.0212 x 852 = 153 Pa

How much air is in each airway ?

Q1 = Rtot.Q2tot = 0.0212 X 852 = 41.3 m3/s


RA 0.09

File UNSW-UME-(M1-3_Networks)(R1d) Page 5 of 29


School Of Mining Engineering Underground Mine Environment
 2003 University Of New South Wales Module 1.3 Ventilation Network Analysis

Worked Example – Airways In Parallel


P = 153 Pa
Q2 = Rtot.Q2tot = 0.0212 X 852 = 43.7 m3/s
RB 0.08
43.7
Check 43.7 + 41.3 = 85 m /s 3 85 85
41.3
3
What is the frictional pressure loss for four of airway A in parallel and an air quantity of 85 m /s?
2
Total resistance Rtot = R/n = 0.09/42 = 0.00563 Ns2/m8

Frictional pressure loss = R. Q2 = 0.00563 x 852 = 40.6 Pa

2.3 Resolving Networks Into Parallel And Series Airways


When a network or part of a network can be resolved into parallel or series airways, the solution can
be obtained by applying the rules and equations described above. The solution provided will be the
same as that obtained by simulation software and, for the same input data, is equally valid. This
technique can be applied using spreadsheets to allow multiple options and flow scenarios to be
modelled.

Quite often, the main problem in applying these techniques to real networks is the ability to visualise
the various parallel and series paths when the mine is drawn to reflect actual geometry. This is best
addressed by rearranging the airways for easier interpretation. For example, refer Figure 2.1

Figure 2.1 Rearranging Mine Networks


Actual circuit

6
7
5 Schematic
7
5
1 4 3
2
1 4 6
2

It is a matter of personal choice how to best represent a ventilation circuit provided that the basic
airway connections remain the same. For example, consider how electrical circuits are drawn to
represent the wiring in a house or car. They rarely represent the actual geometry of the physical wiring
but clearly show the connections and relationship of various components.

File UNSW-UME-(M1-3_Networks)(R1d) Page 6 of 29


School Of Mining Engineering Underground Mine Environment
 2003 University Of New South Wales Module 1.3 Ventilation Network Analysis

Worked Example – Resolving Networks

Consider the following network and individual airway resistances. R = 0.02


What is the total resistance of the circuit ? 7
R = 0.22 R = 0.03
5
3
1 4 6
R = 0.01 2 R = 0.09 R = 0.10
R = 0.12

The resistance of airways 2 and 3 in parallel is calculated by

1 = 1 + 1 = 1 + 1 R23 = 0.0397 Ns2/m8


R23. R2 R3 0.12 0.22

The resistance of airways 5 and 7 in series is calculated by

R57 = R5 + R7 = 0.03 + 0.02 = 0.05 Ns2/m8

The resistance of airways 5 and 7 in parallel with airway 6 is calculated by

1 = 1 + 1 = 1 + 1 R57_6 = 0.01716 Ns2/m8


R57_6. R57 R6 0.05 0.10

The total resistance of the circuit is then given by

Rtot = R1 + R23 + R4 + R57_6

Rtot = 0.01 + 0.0397 + 0.09 + 0.01716 = 0.15686 Ns2/m8


3
If the ventilation quantity in airway 1 is 100 m /s, what is the quantity in each airway and the pressure at
the start and end of each airway ?

By examining the network it can be seen that the quantity in airway 1 and 4 must be 100 m3/s. The quantity of air
in other airways depends on the parallel split.

The quantity of air in airway 3 is calculated by

Q3 = R23 x Q2 Q3 = 0.0397 x 1002 Q3 = 42.5 m3/s


R3 0.22

Therefore the quantity in airway 2 = 100 – 42.5 Q2 = 57.5 m3/s

The quantity in airway 6 is calculated by

Q6 = R56_7 x Q2 Q6 = 0.01716 x 1002 Q6 = 41.4 m3/s


R6 0.10

Therefore the quantity in airway 5 and 7 = 100 – 41.4 Q57 = 58.6 m3/s

The frictional pressure drop can then be calculated and hence the pressure at the start and end of each airway.

Resis Quantity Friction From To


58.6
Airway Ns2/m8 m3/s Pa Pa Pa -1500
1 0.010 100.0 100 0 -100 7
42.5 58.6
2 0.120 57.5 397 -100 -497 -1569
5
3 0.220 42.5 397 -100 -497 0 -100 3 -497
4 0.090 100.0 900 -497 -1397 1 4 6
5 0.030 58.6 103 -1397 -1500 100 2 100
-1397 41.4 -1569
6 0.100 41.4 172 -1397 -1569
57.5
7 0.020 58.6 69 -1500 -1569

Check total pressure from P= R.Q2 P = 0.15686 x 1002 = 1568.6 Pa. This would be the fan pressure required
to cause a flow of 100 m3/s in this circuit.

File UNSW-UME-(M1-3_Networks)(R1d) Page 7 of 29


School Of Mining Engineering Underground Mine Environment
 2003 University Of New South Wales Module 1.3 Ventilation Network Analysis

The following example applies these methods to a general coal mine circuit.

Worked Example – Resolving Mine Networks

Consider the following coal mine network and individual airway resistances. The mine has three main intakes, two
main returns and operates both maingate headings as intakes.

Longwall
Gateroad 11
7 8

R
Mains

R 6 9 10

5 4 3 2
Conveyor drift

R Ns2/m8 Transport drift


Junction Junction Per single Number
Airway From To airway Parallel 1
Transport Drift 1 3 0.029 1
Conveyor Drift 2 3 0.072 1
Mains intake 3 4 0.050 3
Mains intake 4 5 0.050 3
Mains 5 6 0.150 1
Gateroad 5 6 0.080 1
Maingate 4 7 0.050 2
Face 7 8 0.030 1
Tailgate 8 9 0.055 1
Mains return 6 9 0.050 2
Mains return 9 10 0.050 2
Exhaust shaft 10 11 0.003 1

To simplify the analysis, the circuit could be drawn like this;


Shaft
6 9 10 11

8
R R
Longwall
Mains Gateroad
7
Conveyor drift
2
5 4 3
1
Transport drift

The schematic does not reflect the actual mine geometry but all airway connections are maintained
making it easier to identify series and parallel airways. Each individual airway can represent any
number of parallel airways.

File UNSW-UME-(M1-3_Networks)(R1d) Page 8 of 29


School Of Mining Engineering Underground Mine Environment
 2003 University Of New South Wales Module 1.3 Ventilation Network Analysis

Worked Example – Resolving Mine Networks

The first step is to calculate the resistance of parallel airways in each group and for the two intake drifts.

For example, the resistance of the two drifts in parallel


(1 to 3 and 2 to 3) is; R Ns2/m8
Junction Junction Per single Number Parallel
1 = 1 + 1 = 1 + 1 R1-3 = 0.01085 Ns2/m8 Airway From To airway Parallel Resistance
Rtot. R1 R2 0.029 0.072 Transport Drift 1 3 0.029 1 0.0290
Conveyor Drift 2 3 0.072 1 0.0720
Mains intake 3 4 0.050 3 0.0056
The resistance of the three main intake headings from
Mains intake 4 5 0.050 3 0.0056
3 to 4 is; Mains 5 6 0.150 1 0.1500
Gateroad 5 6 0.080 1 0.0800
R3-4 = R/n2 = 0.05/32 = 0.00556 Ns2/m8 Maingate 4 7 0.050 2 0.0125
Face 7 8 0.030 1 0.0300
For the longwall Tailgate 8 9 0.055 1 0.0550
2 8
R4-9 = 0.0125 + 0.03 + 0.055 = 0.0975 Ns /m Mains return 6 9 0.050 2 0.0125
Mains return 9 10 0.050 2 0.0125
The completed calculations are shown in the table Exhaust shaft 10 11 0.003 1 0.0030

Gateroad Longwall 11
The next step is to combine the mains and gateroad in
parallel.
R = 0.0975
1 = 1 + 1 R5-6 = 0.0267 Ns2/m8
R5-6. 0.150 0.080 R
Mains

R 6 9 10

5 4 3
R = 0.01085

Longwall 1
11
The next step is to combine the mains and gateroad in
series with main intakes 4 to 5 and returns 6 to 9. R = 0.0267 R = 0.0975

R4-9 = R4-5 + R5-6 + R6-9

R4-9 = 0.0056 + 0.0267 + 0.0125 = 0.0448 Ns2/m8


6 9 10

5 4 3
R = 0.01085
The next step is to combine this with the longwall 4 to 9 to
provide a final series network. Longwall 1
11

1 = 1 + 1 R4-9 = 0.0159 Ns2/m8


R4-9. 0.0975 0.0448

R = 0.0975

9 10
R = 0.0448
4 3 R = 0.01085

File UNSW-UME-(M1-3_Networks)(R1d) Page 9 of 29


School Of Mining Engineering Underground Mine Environment
 2003 University Of New South Wales Module 1.3 Ventilation Network Analysis

Worked Example – Resolving Mine Networks

The total mine resistance is then given by; 11


2 8
Rtot = 0.01085 + 0.0056 + 0.0159 + 0.0125 + 0.003 = 0.04785 Ns /m R = 0.003
R = 0.0125
9 10
R = 0.0159

4 3
R = 0.0056 R = 0.01085

Knowing the resistance of individual airways and airway groups, the pressure and quantity at any point within the
network can be determined.
3
For example, if the total mine ventilation rate is 110 m /s, what is the surface fan pressure, quantity in
each airway and pressure at each junction ?

2 2
Surface fan pressure P = R.Q = 0.04785 x 110 = 579 Pa
579 Pa
Shaft
6 9 10 11
3 3
110 m /s 110 m /s

8
R R
Longwall
Mains Gateroad
7
Conveyor drift
3
110 m /s 2
5 4 3
1
Transport drift

If the mine quantity is 110 m3/s, then the quantity in airways3 to 4, 9 to 10 and 10 to 11 must also be 110 m3/s.

The quantity in the transport drift 1 to 3 is given by

Q = Rtot.Q2tot = 0.01085 X 1102 = 67.3 m3/s


R 0.029

Therefore the quantity in the conveyor drift is 110 – 67.3 = 42.7 m3/s

The quantity in the longwall 4 to 7 to 8 to 9 is given by

2
Q = Rtot.Q tot = 0.0159 X 1102 = 44.4 m3/s
R 0.0975

Therefore the quantity in airways 4 to 5 and 6 to 9 is 110 – 44.4 = 65.6 m3/s

File UNSW-UME-(M1-3_Networks)(R1d) Page 10 of 29


School Of Mining Engineering Underground Mine Environment
 2003 University Of New South Wales Module 1.3 Ventilation Network Analysis

Worked Example – Resolving Mine Networks

The quantity in the gate road is given by

Q = Rtot.Q2tot = 0.0267 X 65.62 = 37.9 m3/s


R 0.0800

Therefore the quantity in airways 4 to 5 and 6 to 9 is 65.6 – 37.9 = 27.7 m3/s

Entering the calculated quantities in the table of airway resistance allows the pressure drop in each airway to be
calculated (P=R.Q2) and hence the distribution of pressure within the circuit.
R Ns2/m8
Junction Junction Per single Number Parallel Quantity Loss Pressure Pressure
Airway From To airway Parallel Resistance m3/s Pa From Pa To Pa
Transport Drift 1 3 0.029 1 0.0290 67.3 131 0 -131
Conveyor Drift 2 3 0.072 1 0.0720 42.7 131 0 -131
Mains intake 3 4 0.05 3 0.0056 110.0 67 -131 -199
Mains intake 4 5 0.05 3 0.0056 65.6 24 -199 -222
Mains 5 6 0.15 1 0.1500 27.7 115 -222 -337
Gateroad 5 6 0.08 1 0.0800 37.9 115 -222 -337
Maingate 4 7 0.05 2 0.0125 44.4 25 -199 -223
Face 7 8 0.03 1 0.0300 44.4 59 -223 -282
Tailgate 8 9 0.055 1 0.0550 44.4 109 -282 -391
Mains return 6 9 0.05 2 0.0125 65.6 54 -337 -391
Mains return 9 10 0.05 2 0.0125 110.0 151 -391 -542
Exhaust shaft 10 11 0.003 1 0.0030 110.0 36 -542 -579

For example, for the quantity in each of the conveyor and transport drifts, the pressure drop is 131 Pa, therefore
the pressure at junction 3 is –131 Pa with respect to surface. The pressure drop in airway 3 to 4 is 67 Pa so the
pressure at junction 4 is –131 – 67 = -198 Pa etc etc.

The final solution is shown on the mine schematic for clarity.


-337 Pa -391 Pa -542 Pa -579 Pa

6 9 10 11
3 3 3
65.6 m /s 110 m /s 110 m /s
3
44.4 m /s

8 -282 Pa
R R
3 3
27.7 m /s 37.9 m /s

7 -223 Pa
Mains Gateroad
Longwall 0 Pa
-131 Pa Conveyor drift
-222 Pa -199 Pa 2
5 4 3
3 3
65.6 m /s 110 m /s
1 Transport drift
0 Pa

The purpose of this example is to demonstrate that a balanced network solution can be obtained using
basic series and parallel airway rules without resorting to simulation programs. Clearly, these
calculations are best manipulated using spreadsheets, but once established, rapid solutions to mine
problems can be obtained.

This method is appropriate when considering mine wide issues, but tends to become overly
complicated when applied to more complex networks. Computer simulation should then be used to
analyse the circuit in more detail or solve other issues such as expected flow rates after holing
development headings.

File UNSW-UME-(M1-3_Networks)(R1d) Page 11 of 29


School Of Mining Engineering Underground Mine Environment
 2003 University Of New South Wales Module 1.3 Ventilation Network Analysis

3.0 VENTILATION NETWORK ANALYSIS

Ventilation networks can range in complexity from one or two airways, in civil tunnelling projects, to
hundreds of kilometres of interconnected airways in complex multilevel metalliferous mines. The
fundamental relationships apply to all situations with appropriate consideration to effects of
compressibility of air, as identified in the Fluid Flow module.

It is generally assumed that the square law applies (P=RQ2) and that flow is incompressible. As
previously identified, this is acceptable for shallow mine circuits but may cause unacceptable errors in
deep mine applications. Approximate methods of overcoming this issue are identified in section 4.0 or
alternatively, simulation must be undertaken on a mass flow basis.

3.1 Terminology And Relationships


Consider the ventilation circuit described in Figure 3.1.

Figure 3.1 Generic Ventilation Circuit

6
A E
Mesh
Branch with a fan
or airway
1
5

B 2 D Mesh
Junction with a
or node regulator
R
Mesh 3
4
direction
C
Terms and relationships are as follows;

Branch or airway – a single airway between two nodes. For example, airway 1 is defined by junctions
A and B and the example network contains a total of 6 airways.

Airways can be free flowing, contain a fan or have a defined fixed flow. In fixed flow airways, the
simulation will determine if a fan or regulator is required. A dummy surface airway must be included to
close the network.

Node or junction – the point at which two or more airways meet. For example airways 2, 4 and 5
meet at junction D. The example network contains a total of 5 junctions.

Mesh – a closed loop of connected airways within the network. For example, airways 2,3 and 4 or
1,2,5 and 6 form closed meshes. The following rules apply to meshes;

1. All airways must be in at least one mesh.


2. Airways can be in more than one mesh.
3. Every mesh must have at least one airway that is only in that mesh and no other. This is known as
the Chord of the mesh and defines the mesh direction.
4. Meshes do not have to contain a fan or fixed quantity airway but if they do, this airway must be the
chord of the mesh. That is, fan and fixed quantity airways only occur in a single mesh and define
the direction of that mesh.
5. Every airway in a mesh has an incidence with that mesh of +1 if in the same direction as the
mesh, –1 if in the opposite direction or 0 if not contained within the mesh.

File UNSW-UME-(M1-3_Networks)(R1d) Page 12 of 29


School Of Mining Engineering Underground Mine Environment
 2003 University Of New South Wales Module 1.3 Ventilation Network Analysis

6. The minimum number of meshes required to describe a network, Nm, is related to the number of
junctions Nj and airways Na by equation 2.3.

Equation 2.3 Minimum Number Of Meshes In A Network

Nm = Na – Nj + 1

A mesh incidence matrix can be used to represent the position of airways within network meshes. In
this example there are two meshes, M1 and M2, with chords 5 and 4 respectively. The incidence
matrix would be as follows;

6
Airway No. M1 M2 A E
A to B 1 1 0 Only in mesh 1
B to D 2 1 -1 In both meshes
M1
B to C 3 0 1 Only in mesh 2 1
C to D 4 0 1 Chord of mesh 2 5
D to E 5 1 0 Chord of mesh 1
E to A 6 1 0 Only in mesh1
B 2 D

M2
Note that the incidence of airway 2 is positive for mesh 1 and R
negative for mesh 2. 3
4

3.2 Kirchoff’s Laws C

The two fundamental relationships used in network analysis are stated as Kirchoff’s laws;

Kirchoff’s First Law – the algebraic sum of air quantity at a junction is zero, or put simply, the total
flow into a junction must balance the total flow out of a junction. For example, if two airways enter a
junction (one carrying 25 m3/s and one carrying 50 m3/s) the total amount of air leaving the junction
must be 75 m3/s.

75
25

50

Kirchoff’s Second Law – the sum of frictional loss and fan pressures in each closed mesh (with
consideration to direction with respect to the mesh) must be zero in steady state conditions.

For example, the pressure drop from B to D via airway 2 must be the same as the pressure drop from
B to D via airways 3 and 4
-250 Pa
B 2 D

-50 Pa
Mesh 3 4
Direction
-200 Pa
C

If airway 4 is the chord of the mesh, a steady state condition would occur if the pressure loss in airway
2 was - 250 Pa, airway 3 was - 200 Pa and airway 4 was - 50 Pa. Note that frictional loss is negative.

The sum would be; Pmesh = P(3) + P(4) – P(2) = (-200) + (-50) – (- 250) = 0 Pa

P(2) is subtracted because it is in the opposite direction to the mesh.

File UNSW-UME-(M1-3_Networks)(R1d) Page 13 of 29


School Of Mining Engineering Underground Mine Environment
 2003 University Of New South Wales Module 1.3 Ventilation Network Analysis

If, for arguments sake, airway 2 was the chord with P(3) and P(4) in the opposite direction the sum
would be;

Pmesh = -P(3) – P(4) + P(2) = - (-200) – (- 50) + (- 250) = 0 Pa

This means that the pressure at junction D is still 250 Pa lower than B due to frictional losses but it can
only have one pressure regardless of the path through the network.

If the mesh contained a fan, the same rules apply and a possible solution would be as follows. Note
that the fan pressure is positive.
0 Pa
6
P(1) = - 300
P(2) = -250 A E
P(5) = - 340 890 Pa
Fan
P(6) = 0 (dummy surface airway)
1
Friction = -890 Pa 5 -340 Pa
-300 Pa

Fan = 890 Pa Mesh


Direction
B 2 D
-250 Pa

The sum of pressures in the mesh is then the sum of frictional losses (-890 Pa) and fan pressures
(890 Pa) or a total of 0 Pa. This assumes that the fan characteristic curve provides the air quantity at
890 Pa.

The fundamental network problem is to determine a volumetric distribution that balances quantity and
pressure at every junction and which also satisfies fan characteristics.

In fixed airways, the mesh quantities remain the same and an imbalance of pressure will exist. This
pressure determines if a fan or regulator would be required to provide the desired solution.

For example, if the pressure drop remains at –250 Pa in airway 2, but airway 4 is fixed, then the
pressure drop in airways 3 to 4 will depend on the fixed quantity. If it is fixed low, then the pressure
drop will be less than that in airway 2 and a regulator would be required. If it is fixed high, then the
pressure drop will be greater than that in airway 2 and a booster fan would be required.

Regulator Required Fan Required


-250 Pa -250 Pa
B 2 D B 2 D

R
-100 Pa 3 -375 Pa 3
4 -25 Pa 4 -175 Pa

Pressure drop 3 and 4 = -125 Pa Pressure drop 3 and 4 = -550 Pa


Regulator required = - 125 Pa C Fan required = + 300 Pa C

File UNSW-UME-(M1-3_Networks)(R1d) Page 14 of 29


School Of Mining Engineering Underground Mine Environment
 2003 University Of New South Wales Module 1.3 Ventilation Network Analysis

3.3 Application To Simple Ventilation Networks


As described previously, simple ventilation networks are those that can be resolved into a combination
of series and parallel airways. In this case the solution is definitive and does not necessarily require
iterative techniques. The techniques used in section 2 can be used to obtain a solution that obeys
Kirchoff’s laws. Consider the following worked example;

Worked Example – Simple Ventilation Networks

Calculate the resistance of the following network from A to F ?

R = 0.009 R = 0.003 R = 0.011


A 1 B 2 E 6 F

R = 0.007 3
5 R = 0.010
R = 0.004

C 4 D

Resistance of B to E = R3 + R4 + R5 and R2 in parallel

Resistance of B to E via airway 3,4 and 5 R345 = R3 + R4 + R5 = 0.007 + 0.004 + 0.010 = 0.021 Ns2/m8

Resistance of R345 parallel to airway 2 is

1 = 1 + 1 = 1 + 1 = 18.275 + 6.901 = 25.176 RBE = 1 = 0.00158 Ns2/m8


2
RBE. R2 R345 0.003 0.021 25.176

Resistance A to F = R1 + RBE + R6 = 0.009 + 0.00158 + 0.011 = 0.02158 Ns2/m8

If a fan with the following characteristic curve is installed in airway 6, what will the operating duty be ?

Quantity m3/s 100 145 190 215 235 250


Pressure Pa 2200 2000 1500 1000 500 0
The open circuit capacity of the fan is 250 m3/s, therefore plot calculated system pressure loss for 0 to 250 m3/s
as the solution must lie in this range.

A quantity increment of 50 m3/s is used. For example, at 150 m3/s the frictional loss is 0.02158 x 1502 = 486 Pa.

Quantity Pressure
m3/s Pa 2500
0 0 2250
50 54 2000
100 216 1750
150 486
Pressure Pa

1500
200 863
1250
250 1349
1000
Plotting both fan characteristic and system resistance 750
indicates an operating point (fan pressure = frictional
500
losses) of 215 m3/s at 1000 Pa.
250
2
Check 0.02158 x 215 = 998 Pa which is acceptable 0
given the graphical solution used. 0 50 100 150 200 250 300
Quantity m3/s

File UNSW-UME-(M1-3_Networks)(R1d) Page 15 of 29


School Of Mining Engineering Underground Mine Environment
 2003 University Of New South Wales Module 1.3 Ventilation Network Analysis

Worked Example – Simple Ventilation Networks

What will the ventilation rate be in each airway ?

The total ventilation rate is 215 m3/s therefore the quantity in airways 1 and 6 is 215 m3/s.

The total quantity from junctions B to E is also 215 m3/s but there is a parallel split.

To calculate the quantity in airway 2;

Qn = RBE.Q2BE m3/s Q2 = 0.00158 x 2152 = 156 m3/s


Rn 0.003

Quantity in airways 3 – 4 – 5 is therefore 215 – 156 = 59 m3/s .

Check by calculation Q345 = 0.00158 x 2152 = 59 m3/s


0.021

Calculate the frictional loss in each airway and the gauge pressures at each junction relative to A i.e
assume the pressure at A is zero.
Resistance Quantity Friction
J1 J2 No Ns2/m8 m3/s Pa
A B 1 0.009 215 416
B E 2 0.003 156 73
B C 3 0.007 59 24
C D 4 0.004 59 14
D E 5 0.01 59 35
E F 6 0.011 215 509

Junction A = 0 Pa
Junction B = 0– 416 = -416 Pa
Junction E (2) = -416 – 73 = -489 Pa
Junction C = -416 – 24 = -439 Pa
Junction D = -439 - 14 = -453 Pa
Junction E(5) = -453 - 35 = -488 Pa Note 1 Pa rounding error is acceptable – E same pressure
Junction F = -489 - 509 = -998 Pa

The solution to this circuit has been obtained without iterative techniques, as would be the case for all “simple”
circuits.

215 m3/s -416 Pa 156 m3/s -489 Pa -998 Pa


0 Pa A 2 F
1 B E 6

3
215 m3/s
3 59 m /s
5 59 m3/s

C 4 D
3
59 m /s -453 Pa
-439 Pa

Note also that the sum of pressure drops in the closed mesh formed by airways 2,3,4 and 5 is 0 Pa

Pmesh (2345) = (- 73) – (- 24) – (-14) – (- 35) = -73 + 73 = 0 Pa

And also for the mesh formed by airways 1, 2 and 6 (assuming junction A and F are both at surface)

Friction Fan
Pmesh (126) = (- 416) + (- 73) + (-509) + (998) = -998 + 998 = 0 Pa

And the quantity into and out of every junction balances. Therefore the calculated solution satisfies Kirchoff’s first
and second laws.

File UNSW-UME-(M1-3_Networks)(R1d) Page 16 of 29


School Of Mining Engineering Underground Mine Environment
 2003 University Of New South Wales Module 1.3 Ventilation Network Analysis

3.4 Ventilation Network Analysis By Iterative Solution


When a ventilation network can be resolved into a combination of series and parallel airways, the
solution for the pressure loss and quantity distribution in each airway can be readily calculated. This is
normally the case for most coal mine circuits but not always the case for metalliferous mine circuits.

Circuits that require an iterative solution contain a ventilation split that prevents it being described by
conventional series and parallel airways. For example, if an additional airway (7) is added to the circuit
analysed above and airway 6 moved to junction D, it cannot be solved directly, refer Figure 3.2.

Figure 3.2 Compound Ventilation Circuit

A 1 B 2 E

3 5
7

C 4 D 6 F

In this case a compound network is formed because airway 7 is in series with airway 3 and parallel to
both airways 2 and 4-5.

The method commonly used to solve this problem is iterative network simulation using the Hardy
Cross method.

3.4.1 The Hardy Cross Method


The Hardy Cross method employs Kirchoff’s first law to establish an initial distribution of air in the
network and then corrects each closed mesh to provide a solution that satisfies Kirchoff’s second law
throughout the network, as follows;

1. Estimate an initial distribution of air in the circuit that balances at every junction (Kirchoff’s first
law).

2. Calculate the frictional pressure drop in each airway for this initial distribution.

3. Calculate the fan pressure at the estimated quantity in airways containing the fan. The initial guess
must therefore be realistic, it cannot result in a negative fan quantity or a quantity that exceeds the
fan’s open circuit capacity.

4. Assuming the initial guess is wrong, the sum of pressures in some or all meshes will not equal
zero.

5. Correct the quantity in each airway (except fixed quantity airways) for every mesh to reduce the
error of summed pressures.

6. As some airways will be incident in more then one mesh, correcting a mesh will effect the balance
in other meshes.

7. Mesh corrections are carried out repeatedly until the quantity correction in all meshes is very
small, for example less then 0.5 m3/s.

8. The final imbalance of pressure in meshes with a fixed quantity chord determines if a fan or
regulator is required.

9. The solution is therefore determined by iteration and may not be exactly correct but will provide a
solution to within an acceptable tolerance.

File UNSW-UME-(M1-3_Networks)(R1d) Page 17 of 29


School Of Mining Engineering Underground Mine Environment
 2003 University Of New South Wales Module 1.3 Ventilation Network Analysis

The mesh correction is developed by considering the error associated with a change in quantity, as
follows;

Let Q be the initial estimated quantity and P be the frictional pressure loss at this estimated quantity.

Let Qs be the actual quantity that provides a solution and Ps be the frictional pressure loss for that
quantity.

The slope of the frictional loss curve has the units Pa per m3/s. That is, the slope of the curve
determines the change in pressure for a given change in quantity. Higher resistance curves have a
steeper gradient and therefore the change in pressure for a given change in quantity is greater, refer
Figure 3.3.
Figure 3.3 Errors Associated With Estimated Quantity

P = R.Q2
Frictional Pressure Pa

Ps
∆P Slope
P

∆Q

Q Qs
Quantity m 3/s

The problem is that, without knowing Qa, by how much should the quantity be changed to provide a
solution ?

The slope of the curve = ∆P Pa per m3/s


∆Q

For the equation P = R.Q2 by differentiation, ∆P = 2.R.Q Pa per m3/s at the estimated quantity.
∆Q

Therefore, the error in quantity ∆Q = ∆P m3/s


2.R.Q

But ∆P = Ps – P = R.Qs2 - R.Q2 Therefore, ∆Q = R.Qs2 - R.Q2 m3/s


2.R.Q

For a mesh containing b branches without fans or natural ventilation pressure, the mean mesh
correction is given by
I=b
Σ ( Ri.Qsi2 - Pi.Qi2)
I=1
∆Q = m3/s
I=b
Σ 2.Ri.Qi
I=1

File UNSW-UME-(M1-3_Networks)(R1d) Page 18 of 29


School Of Mining Engineering Underground Mine Environment
 2003 University Of New South Wales Module 1.3 Ventilation Network Analysis
I=b
I=b -Σ Pi.Qi2
But in a closed mesh, Σ Ri.Qsi2 =0 therefore ∆Q = I=1
m3/s
I=1 I=b
Σ 2.Ri.Qi
I=1

If natural ventilation pressure and fan terms are included, the final form of the correction equation is as
follows;

I=b
-Σ Pi.Qi2 – Pfan - Pnvp
I=1
∆Q = m3/s
I=b
Σ (2.Ri.Qi – Sf)
I=1

where

Pnvp = the natural ventilation pressure in the mesh


Pfan = the total fan pressure in the mesh
Sf = the slope of the fan curve, Pa per m3/s

Refer to Fans And Auxiliary Ventilation module for the mathematical description of fan curves used by
simulation programs.

The numerator is the sum of pressure terms within the mesh. If the mesh is perfectly balanced the
sum is zero and therefore the correction is zero.

The denominator is the combination of the resistance and fan curve gradients. If the sum of these is
large (high resistance) then the correction is small.

3.4.2 Establishing An Initial Ventilation Distribution


In addition to selecting meshes, an important requirement of simulation is to provide an initially
balanced circuit to initiate correction by iterative techniques. This method can also be used to provide
a balanced distribution within a ventilation circuit from observed data and again, allows simple circuits
to be analysed by spreadsheets for clarity.

The procedure is to calculate the quantity in every network airway by estimating the quantity flowing in
each chord. For reconciling survey data, the values assigned to each regulator and fan (which must be
chords) would be as observed.

File UNSW-UME-(M1-3_Networks)(R1d) Page 19 of 29


School Of Mining Engineering Underground Mine Environment
 2003 University Of New South Wales Module 1.3 Ventilation Network Analysis

Worked Example – Establishing And Initial Ventilation Distribution

Consider the example ventilation circuit and incidence matrix described above.

6
Airway No. M1 M2
A to B 1 1 0 A E
B to D 2 1 -1
B to C 3 0 1
C to D 4 0 1 Chord mesh 2 M1
1
D to E 5 1 0 Chord mesh 1
E to A 6 1 0 5

Let the initial estimate of flow in airway D to E be 160 m3/s and in B 2 D


airway C to D be 45 m3/s.

The correction to all airways in mesh 1 is then 160 m3/s in the


M2
direction of the mesh.
R
3
3
4
For example, quantity in airway A to B is + 160 m /s, but the quantity in airway D to E is 0
because it is not incident with mesh 1.
C

Airway No. M1 M2 Airway quantity


after M1 correction
A to B 1 1 0 160
B to D 2 1 -1 160
B to C 3 0 1 0
C to D 4 0 1 0
D to E 5 1 0 160
E to A 6 1 0 160

Then all airways in mesh 2 are corrected. For example, B to C is +45 m3/s and airway 2 is – 45 m3/s due to it’s
negative incidence in mesh 2. The quantity in airway 2 after mesh 2 correction is therefore 160 – 45 = 115 m3/s.
And for all airways is as follows;
160
6
Airway No. M1 M2 Airway Mesh 2 Airway E
A
quantity Correction quantity
after M1 after M2
correction correction
M1 160
A to B 1 1 0 160 0 160 1
B to D 2 1 -1 160 -45 115
B to C 3 0 1 0 45 45 160 5
C to D 4 0 1 0 45 45 115
D to E 5 1 0 160 0 160 B 2
E to A 6 1 0 160 0 160
D

M2
Although this is a simple example, it can be seen that by knowing R 45
the quantity in 2 airways, a balanced distribution in all 6 airways can 45 3
be determined. 4

File UNSW-UME-(M1-3_Networks)(R1d) Page 20 of 29


School Of Mining Engineering Underground Mine Environment
 2003 University Of New South Wales Module 1.3 Ventilation Network Analysis

3.4.3 Application To Simple And Compound Networks


The following worked examples apply this method to a simple network, which could have been solved
using parallel and series airways, and a compound network that can only solved using an iterative
method.

Worked Example – Simple Ventilation Network

Consider the mesh contained by airways 2,3,4 and 5 from the example above. The answer to the
distribution in this mesh can also be obtained by iterative techniques.
Mesh
Direction R = 0.003
A B 2 E F

R = 0.007 3
5 R = 0.010

C 4 D Chord

R = 0.004
3
Assuming that the estimated quantity in airway A to B is 215 m /s, what is the distribution from B to E ?

The first guess will be that 125 m3/s flows through airway 2 and 90 m3/s flows through airways 3,4 and 5. Also the
direction of the mesh will be taken as that of airway 5 although, in this case, it does not really matter.

Without fan or natural ventilation terms, the mesh correction is


I=b
2
- Σ Pi.Qi
∆Q = I=1 = - (R3.Q32 + R4.Q42 + R5.Q52 – R2.Q22)
I=b 2.( R3.Q3 + R4.Q4 + R5.Q5 + R2.Q2)
Σ 2.Ri.Qi
I=1
= - ( 0.007 x 902 + 0.004 x 902 + 0.010 x 902 – 0.003 x 1252) = - 123.23 = - 27.2 m3/s
2 x ( 0.007 x 90 + 0.004 x 90 + 0.010 x 90 + 0.003 x 125) 4.53

The imbalance of pressure in the mesh at the estimated quantity is 123.23 Pa, ie there is too much air in airways
3,4 and 5. This means that, after the first iteration, the quantity of every airway in the mesh must be reduced by
27.2 m3/s in the direction of the mesh.

Airway
2 = 125 - 27.2 x -1 = 152.2 correction is –ve with respect to mesh direction but airway 2 incidence is -1
3 = 90 - 27.2 = 62.8
4 = 90 - 27.2 = 62.8
5 = 90 - 27.2 = 62.8

The process is then repeated for this second, but improved, estimation of quantity. In more complex networks
this process is also repeated for every mesh with every iteration, normally with the correction being applied before
the next mesh is considered.

Second iteration correction = - 13.2 = -3.75 m3/s


3.55

Third iteration correction = 0.25 = 0.07 m3/s at this point the pressure error is 0.25 Pa and the quantity
3.42
correction is 0.07 m3/s. This would be considered acceptable.

Airway 2 quantity is calculated by 125 – 27.2x(-1) – 3.75x(-1) – 0.07x(-1) = 156.02 m3/s

Airway 3,4,5 quantity is calculated by 90 - 27.2x(+1) – 3.75x(+1) – 0.07x(+1) = 58.97 m3/s

The solution is therefore within 0.1 m3/s of that calculated by direct methods above.

File UNSW-UME-(M1-3_Networks)(R1d) Page 21 of 29


School Of Mining Engineering Underground Mine Environment
 2003 University Of New South Wales Module 1.3 Ventilation Network Analysis

Worked Example – Compound Ventilation Network

The network analysed above is made compound by inserting an airway (No.7 R = 0.005) from
junctions C to E and moving airway 6 to junction D.
Mesh 1
Direction
R = 0.003
Chord
A B 2 E
R = 0.005
R = 0.007 3 7 5
R = 0.010
Chord 4
C D F
R = 0.004
Mesh 2
Direction

The flow from A to F is 215 m3/s, what is the distribution of air quantity and pressure within the
circuit ?

There are no regulators or fans in the circuit so any airway can be the mesh chords.
There are four junctions and five airways, therefore 2 meshes (5 – 4 + 1) are required.
No airway contains a fan, therefore any airway (except No.7which is in both meshes) can be a chord.

The first guess at a balanced quantity distribution will be that 125 m3/s flows through airway 2, 90 m3/s
in airway 3 and 50 m3/s in airway 7.
R = 0.003
Q = 125
Q = 215
A B 2 E
R = 0.007
M1
Incidence Resis Q Q = 90 3 5 R = 0.010
7 R = 0.005
Airway Mesh 1 Mesh 2 Ns2/m8 m3/s Q = 175
Q = 50
2 -1 0 0.003 125 M2
3 1 0 0.007 90 C 4 D F
Q = 215
4 0 -1 0.004 40 R = 0.004
5 0 1 0.010 175 Q = 40
7 1 1 0.005 50

The network is balanced from a quantity point of view. The mesh corrections are then
calculated and applied in order.

Without fan or natural ventilation terms, the correction for mesh 1 is;
I=b
2
- Σ Pi.Qi
∆Q = I=1 = - (R3.Q32 + R7.Q72 – R2.Q22)
I=b 2.( R3.Q3 + R7.Q7 + R2.Q2)
Σ 2.Ri.Qi
I=1
= - ( 0.007 x 902 + 0.005 x 502 – 0.003 x 1252) = - 22.23 = - 8.89 m3/s
2 x ( 0.007 x 90 + 0.005 x 50 + 0.003 x 125) 2.51

The imbalance of pressure in the mesh at the estimated quantity is 22.23 Pa and the correction required is –8.89
m3/s .

File UNSW-UME-(M1-3_Networks)(R1d) Page 22 of 29


School Of Mining Engineering Underground Mine Environment
 2003 University Of New South Wales Module 1.3 Ventilation Network Analysis

Worked Example – Compound Ventilation Network

The correction of –8.89 m3/s is then made to all airways in mesh 1, as follows;
Incidence Resis Q Mesh 1 Q
Airway Mesh 1 Mesh 2 Ns2/m8 m3/s Corr m3/s
2 -1 0 0.003 125 8.89 133.89
3 1 0 0.007 90 -8.89 81.11
4 0 -1 0.004 40 0 40
5 0 1 0.010 175 0 175
7 1 1 0.005 50 -8.89 41.11

For example, airway 2 = 125 + -8.89 x –1 = 133.89 m3/s. Airways 4 and 5 are not effected
Mesh 2 correction is then calculated using these new quantities.

Without fan or natural ventilation terms, the correction for mesh 2 is;
I=b
2
- Σ Pi.Qi
∆Q = I=1 = - (-R4.Q42 + R5.Q52 + R7.Q72)
I=b 2.( R4.Q4 + R5.Q5 + R7.Q7)
Σ 2.Ri.Qi
I=1
= - ( 0.004 x 402 + 0.010 x 1752 – 0.005 x 41.112) = - 308.3 = - 72.87 m3/s
2 x ( 0.004 x 40 + 0.010 x 175 + 0.005 x 41.11) 4.23

The imbalance of pressure in the mesh at the estimated quantity is 308.3 Pa and the correction is –72.87 m3/s

The correction of –72.87 m3/s is then made to all airways in mesh 2, as follows;
Incidence Resis Q Mesh 1 Q Mesh 2 Q
Airway Mesh 1 Mesh 2 Ns2/m8 m3/s Corr m3/s Corr m3/s
2 -1 0 0.003 125 8.89 133.89 0 133.89
3 1 0 0.007 90 -8.89 81.11 0 81.11
4 0 -1 0.004 40 0 40 72.87 112.87
5 0 1 0.010 175 0 175 -72.87 102.13
7 1 1 0.005 50 -8.89 41.11 -72.87 -31.76
Note the change in direction of airway 7.This process is then repeated for meshes 1 and 2 in
turn until the correction is acceptable (say less than 0.5 m3/s) which is achieved after 3
iterations (6 corrections) to provide the final solution. In this case the initial estimate of flow
direction in airway 7 was also incorrect.

Mesh correction Mesh pressure error


350.0
300.0 Final Solution
250.0
200.0 Q = 126.1

150.0 2
B E
100.0
Q = 88.9
50.0 3 5
7 Q = 39.9 Q = 86.2
0.0

-50.0 1 2 3 4 5 6 7 8 9 10 11 12
C 4 D F
Q = 215.0
-100.0 Q = 128.8
Correction Number

The rate at which the solution is obtained (convergence) depends on the size of the network and
resistance of network airways.

File UNSW-UME-(M1-3_Networks)(R1d) Page 23 of 29


School Of Mining Engineering Underground Mine Environment
 2003 University Of New South Wales Module 1.3 Ventilation Network Analysis

4.0 COMPUTER SIMULATION PROGRAMS

Numerous network simulation programs are available and generally fall into two categories, namely
those that assume incompressible flow and those that allow for the effects of heat transfer by
undertaking analysis on a mass flow basis. Incompressible flow programs are most common as the
errors involved are often acceptable and data entry is considerably simpler.

Other programs also include subroutines for calculation of contamination (heat gas or products of
combustion) spread through the network. The general flow diagram for network simulation is shown in
Figure 4.1.

4.1 Developing And Reconciling A Network


When developing a network, the following issues should be considered;

1. Establish a systematic junction naming/numbering system that indicates junction location. Most
programmes will allow an alpha-numeric system that makes this task easier.

2. Decide exactly what problem the network model is attempting to answer. For example, if surface
fan duties are required, many underground airways can be combined into effective resistances
using series and parallel rules.

3. Start with a relatively simple network to describe all major splits but without generating a large
(>>100) number of airways.

4. The pressure lost in an airway is proportional to the square of quantity. Therefore it is important to
obtain the most accurate resistance data for airways carrying higher ventilation quantities, for
example intake or exhaust shafts. In other, internal mine airways it may be acceptable to apply a
generic or default resistance calculated from a defined height, width and K factor.

5. When comparing predicted with observed data, consider the effect of natural ventilation pressure
and the tolerance of the iterative solution. For example, in complex multi level mines, the pressure
required to distribute air within the mine workings may only be 10 to 20 % of that in main intake
and exhaust shafts. In low resistance airways, pressure differentials less than 10 Pa may result in
significant airflow.

It is not realistic to expect simulation models to accurately predict flows in each and every airway
within a mine when the pressure differentials required exceed the calculation tolerance possible
from available data.

6. Including fan curves provides an additional degree of freedom to the model. It is often easier to
“tune” a model to observed data by fixing all fan and regulator airways. Then make realistic
adjustments to resistances in order to match observed pressure differentials. Once this has been
achieved, fans can be added with greater confidence.

7. When reconciling predicted with observed data it is important to recognise the tolerance of the
solution together with that of the survey data. Quite often, engineers spend inordinate amounts of
time trying to match each and every airway within the mine to a level of precision far greater than
that obtained from survey methods. They also tend to focus on air quantity rather then both air
quantity and pressure differentials.

Survey data should include air quantities in all major ventilation splits together with pressure
differentials at cardinal points in the network ie not just surface fan pressures. An acceptable
match would normally be between 5 and 15% of observed pressure and quantity in main airways.

In some cases, for example deeper mines with high heat transfer rates, incompressible flow
programs will never predict all internal airflow correctly or even the correct flow direction. Again, it
is important to identify exactly what question is being addressed by the simulation.

File UNSW-UME-(M1-3_Networks)(R1d) Page 24 of 29


School Of Mining Engineering Underground Mine Environment
 2003 University Of New South Wales Module 1.3 Ventilation Network Analysis

Figure 4.1 General Network Simulation Program Flow Diagram

Airway and Edit airway and fan data


fan data base • Junctions
• Airway geometry
• Airway resistance
• Location of fans
Start simulation • Fan characteristics
• Fixed quantity airways

Select meshes yes

Modify
no
End
Establish circuit
balanced
distribution
Survey
data base
Reconcile with
Calculate and observed data
apply mesh Real time
corrections monitoring
data
Manual Analysis
yes • Compare with design
Network
• Alternatives considered
balanced ?
• Acceptable pressures
• Acceptable velocities
no
• Economics of solution

no Iterations
exceeded?
Output
yes • PC screen
• Tabulated results
Stop simulation • Fixed quantity
requirements
• Plot of network with data
• Fan curves and
Results operating points
data base

Contaminant no
spread ?

yes
Distribute
contaminant

yes no
Does heat
change NVP

File UNSW-UME-(M1-3_Networks)(R1d) Page 25 of 29


School Of Mining Engineering Underground Mine Environment
 2003 University Of New South Wales Module 1.3 Ventilation Network Analysis

4.2 Compressible Flow And Natural Ventilation Pressure


When using an incompressible flow program in deeper mines, errors will arise due to the changes in
density, and hence volumetric flow rate, that actually occur. If these effects cannot be directly
modelled by the software using a mass flow analysis, the following approximations may be
considered.

It is emphasised that these methods are an approximation and caution should be exercised when
using the results as the basis for decision making.

4.2.1 Effect Of Compression


In deep mines, the volumetric flow rate of air can vary quite significantly due to the change in
atmospheric pressure in intake and exhaust shafts. For example, in a 1000 m deep shaft, the absolute
pressure at the bottom would be about 11.8 kPa higher than on surface. Consequently, the volumetric
flow rate in the mine’s workings would be about 12% less than that entering the shaft collar. The
opposite effect will occur in upcast shafts.

This can be accounted for by using fixed quantity “dummy” airways between intake and exhaust shafts
to transfer the appropriate amount of air, refer Figure 4.2. Of course the amount of air to be transferred
is a percentage of the intake volume and therefore not a constant.

4.2.2 Natural Ventilation Pressure


Natural ventilation pressures should theoretically be calculated for every mesh within a network and
incorporated in the mesh correction. However, unless air density and elevation are specified
throughout the network, or the software has the ability to calculate these terms, this cannot be
achieved.

If density cannot be taken into account, then an overall mine effect can be accounted for by correcting
the surface fan curves by an appropriate amount. That is, if natural ventilation is in the same direction
of flow, then add the pressure to each fan duty point.

A major problem in deeper metallifeous mines is that internal frictional losses can be very low and in
some closed meshes of the same order of magnitude as density effects (<<20 Pa). In these situations,
incompressible flow analysis is suitable for main shafts and airways but may fail to adequately model
all internal airways, refer Figure 4.3.

Figure 4.2 Accounting For Compression Figure 4.3 Internal Mesh NVP
In Shafts
Upcast
Upcast Downcast
Q surface
Q surface
Orebody
Downcast
Airflow
reversal due to
internal NVP

Fixed % of
Q surf

Q U/G

Deep workings

File UNSW-UME-(M1-3_Networks)(R1d) Page 26 of 29


School Of Mining Engineering Underground Mine Environment
 2003 University Of New South Wales Module 1.3 Ventilation Network Analysis

4.3 Contaminant Distribution


A variety of contaminants may be modelled in ventilation networks to various degrees of
sophistication. The most common are products of combustion, heat or mine gases (seam gas or
radon).

The most simplistic approach is to assume that contaminants enter the network as point sources and
then move at the same velocity as air by ignoring the effect of diffusion. At airway junctions, the
contaminant concentration leaving is then given by the weighted average concentration entering a
junction. If the software does not incorporate these routines, the results can be manipulated with
spreadsheets to provide reasonable solutions.

Worked Example – Contaminant Spread

In the following network, airway 1 is 500 m long with a methane release rate of 9.5 l/s/100 m and airway 3
is 1100 m long with a methane release rate of 6.5 l/s/100 m. Airway 2 is uncontaminated intake air. What is
the methane concentration at the end of airway 3 ?

A
20 m3/s
1

B 3 D
C 2
25 m3/s
Methane released in to airway 1 = 9.5 x 500/100 = 47.5 l/s

Concentration at end of airway 1 = 47.5 x 100 = 0.238 % CH4


(20 x 1000)

Total air quantity entering junction B = 20 + 25 = 45 m3/s

Total methane entering junction B = 20 x 0.238 x 10 + 25 x 0 x 10 = 47.5 l/s

Total methane entering airway 3 = 47.5 l/s in 45 m3/s.

Concentration at start of airway 3 = 47.5 x 100 = 0.106 % CH4


(45 x 1000)

Methane released in to airway 3 = 6.5 x 1100/100 = 71.5 l/s

Total methane at end of airway 3 = 47.5 + 71.5 = 119 l/s

Concentration at end of airway 3 = 119 x 100 = 0.264 % CH4


(45 x 1000)

The results then provide a distribution of gas concentrations throughout the network. For simple
products of combustion analysis, a single point source would then provide an indication of all parts of
the network effected and the time taken to spread.

This approach is suitable for stable gases (methane or carbon dioxide) but is not adequate for
contaminants with a time dependency (fire propagation or radon decay) or other release functions
(heat from strata as a function of temperature differentials). In addition, the results of heat and fire
propagation models effect natural ventilation pressures and hence the ventilation distribution itself. For
accurate analysis of these situations, specialist software is required.

File UNSW-UME-(M1-3_Networks)(R1d) Page 27 of 29


School Of Mining Engineering Underground Mine Environment
 2003 University Of New South Wales Module 1.3 Ventilation Network Analysis

5.0 REFERENCES

5.1 References Underpinning This Module


Le Roux W.L. 1990 “Le Roux’s Notes On Mine Environmental Control Ch.5 Ventilation Networks”
Published by Mine Ventilation Society Of South Africa

5.2 References Included In The Course Notes

Phiri. J.K 1992 “Ventilation Simulation In Deep Hot Mines: A Comparative Analysis Of Application And
Limitations Of Volumetric Based Network Programs”, 5th International Mine Ventilation Congress,
Johannesburg.

Von Glehn, F.H. 1992 “Environmental Engineering Computer Software In Use On Mines” 5th
International Mine Ventilation Congress, Johannesburg.

5.3 Further References


McPherson M.J 1982 “Environmental Engineering In South African Mines –Ch.8 Ventilation Network
Analysis”. Published by Mine Ventilation Society Of South Africa.1982

Moreby R.G “Thermodynamic Network Simulation Incorporating Heat And Moisture Transfer”
Fourth International Mine Ventilation Congress. Brisbane 1988 pp 265-272

Wala, A., Dziurzynski, W., Tracz, J. and Wooton, D. 1995. Validation Study of the Mine Fire Simulation
Model. Proc. 7th US Mine Vent. Symp. Society of Mining Engineers: (Ed. A. Wala) 1995. pp.199-206.

Wang Y.J 1982 “Mine Ventilation And Air Conditioning- Ch.17 Ventilation Network Theory”
Pub John Wiley & Sons, New York – Ed. Hartman H.L. 1982

File UNSW-UME-(M1-3_Networks)(R1d) Page 28 of 29


School Of Mining Engineering Underground Mine Environment
 2003 University Of New South Wales Module 1.3 Ventilation Network Analysis

Appendix A Modifications To Frictional Loss Spreadsheet


The frictional loss spreadsheet developed in Fluid Friction and Shock Loss module 1.1 Appendix A,
can easily be modified to model simple circuits.

The first modification for series airways is to copy rows down and add two columns that calculate start
and end pressures.

A B C D E F G H I J K L M N O P Q R S
2 Length Height Width Area Perim Density Rough k Resis(1) X No R(total) Q V DP From To
3 m Diam m2 m kg/m3 mm Ns2/m4 Ns2/m8 Ns2/m8 m3/s m/s Pa Pa Pa
4 A1 1000 3 5.4 16.20 16.80 1.2 100 0.00807 0.03189 0.0 2 0.00797 200.00 6.2 319 0 -319
5 A2 1000 3 5.4 16.20 16.80 1.2 100 0.00807 0.03189 0.0 1 0.03189 50.00 3.1 80 -319 -399
6 A3 500 3 5.4 16.20 16.80 1.2 100 0.00807 0.01594 0.0 1 0.01594 150.00 9.3 359 -319 -678

=0
50 m3/s
200 m3/s A2 =S4
=R6 – Q6
A1
A3
150 m3/s

In this example, airway A1 comprises two parallel airways which divides into two separate airways in
series. For the defined distribution of quantity, the pressure at the end of airway A2 is –399 Pa and the
pressure at the end of A3 is –687 Pa

Cell Comment
R:4 Set to 0 Pa or whatever start pressure is required.
S:4 to S:6 Equals start pressure (column R) minus frictional loss
R:5 to R:6 Equals end pressure of previous airway eg R5 and R6 = S4

The second modification is used to calculate the split in parallel airways when they have different
resistances.

200 m3/s

Total Q
200
Parallel Length Height Width Area Perim Density Rough k Resis(1) X No R(total) Q V DP From To
Airways m Diam m2 m kg/m3 mm Ns2/m4 Ns2/m8 Ns2/m8 m3/s m/s Pa Pa Pa
Low resistance 1000 3 5.4 16.2 16.8 1.2 100 0.00807 0.03189 0.0 1 0.03189 112 6.9 403 0 -403
High resistance 1000 3 5.4 16.2 16.8 1.2 300 0.01331 0.05259 0.0 1 0.05259 88 5.4 403 0 -403
0.010079

Parallel
resistance

In this example, of the total 200 m3/s , 112 m3/s flows through the low resistance airway and 88 m3/s in
the high resistance airway for a total pressure loss of 403 Pa. It is then relatively straightforward to link
from and to node pressures to create simple networks.

File UNSW-UME-(M1-3_Networks)(R1d) Page 29 of 29


SCHOOL OF MINING ENGINEERING
UNIVERSITY OF NEW SOUTH WALES
Australia

Underground Mine Environment Course


MODULE 1.0 VENTILATION AND MINE SERVICES

1.4 VENTILATION MONITORING AND


MINE SERVICES

A Minerals Tertiary Education Council Initiative


MINERALS
COUNCIL
Of Australia

File UNSW-UME-(M1-4_VentMon)(R1) Page 1 of 20


Revision date December 2003
School Of Mining Engineering Underground Mine Environment
 2002 University Of New South Wales Module 1.4 Ventilation Monitoring And Mine Services

Contents Page

1.0 INTRODUCTION………………………………….……………………………………………….. 3

2.0 VENTILATION MONITORING AND SURVEYS…………….…………………………………. 4

2.1 Velocity And Quantity………………………………….………………………………………. 4


2.2 Pressure………………………………….…………………..………………………….……… 9
2.3 Temperature…………………………………………….……………………………….……… 10
2.4 Ventilation Surveys………………………………….……….………………………………… 11
2.4.1 Analysis, Reporting And Interpretation Of Data…….………………………….…… 12

3.0 MINE SERVICES…………….……………………………………….…………………………… 15

3.1 Service Water And Pumping………………………………….……………………………… 15


3.2 Compressed Air………………………………….…………………………….……….……… 16
3.3 Gas Drainage Reticulation………………………………….………………………………… 17

4.0 REFERENCES…………….……………………………………….…………………………….… 20

5.0 NOMENCLATURE…………….……………………………………….………………………..… 20

File UNSW-UME-(M1-4_VentMon)(R1) Page 2 of 20


School Of Mining Engineering Underground Mine Environment
 2002 University Of New South Wales Module 1.4 Ventilation Monitoring And Mine Services

1.0 INTRODUCTION
The purpose of this module is to describe various equipment used for monitoring ventilation systems
either during discrete surveys or as permanent real time monitoring points. The scope of the module
covers monitoring of velocity, quantity and pressure. Monitoring of mine gases and interpretation of
mine atmospheres is covered in Atmospheric Gas Monitoring module 2.2.

With consideration to the highly dynamic nature of underground mining and the numerous parameters
and assumptions involved in predictive models, it is essential that real data is obtained to quantify
various aspects of the systems performance. This data is required to test the validity of predictive
models employed and to establish the "tram lines" within which the system is operating.

It is important to maintain a practical approach to surveys and monitoring to avoid excessive or


inadequate amounts of data being collected. The first step must always be to decide why monitoring
or surveys are being undertaken, what degree of accuracy is required, how is the data to be analysed
and archived for future reference.

Basic network analysis of the mine should be employed to estimate the likely magnitude of variables
to be measured and hence a suitable intensity of survey or monitoring stations.

In addition, a brief summary of issues associated with mine services (water, compressed air and gas
reticulation) is provided with particular emphasis on the application of fluid flow laws and relationships
previously described. The impact of these systems on mine ventilation systems clearly varies between
metalliferous and coal industries and even between individual mine sites.

These module notes summarise key aspects of ventilation monitoring, mine water, air and gas
reticulation, performance, formulae and calculation methods to a standard required for this course. In
this respect, they are not exhaustive and should only be used for guidance. It is essential that
candidates read and understand underpinning references for this module. Further references are
provided for candidates requiring more detailed information.

File UNSW-UME-(M1-4_VentMon)(R1) Page 3 of 20


School Of Mining Engineering Underground Mine Environment
 2002 University Of New South Wales Module 1.4 Ventilation Monitoring And Mine Services

2.0 VENTILATION MONITORING AND SURVEYS


In the Fluid Flow, Friction And Shock Loss and Fans And Auxiliary Ventilation modules, frequent
reference is made to measuring the principal values for defining fluid flow, namely velocity, pressure
and temperature. This section describes the equipment required and methods of application.

2.1 Velocity And Quantity


Cross sectional areas are required for calculation of quantity from observed velocity. In uniform
rectangular or circular sections these can be obtained by simply measuring height-width or diameter.
In non uniform sections, the most common method is to measure offsets from a tape or protractor,
refer Figure 2.1A. Alternatively, photographic methods can be used to highlight the airway profile with
a central disc for scale, refer Figure 2.1B.

Figure 2.1A Measuring Cross Sectional Area Figure 2.1B Measuring Cross Sectional Area
With Offsets From Tape or Protractor With Photography

Illuminated
profile

Flash

Centre disc for scale

The type of instrument or equipment used for measuring velocity depends if the flow is in a duct
(typically high velocity) or a mine airway (typically lower velocity). Methods used for airways are;

Instrument Range m/s Method and Comments


Smoke tubes < 0.5 m/s Used for very low velocity. Using cap lamp beams at right angles to flow, measure
the time for a cloud of smoke to travel a measured distance eg 3.0 m.
To allow for dispersion correct observed velocity by a factor of 0.8
Refer Figure 2.2
Anemometers 1.0 to 15 m/s Either mechanical clutch operation or electronic pulse counter.
Actually measures distance travelled for a known time.
5.0 to 50 m/s
Used for traverse or spot readings.
Traverse times should be greater than 30 seconds after blades rotating at steady
Refer Figure 2.3 state speed.

Hot Wire < 5.0 m/s Measures the variation in current required to keep a wire filament at constant
temperature.
Anemometers
The rate of change of heating decreases with increased velocity making the
instrument unsuitable for high velocities.

Velometers 0.2 to 30 m/s Measures the deflection of a vane due to impact velocity and flow through the
vane annulus.
Suitable for spot readings with a range of nozzles for different ranges.

Vortex Shedding > 1.0 m/s Operate by electronically monitoring fluctuations of flow in a vortex created by the
geometry of the instrument head. These are suitable for spot readings in high
Transducers
velocity ducts rather than permanent installations in low velocity airways but have
been used to measure main return ventilation rates with some degree of success
in both the metalliferous and coal mining industries.

Tracer gas Wide range of The tracer gas technique measures the concentration of a known quantity of gas
released into the ventilation stream.
quantity
Tracer gases such as sulphur hexafluride or carbon dioxide have been used.
This technique is used to quantify leakage paths in coal mines or airflow
Refer Figure 2.4 distributions in multi level shafts where direct measurement of velocity is
impracticable.

File UNSW-UME-(M1-4_VentMon)(R1) Page 4 of 20


School Of Mining Engineering Underground Mine Environment
 2002 University Of New South Wales Module 1.4 Ventilation Monitoring And Mine Services

Figure 2.2 Smoke Cloud Method For Figure 2.3A Uneven Velocity Profile
Low Velocities And Anemometer Spot Readings
Beam 1 Beam 2

Smoke
cloud

Flow

Figure 2.3B Anemometer Traverse Figure 2.3C Anemometer Calibration Curve


Correction
m/s
+0.10

+0.05

0.00

-0.05

-0.10

1.5
Observed Velocity m/s

For an anemometer Vcorrected = Vobserved x ρo + Vcorrection m/s


ρc
Where
ρo = density at observed velocity ρc = density at calibrated velocity

Worked Example – Anemometer Correction


An anemometer with the calibration curve Figure 2.3C is used to measure three velocities in an airway
(1.45, 1.5 and 1.55 m/s). The calibration density is 1.2 kg/m3 and the underground density is 1.14 kg/m3.
What is the corrected velocity?

Average observed velocity = (1.45 + 1.5 + 1.55) / 3 = 1.5 m/s Correction = + 0.05 m/s

Corrected velocity = 1.5 x 1.14 + 0.05 = 1.512 m/s


1.20

Figure 2.4 Tracer Gas Method

Q= m m3/s
ρ.A
Concentration at
observation m = mass of tracer gas, kg
point(s) Release ρ = density of tracer gas, kg/m3
time A = area under the
Flow time for
concentration – time graph
velocity

Time

File UNSW-UME-(M1-4_VentMon)(R1) Page 5 of 20


School Of Mining Engineering Underground Mine Environment
 2002 University Of New South Wales Module 1.4 Ventilation Monitoring And Mine Services

Higher velocities in ducts are normally measured with pitot tubes attached to a manometer of some
type. The air velocity is determined by measurement of the velocity pressure as described in the Fluid
Flow Friction And Shock Loss module 1.1.

Pitot tubes are used to measure spot readings rather than traverses and therefore each observation
point should be representative of a defined area. Where the velocity profile is very uneven, a greater
number of readings will be required.

The methods used for rectangular ducts is to divide the area in to 16 segments with observation points
as shown in Figure 2.5 A. For circular ducts, the cross section is divided into concentric circles
(typically 5) of equal area, refer Figure 2.5 B. It is also important to note that the velocities for each
area should be averaged and not the velocity pressures.

Figure 2.5 A Pitot Tube Location In Figure 2.5 B Pitot tube Location In
Rectangular Duct Circular Duct

Location of measuring
Rn = d 2n – 1 point with respect to
Cross section divided 4N centre
into 16 equal areas
n = nth reading from centre Rn = distance of nth reading from centre
d = diameter of pipe in mm N = number of readings across diameter

Figure 2.5 C Pitot Tube Geometry

Total pressure hole


(Facing)

Static pressure Stem


holes (Side)

Alignment
arm
Static
pressure

Total
pressure

File UNSW-UME-(M1-4_VentMon)(R1) Page 6 of 20


School Of Mining Engineering Underground Mine Environment
 2002 University Of New South Wales Module 1.4 Ventilation Monitoring And Mine Services

Worked Example – Circular Duct Points

The quantity of air flowing in a 760 mm diameter duct is to be measured with a pitot tube. Calculate the
location of the measuring 5 points on the radius with respect to the duct wall ?

Number of equal areas =5 therefore N = 10 With respect to one side of the duct

R1 = 760 x (2 x 1 – 1) / (4 x 10) = 120 Position 1 = 380 – 360 = 20 and position 10 = 380 + 360 = 740

R2 = 760 x (2 x 2 – 1) / (4 x 10) = 208 Position 2 = 380 – 318 = 62 and position 9 = 380 + 318 = 698

R3 = 760 x (2 x 3 – 1) / (4 x 10) = 269 Position 3 = 380 – 269 = 111 and position 8 = 380 + 269 = 649

R4 = 760 x (2 x 4 – 1) / (4 x 10) = 318 Position 4 = 380 – 208 = 172 and position 7 = 380 + 208 = 588

R5 = 760 x (2 x 5 – 1) / (4 x 10) = 360 Position 5 = 380 – 120 = 260 and position 6 = 380 + 120 = 500

These positions are normally marked on the pitot tube stem with tape or sliding rings

If the observed velocity pressures (density = 1.17 kg/m3) are as follows what quantity is in the duct ?

Measuring position 1 2 3 4 5 6 7 8 9 10
Velocity pressure Pa 290 300 310 325 340 335 320 310 305 280

For each location calculate observed velocity from V = 2 x VP m/s


ρ
VP V
Position D mm Pa m/s 24.5
1 20 290 22.3
24.0
2 62 300 22.6
3 111 310 23.0 23.5
Velocity m/s

4 172 325 23.6


23.0
5 260 340 24.1
6 500 335 23.9 22.5
7 588 320 23.4 22.0
8 649 310 23.0
9 698 305 22.8 21.5
10 740 280 21.9 0 200 400 600 800

Average 23.1 Distance From Wall mm

The average velocity is 23.1 m/s

The cross sectional area of a 760 mm duct = π. 0.762 /4 = 0.454 m2


The quantity in the duct is therefore 0.454 x 23.1 = 10.46 m3/s

File UNSW-UME-(M1-4_VentMon)(R1) Page 7 of 20


School Of Mining Engineering Underground Mine Environment
 2002 University Of New South Wales Module 1.4 Ventilation Monitoring And Mine Services

In some applications it is necessary to have permanent flow monitoring points (eg fan testing ducts,
gas drainage or service water pipes). In these situations, orifice plates, venturies or outlet cones may
be suitable, refer Figure 2.6.

Figure 2.6 Orifice Plates, Venturies And Outlet Cones


D1
D1/2
Q = A2 Cd 2 . ∆P m3/s
ρ( 1 – m2)
D1 D2
Cd = coefficient of discharge
A2 = cross sectional area of constriction, m2
m = approach factor, D22/D12
Cd = 0.60 to 0.61 ρ = air density, kg/m3
Cd = 0.99
5 to 14 0
D1 230 d D1 D2

D1/2 Cd = 0.93 to 0.85


D1/2 depending on angle of
B convergence 20 to 45 O
A

For each type of flow measuring device, the general orifice plate formula is applied with an appropriate
coefficient of discharge. The venturi has the lowest pressure loss but costs more to fabricate, the
orifice plate has a higher pressure loss but is cheaper to fabricate and allows for different plates to be
used for variations in flow rate.

Refer to Fluid Flow, Friction And Shock Loss module 1.1 for worked examples

File UNSW-UME-(M1-4_VentMon)(R1) Page 8 of 20


School Of Mining Engineering Underground Mine Environment
 2002 University Of New South Wales Module 1.4 Ventilation Monitoring And Mine Services

2.2 Pressure
A wide range of pressure measuring instruments is available, from simple U tubes to absolute
barometric pressure transducers. Selection of appropriate equipment depends on the magnitude of
the pressure being measured, whether gauge or absolute pressure is required and the accuracy to
which the pressure value is to be required. A further consideration is the number of readings to be
taken and the rigorous nature of the underground environment. For example, if flow rates are to be
measured from 10s of gas drainage holes, a simple mercury U tube would be appropriate. Detailed
mine pressure surveys with traversing aneroid barometers are not frequently undertaken due to the
complexity and time required.

Instrument Range Method and Comments


Pa
Manometers Wide range Used for gauge pressure measurements, relatively simple to operate, cheap and
relatively robust.
Should consider sensitivity resulting from fluid density and angle of inclined limbs.

Magnehelic Wide range A form of aneroid barometer in which a pressure differential is applied to a
diaphragm causing it to move against a spring. A magnet attached to the diaphragm
Gauges 0 to 100 Pa up
acts on a helix connected to a pointer.
to 130 kPa These are relatively inexpensive and suitable for surveys or permanent installations
across doors or regulators.

Mercury Atmospheric Fortin barometers are only suitable for use on surface and require corrections for
temperature and gravity. They can serve as a base control barometer but are not
Barometers pressure
used underground.

Aneroid Atmospheric Operation based on the expansion and contraction of a hollow evacuated flexible
tube. Barometers with mechanical linkage to reading dials are subject to hysteresis
Barometers pressure
where the error for rising pressure is different to falling pressure. These barometers
must be calibrated against mercury barometers prior to use.

Electronic Wide range Operation based on the natural frequency of oscillation of a quartz crystal beam
being directly related to absolute pressure. The ability to accurately electronically
barometers atmospheric or
measure changes in frequency makes these instruments very accurate ( 15 Pa in
differential 100 kPa).

Pressure Wide range A wide range of pressure transducers are available from various industrial and
aviation applications for use in underground mines. These normally produce a 4 to
Transducers atmospheric or
20 mA signal scaled for the required pressure range. They are relatively
differential inexpensive and provide an excellent method of continuous monitoring.

Figure 2.7 Various Manometers And Barometers


Multiple range inclined manometer

Pointer or chart
Linkage recorder

Spring

Vacuum

Inclined/Vertical limb
manometer
Aneroid Barometer

Fortin Barometer
File UNSW-UME-(M1-4_VentMon)(R1) Page 9 of 20
School Of Mining Engineering Underground Mine Environment
 2002 University Of New South Wales Module 1.4 Ventilation Monitoring And Mine Services

2.3 Temperature
As with pressure measurement, a wide range of instrumentation is available for measuring
temperatures of fluids or gases. Air temperatures for routine ventilation survey are normally carried out
with mercury in glass thermometers for wet and dry bulb temperatures. Again, the important
parameters are the range of temperatures to be measured and accuracy required.

The main groups of temperature measuring instruments available are as follows;

Instrument Method and Comments


Mechanical Employ differential expansion as the means of measuring temperature, for example thermostats.
They not normally used for direct measurement of temperature.
Thermometers

Fluid Most common is mercury in a glass capillary with graduations marked to measure the temperature
rise. A wide range of temperatures and accuracy are available, although for ventilation surveys a 0 to
Thermometers O O
110 C thermometer in 0.2 or 0.5 C divisions is the most common.

Electrical Thermocouples – use the thermoelectric properties of dissimilar alloy junctions.


Thermopiles – a series of connected thermocouples
Thermometers
Thermoresistive Thermistors – measures the effect of temperature on electrical resistance
Infrared detectors – employ the various absorption and emission rates of the infrared spectrum to
measure temperature – known as thermography – now available as recording video cameras for
diagnostics and archiving.

For ventilation purposes, wet and dry bulb temperatures are often required to calculate heat energy,
density, moisture content or humidity of air (refer Psychrometry module 3.1) or measurement of water
temperature in service water or chilled water pipes, refer Figure 2.7

When using mercury in glass thermometer, the largest source of error is caused by radiant heat from
objects hotter than the actual air temperature (Van Der Walt , 1982). An error of 0.1 OC DB can occur
if objects, such as the surface of airway strata, are only 2.0 OC hotter than the air. The effect on wet
bulb temperatures is reduced by increasing evaporation from the wet sock at an optimum 3.0 m/s, but
for both wet and dry bulb temperatures, measurements should not be taken in close proximity to hot
objects such as mobile diesel equipment.

Figure 2.7A Wet And Dry Bulb Thermometers Figure 2.8B Whirling Hygrometer
110 Water
Dry bulb reservoir

35
110 OC
range Wet bulb
sock
34 Handle

0.2 OC
graduations
0 Figure 2.7C Measuring Fluid
Temperature In A Pipe

Wet bulb
Dry bulb sock

File UNSW-UME-(M1-4_VentMon)(R1) Page 10 of 20


School Of Mining Engineering Underground Mine Environment
 2002 University Of New South Wales Module 1.4 Ventilation Monitoring And Mine Services

2.4 Ventilation Surveys


The principles of trailing hose and traverse barometer surveys, using the density method, were
described in Fluid Flow Friction And Shock Losses module 1.1. Other survey types that may be
required are as follows;

Survey Type Comments


Quantity survey Balance quantities assuming incompressible flow or on a mass flow of
dry air basis.
Normally undertaken using anemometers for airways or pitot tubes and
manometers for ducts.

Pressure surveys Trailing hose for direct measurement of frictional loss

Differential pressures for direct measurement of pressure differential


across stoppings or doors to determine frictional pressure loss in
sections of a mine.

Traverse barometer for indirect measurement of frictional loss in mine


airways. Requires that air density be measured (wet and dry bulb).
Fan performance To determine fan operating duty- quantity , static and velocity
pressures.
Heat source surveys Requires pressure, quantity wet and dry bulb to calculate heat (kW)
and moisture (g/s) exchange in mine airways, refer Heat In
Underground Mines, module 3.0
Gas emission Requires ventilation quantity and gas concentrations (hand held
instruments or samples for gas chromatograph etc)
Statutory compliance Typically require that volumetric and gas / dust concentration
surveys requirements are audited by means of periodic surveys. These will be
specified by applicable regulations or guidelines.

The amount of detail, planning and resources required will vary significantly with survey type, however,
some basic guidelines apply to most surveys, as follows;

1. Decide why the survey is being undertaken – this may be a routine statutory requirement or to
obtain specific information for decision making. Exactly what values are being sought ?

2. How will the survey data be documented, archived and analysed to provide the required data?
Check all pertinent data is collected (for example check that times are recorded for surface base
barometer and underground traverse barometer to correlate data sets).

3. Is it acceptable to assume incompressible flow or will alternative methods be required? For


example, it may be necessary to balance the survey using mass flow rates kg/s instead of
volumetric flow rates m3/s.

4. Given current understanding of the data to be collected and results of previous surveys, estimate
what range of values is to be expected. This will indicate the type of equipment required and
provides an ongoing check during execution of the survey.

5. What degree of accuracy is required and what degree of accuracy can be obtained with the
equipment to be used? Specify measurement procedures to those undertaking the survey, for
example three consistent anemometer readings or balance at an airway intersection.

6. Has all equipment been appropriately calibrated? For important surveys, which may involve
significant resources and time, it is often worth undertaking some site checks, for example use all
anemometers in the same airway, or run traverse barometers up and down a shaft to exercise the
linkage There is nothing worse than completing a survey only to find out afterwards that part of
the data set is invalid due to equipment errors or because some critical data has not been
collected.

File UNSW-UME-(M1-4_VentMon)(R1) Page 11 of 20


School Of Mining Engineering Underground Mine Environment
 2002 University Of New South Wales Module 1.4 Ventilation Monitoring And Mine Services

7. Plan the survey measuring locations and mark on a site plan. As with network analysis, avoid
making the survey overly complex. If a large survey is to be undertaken over more then one
underground visit, ensure that the surveys overlap at key points to check for variations. This is
particularly important for traverse barometer surveys.

8. It is preferred for as much preparatory work as possible (eg measuring cross sectional areas) to
be complete prior to the actual survey. These can be marked underground for long term use.

9. Consider timing of surveys to minimise the effect of mine operations (ventilation doors, production,
vehicle movement) on survey results. Alternatively, with surveys designed to measure production
effects (gas emission or heat) make sure that accurate production data can be obtained for the
survey period.

10. Always make duplicate copies of underground field notes while the person who took them is
available to explain their notation. Many notebooks containing previous days observations have
been lost or destroyed on subsequent trips underground.

11. Routine survey results should be archived in a manner that allows trends to be established, for
example quantity at the face with development advance. These provide an excellent means of
reconciling predictive models and establishing the mine’s norm.

It is also the case, particularly in coal mines, that quantity measurements taken to meet statutory
requirements are rarely sufficient to provide a mine wide balance for diagnostic assessment of system
performance. It is recommended that additional quantities are determined at locations in main
headings to quantify leakage.

2.4.1 Analysis Reporting And Interpretation Of Data


Routine ventilation surveys should be structured to provide data and supporting information for;

1. Audit and review of ventilation system performance against plan and statutory requirements.
2. Evaluation and control of changes to mine ventilation systems.
3. Forecasting, calculating and planning for future ventilation requirements.
4. Establishment of the mine norm and provide site specific design parameters.
5. Indications of ventilation system failure or non compliance.
6. Reconciliation with predictive models.

Whilst pertinent standards vary between coal and metalliferous mines, the fundamental requirements
of ventilation survey and real time monitoring systems should be to provide

1. Quantity and quality of air supplied to each working place.


2. The volumetric distribution of ventilation and leakage in the mine.
3. Distribution of pressure differentials between cardinal points within the circuit.
4. Primary circuit fan pressure – quantity duty points.

The term “volumetric efficiency” is used to describe a measure of how much of the mine’s total
ventilation capacity is used effectively. This can be either how much of the total mine capacity reaches
the start of panels or working sections, or how much of the total is actually used in work places. These
values therefore provide a measure of how efficiently available ventilation capacity is being used.

Low volumetric efficiencies can result from leakage through stoppings, direct short circuits, or diversity
of mining locations. In both coal and metalliferous mines the key issue is to quantify the magnitude of
these effects and their location.

An example of balancing coal and metalliferous mine circuits are shown in Figures 2.9 and 2.10
respectively.

File UNSW-UME-(M1-4_VentMon)(R1) Page 12 of 20


School Of Mining Engineering Underground Mine Environment
 2002 University Of New South Wales Module 1.4 Ventilation Monitoring And Mine Services

Figure 2.9 Survey Locations For A Generic Longwall Circuit

Q bleeder
Inbye
maingate

Face

Last c/t

Exhaust
Start Gate shaft
Q gate
R
R
Q mains

Outbye Start of
active Outbye mains Intake
Mains intakes Longwall intakes drifts
tailgate leakage
and returns and returns Q mine
Inbye Q longwall
leakage

In coal mines, a volumetric balance is obtained by measuring horizontal cutsets through intake and
return airways.

Worked Example – Ventilation Survey Analysis Coal

The results of a quantity distribution survey in the circuit shown in Figure 2.9 are as follows
Quantity Entering Panels Main Exhaust Shafts
Panel m3/s Intakes m3/s m3/s
1 Gateroad 1 45 1 P& E drift 110 1 No.1 shaft 200
2 2 Conveyor 85 2
3 Mains 50 3 3
4 Longwall 60 4 4
5
6 Total panels 155 Total Intake 195 Total Exhaust 200
7

Using the Survey Balance spreadsheet, identify leakage and volumetric efficiency if intake mains at pit
bottom carry 185 m3/s and intake mains at the tailgate carry 160 m3/s

The total leakage amounts to 40 m3/s, of this


25 m3/s occurs outbye the tailgate and 15 m3/s Survey Data m3/s Analysis
inbye. Total intake 195
Total exhaust 200 Total balance 5 m3/s
The volumetric efficiency based on total to panels Mains intake pit bottom 190 Pit bottom leak 5 m3/s
Compared to total mine capacity is 79.5%. Mains intake at tailgate 170 Outbye leak 20 m3/s

Total to panels 155 Total leak 40 m3/s


Inbye leak 15 m3/s
Vol efficiency 79.5 %

File UNSW-UME-(M1-4_VentMon)(R1) Page 13 of 20


School Of Mining Engineering Underground Mine Environment
 2002 University Of New South Wales Module 1.4 Ventilation Monitoring And Mine Services

Figure 2.10 Survey Locations For A Generic Metalliferous Circuit


No.1 Shaft No.2 Shaft
Internal distribution
as required

1 1
1
2 2 Upper production zone 2
3 3
3
4 4
4
5 5
5
6 6
6
7 7
7
8 8
9 9 Lower production 8
zone 9
10 10

Exhaust shaft
Transfer between levels
sections
Intake shaft levels

In metalliferous mines, a more complex situation may arise from multiple horizontal and vertical
connections. In addition, the effect of compression may need to be taken into account in order to
balance the circuit on a mass flow rather than volumetric basis.

Worked Example – Ventilation Survey Analysis Metalliferous

The results of a quantity distribution survey in the circuit shown in Figure 2.10 are as follows

What is the circuit balance ? Level m3/s Level m3/s Level m3/s
1 1 1
Total intake to upper production levels = 95 m3/s from No.1 shaft 2 10 2 2
3 25 3 3
Total intake to lower production levels = 310 m3/s 4 50 4 4
Of this, 90 m3/s is from No.1 shaft and 220 m3/s from No.2 shaft 5 10 5 5
6 6 6
The total measured intake is 185 + 220 = 405 m3/s 7 7 7
8 8 50 8
The total exhaust is 430 m3/s 9 90 9 90 9
10 10 80 10
The total return from upper levels is 120 m3/s indicating about 185 220 0
25 m3/s is being transferred from lower levels.

Level m3/s Level m3/s Level m3/s


1 1 1
2 2 2
3 60 3 3
4 80 4 4
5 5 5
6 6 6
7 7 7
8 100 8 8
9 190 9 9
10 10 10
430 0 0

It is recognised that both these examples are relatively trivial in nature but it is basic balances of this
type that are sometimes overlooked. The issues to note are that dividing a mine up into distinct
districts or panels allows an overall picture to be established without an excessive number of readings.
In addition, conventional monitoring of workplaces alone, for statutory compliance purposes, will not
provide this information.

File UNSW-UME-(M1-4_VentMon)(R1) Page 14 of 20


School Of Mining Engineering Underground Mine Environment
 2002 University Of New South Wales Module 1.4 Ventilation Monitoring And Mine Services

3.0 MINE SERVICES


The purpose of this section is to apply fluid flow laws and relationships, covered previously, to mine
services that may be required for environmental control. It is not the intention to cover all operational
issues associated with these systems.

The spreadsheet developed for calculation of frictional losses in mine airways will be applied to these
system with appropriate corrections for pipe roughness and fluid density. It is also necessary to use
gas laws described in the Mine Gases and Gas Laws module 2.1.

3.1 Service Water And Pumping


Service water is required for a number of purposes in underground mines, principally dust
suppression, and ground water make will depend on the mine geometry and location of aquifers.

For the purposes of this module, the issues to address are pipe sizing and pump power requirements.
Water quality may also be an issue, depending on the source of make up water or treatment of re
circulating fractions.

To calculate pressure losses and pumping requirements, water can of course be considered an
incompressible fluid and therefore the principles previously developed for turbulent flow apply with
equal validity. However, the water management system of a mine is typically “open circuit” and
therefore power is required to raise water out of the mine in a addition to overcoming frictional losses.

In note 10.2 of Le Roux’s notes, a table of frictional loss (m per 100 m) in water pipes is provided. This
is the loss in m H2O per 100 m pipe. A more detailed graphical representation is also provided by
Stroh, 1982.

These values can also be obtained using the spreadsheet developed for analysis of frictional losses in
mine airways and ducts. For example, the pressure loss for a water flow rate of 8 l/s in a 100 m long,
51 mm diameter pipe is 4.79 x 105 Pa, refer Table 3.1. (note that a density of 1000 kg/m3 and
roughness height of 0.30 mm is used).

Table 3.1 Application Of Frictional Loss Spread Sheet To Water Pipe Losses

Length Height Width Area Perim Density Rough k Resis(1) X No R(total) Q V DP


m Diam m2 m kg/m3 mm Ns2/m4 Ns2/m8 Ns2/m8 m3/s m/s Pa
100 0.051 0 0.00204 0.16022 1000.0 0.3 0.00478 7.49E+09 0.0 1 7.49E+09 0.008 3.9 4.794E+05

In terms of loss m per 100 m = 4.79 x 105 = 48.9 m which compares with Le Roux’s 49 m
(1000 x 9.81)

This demonstrates that the basic fluid flow principals used to develop the spreadsheet for gas flow can
also be applied to liquid flow.

In a pump column discharging to atmosphere (surface dam) the pressure at the bottom of the column
is obtained from P = ρ.g.h + Pfric in the same way that static pressure would be obtained in a
downcast or upcast shaft, but again using a density of 1000 kg/m3.

The pump power required to raise water is given by;

Equation 3.1 Pump Power

Pump power = Pressure x Quantity = (ρ.g.h + Pfric) x Q kW


1000 η

Note the similarity with fan power, both equations are really stating the same thing except that the
pump out line is open circuit from underground to surface.

File UNSW-UME-(M1-4_VentMon)(R1) Page 15 of 20


School Of Mining Engineering Underground Mine Environment
 2002 University Of New South Wales Module 1.4 Ventilation Monitoring And Mine Services

Worked Example – Pump Line Diameter And Pump Power


A pump out line is required to manage 45 l/s and raise water through an elevation of 250 m. If the pump
efficiency is 75%, what pipe diameter and pump duty should be used ?

At 55 l/s and a velocity of 2 m/s (refer Le Roux) the pipe diameter is given by

Q=V.A 0.045 = 2 x πD2 D = 0.055 x 4 = 0.187 m = 187 mm


4 2xπ

Closest larger standard pipe size = 205 mm

Using the friction spreadsheet, the loss per 100 m = 14,608 Pa or 1.489 m H2O

Length Height Width Area Perim Density Rough k Resis(1) X No R(total) Q V DP


m Diam m2 m kg/m3 mm Ns2/m4 Ns2/m8 Ns2/m8 m3/s m/s Pa
100 0.205 0 0.03301 0.64403 1000.0 0.3 0.00324 4.83E+06 0.0 1 4.83E+06 0.055 1.7 1.461E+04

Pump power required = (ρ.g.h + Pfric) x Q/1000 = (1000 x 9.81 x 350 + 14,608 x 250 ) x 0.055 = 136 kW
100 1000

Pump motor power = 136 / 0.75 = 181 kW

3.2 Compressed Air


Reticulation of compressed air provides another application of the frictional loss spreadsheet, but this
time to compressed gases at significantly higher density than normal air.

Note that the imperial unit of pressure, pounds per square inch (PSI) is related to Pascals by

1 PSI = 6.895 kPa For example, a typical 80 PSI supply is the same as 551 kPa gauge.

Assuming air behaves as a perfect gas (refer Mine Gases And Gas Laws module 2.1), the effect of
changing pressure on density can be calculated using Boyle’s law. As Le Roux points out, this is an
approximation and assumes that the temperature of compressed air within the pipe rapidly equalises
with the temperature of ventilation outside the pipe. For practical purposes this assumption is
acceptable.

Compressed air density is then calculated as follows

Assume density of ambient air entering the compressor is 1.18 kg/m3

The air is compressed to + 550 kPa gauge so (at the same temperature) the volume changes to

V2 = V1 x P 1 m3 V1 = 1.0 m3 , let P1 = 100 kPa therefore P2 = 100 + 550 = 650 kPa absolute
P2

V2 = 1.0 x 100 = 0.1538 m3 but the mass remains constant at 1.18 kg


650

Therefore the density of compressed air = 1.18 / 0.1538 = 7.67 kg/m3 ie compressed air is some 7 to
8 times denser than air at ambient conditions.

This gives rise to the term “free air delivery” which is used to quantify the amount of compressed air in
a pipe in terms of the volume it will produce when discharged to atmosphere. In Australian conditions
free air density is about 1.2 kg/m3 but in deeper mines, it may be convenient to select a more
appropriate value to reflect the effect of increased atmospheric pressure at depth.

File UNSW-UME-(M1-4_VentMon)(R1) Page 16 of 20


School Of Mining Engineering Underground Mine Environment
 2002 University Of New South Wales Module 1.4 Ventilation Monitoring And Mine Services

Worked Example – Compressed Air

A 150 mm compressed air pipe is to required to deliver 0.5 m3/s free air at 1.2 kg/m3. The pressure inside
the pipe is +550 kPa gauge and atmospheric pressure is 100 kPa.

What is the volumetric flow rate in the pipe and the frictional pressure loss per 100 m ?

Volumetric flow rate inside the pipe = 0.5 x 100 = 0.0769 m3/s
650

Density of compressed air = 1.2 x 650/100 = 7.8 kg/m3

Using the frictional loss spreadsheet, the loss per 100 m = 1.33 kPa

Length Height Width Area Perim Density Rough k Resis(1) X No R(total) Q V DP


m Diam m2 m kg/m3 mm Ns2/m4 Ns2/m8 Ns2/m8 m3/s m/s Pa
100 0.15 0 0.01767 0.47124 7.8 0.5 0.00404 2.24E+05 0.0 1 2.24E+05 0.077 4.4 1.327E+03

The pressure loss is therefore 1.327 kPa per 100 m

It is important to note that, in long pipe lengths, the volumetric flow rate within the pipe changes due to
frictional losses. In the above example, the pressure inside the pipe is required to be 550 kPa gauge
at a flow rate of 0.5 m3/s free delivery or 0.077 m3/s inside the pipe. If frictional losses are 1.33 kPa per
100 m then the pressure inside the pipe will be 1.33 kPa higher 100 m from the end ie 551.33 kPa
gauge. Again, Boyle’s law can be used to recalculate the volumetric flow rate and therefore the effect
on frictional losses.

Refer Deglon, 1982 for further information on compressed air systems in underground mines.

3.3 Gas Drainage Reticulation


The reticulation of seam gas in coal mine gas drainage systems involves multi-component gas
mixtures (CO2 CH4 N2 O2) at negative pressures, typically -10 to –50 kPa gauge. In this situation, gas
density is less than that at free delivery and therefore the volumetric flow rate is higher inside the pipe.
This becomes important when selecting appropriate pipe sizes as the effect of increased negative
pressure is to increase frictional losses.

Normally the design criteria is to maintain the entire system at negative pressure to avoid leakage of
seam gas into the workings. The situation becomes more complex when gas enters the pipe range at
multiple points along it’s length.

The design parameters are the minimum negative pressure required at the furthest extent of the pipe
range, the profile of gas entry to the pipe and limiting suction pressure into the surface extraction
pump or fan. If a liquid ring pump is used, surface gauge pressures of –50 to –70 kPa may be
achieved, if a high pressure centrifugal fan is used, pressure will be limited to –10 to –20 kPa.

Once gas density has been determined (refer Mine Gases and Gas Laws module 2.1), the
calculation procedure is essentially the same s that for compressed air. The following worked example
illustrates the effect of negative pressure on volumetric flow rates and consequential frictional losses.

Operational issues associated with installation and management of gas drainage systems are covered
in more detail in Gas Drainage Outbursts And Windblasts module 5.2. The most significant
operational issue with both compressed air and gas drainage reticulation systems is removal of water
due either to condensation or ingress of ground water.

File UNSW-UME-(M1-4_VentMon)(R1) Page 17 of 20


School Of Mining Engineering Underground Mine Environment
 2002 University Of New South Wales Module 1.4 Ventilation Monitoring And Mine Services

Worked Example – Reticulation

A gas mixture with a density of 0.85 kg/m3 is to be reticulated through a 3500 m long main pipe line.
700 l/s (free delivery) gas enters the end of the pipe and a further 900 l/s (free delivery) enters the pipe
1500 m from the end. The pressure at the end of the pipe is required to be –10 kPa and the minimum
surface pressure is –45 kPa.
900 l/s -45 kPa
-10 kPa
700 l/s
1500 m 2000 m

If the whole pipe length is to be the same size, what pipe diameter is required to meet these design
criteria ?

Starting at the end 1600 l/s from 100 kPa to 90 kPa inside the pipe

Volumetric flow rate inside the pipe = 1.6 x 100 = 1.78 m3/s
(100-10)

Density inside the pipe = 0.85 x 90 = 0.765 kg/m3


100

The total loss in the pipe must be less than 35 kPa (45 – 10) to maintain adequate suction at the end, ie an
average 1.0 kPa per 100 m. The pressure loss for three standard pipe diameters are compared in the Frictional
loss spreadsheet.

Length Height Width Area Perim Density Rough k Resis(1) X No R(total) Q V DP


m Diam m2 m kg/m3 mm Ns2/m4 Ns2/m8 Ns2/m8 m3/s m/s Pa
100 0.305 0 0.07306 0.95819 0.7650 0.5 0.00333 5.22E+02 0.0 1 5.22E+02 1.780 24.4 1.7E+03
100 0.356 0 0.09954 1.11841 0.7650 0.5 0.00320 2.31E+02 0.0 1 2.31E+02 1.780 17.9 7.3E+02
100 0.406 0 0.12946 1.27549 0.7650 0.5 0.00310 1.16E+02 0.0 1 1.16E+02 1.780 13.7 3.7E+02

This indicates that a 305 mm pipe is too small (1.7 kPa per 100 m) and a 406 mm pipe is too large (0.37 kPa per
100 m). A 356 mm pipe provides a loss of 0.73 kPa per 100 m which is about right to initiate the calculation. Of
course a 406 mm pipe would be acceptable from a frictional loss point of view, the only consideration is the
difference in capital and installation costs.

For this example, the pipe will be considered in 500 m lengths for simplicity. This could easily be refined to say
100 m lengths by adding additional rows in the spread sheet.

Starting with the first 500 m length from the end of the pipe. Volume = 0.7 x 100/90 = 0.778 m3/s

Length Height Area Perim Density Rough k Resis(1) X No R(total) Q V DP From To From To
m Diam m2 m kg/m3 mm Ns2/m4 Ns2/m8 Ns2/m8 m3/s m/s Pa Pa Pa kPa kPa
500 0.356 0.0995 1.1184 0.765 0.5 0.00320 1157.38 0.0 1 1,157.4 0.778 7.8 700 90,000 89,300 -10 -11

The frictional pressure loss is 0.7 kPa, therefore the pressure 500 m from the end is 89.3 kPa absolute (-11 kPa
gauge).

The volumetric flow rate in the pipe then changes to 0.778 x 90 / 89.3 = 0.784 m3/s and the density changes to
0.765 x 89.3 / 90 = 0.759 kg/m3

The process is then repeated for the next 1000 m in two 500 m increments
Length Height Area Perim Density Rough k Resis(1) X No R(total) Q V DP From To From To
m Diam m2 m kg/m3 mm Ns2/m4 Ns2/m8 Ns2/m8 m3/s m/s Pa Pa Pa kPa kPa
500 0.356 0.0995 1.1184 0.765 0.5 0.00320 1157.38 0.0 1 1,157.4 0.778 7.8 700 90,000 89,300 -10 -11
500 0.356 0.0995 1.1184 0.759 0.5 0.00320 1148.37 0.0 1 1,148.4 0.784 7.9 706 89,300 88,594 -11 -11
499 0.356 0.0995 1.1184 0.753 0.5 0.00320 1137.02 0.0 1 1,137.0 0.790 7.9 710 88,594 87,884 -11 -12
1 0.356 0.0995 1.1184 0.747 0.5 0.00320 2.26 0.0 1 2.3 0.797 8.0 1 87,884 87,883 -12 -12

The last short 1 m length has been inserted simply to show the density and flow rate leaving the first section.

At the point that 900 l/s enters the pipe, the pressure is 87.883 kPa absolute ( - 11.2 kPa gauge) and the density
is 0.747 kg/m3 .

At these conditions, the volume of gas entering the pipe is 0.90 x 100 / 87.883 = 1.024 m3/s and the quantity in
the pipe is then 0.797 + 1.024 = 1.821 m3/s.

File UNSW-UME-(M1-4_VentMon)(R1) Page 18 of 20


School Of Mining Engineering Underground Mine Environment
 2002 University Of New South Wales Module 1.4 Ventilation Monitoring And Mine Services

Worked Example – Gas Reticulation

The process is then repeated for the 2000 m length to the surface pump in four 500 m increments.
Length Height Area Perim Density Rough k Resis(1) X No R(total) Q V DP From To From To
m Diam m2 m kg/m3 mm Ns2/m4 Ns2/m8 Ns2/m8 m3/s m/s Pa Pa Pa kPa kPa
500 0.356 0.0995 1.1184 0.765 0.5 0.00320 1157.38 0.0 1 1,157.4 0.778 7.8 700 90,000 89,300 -10 -11
500 0.356 0.0995 1.1184 0.759 0.5 0.00320 1148.37 0.0 1 1,148.4 0.784 7.9 706 89,300 88,594 -11 -11
499 0.356 0.0995 1.1184 0.753 0.5 0.00320 1137.02 0.0 1 1,137.0 0.790 7.9 710 88,594 87,884 -11 -12
1 0.356 0.0995 1.1184 0.747 0.5 0.00320 2.26 0.0 1 2.3 0.797 8.0 1 87,884 87,883 -12 -12
500 0.356 0.0995 1.1184 0.747 0.5 0.00320 1130.15 0.0 1 1,130.2 1.821 18.3 3746 87,883 84,137 -12 -16
500 0.356 0.0995 1.1184 0.715 0.5 0.00320 1081.98 0.0 1 1,082.0 1.902 19.1 3913 84,137 80,224 -16 -20
500 0.356 0.0995 1.1184 0.682 0.5 0.00320 1031.66 0.0 1 1,031.7 1.994 20.0 4104 80,224 76,121 -20 -24
499 0.356 0.0995 1.1184 0.647 0.5 0.00320 976.93 0.0 1 976.9 2.102 21.1 4316 76,121 71,804 -24 -28
1 0.356 0.0995 1.1184 0.610 0.5 0.00320 1.85 0.0 1 1.8 2.228 22.4 9 71,804 71,795 -28 -28

The solution is as follows

-10 kPa 900 l/s -28 kPa


778 l/s 797 l/s 1821 l/s 2228 l/s
700 l/s
1500 m -12 kPa 2000 m

With a 356 mm pipe, the pump duty would be 2.3 m3/s at about 72 kPa absolute (-28 kPa gauge).

Once the spreadsheet is set up, the effect of changing pipe diameter can be tested, for example, if all the pipe
was 305 mm diameter, the results are;

Length Height Area Perim Density Rough k Resis(1) X No R(total) Q V DP From To From To
m Diam m2 m kg/m3 mm Ns2/m4 Ns2/m8 Ns2/m8 m3/s m/s Pa Pa Pa kPa kPa
500 0.305 0.0731 0.9582 0.765 0.5 0.00333 2608.81 0.0 1 2,608.8 0.778 10.6 1578 90,000 88,422 -10 -12
500 0.305 0.0731 0.9582 0.752 0.5 0.00333 2563.06 0.0 1 2,563.1 0.792 10.8 1606 88,422 86,816 -12 -13
499 0.305 0.0731 0.9582 0.738 0.5 0.00333 2511.46 0.0 1 2,511.5 0.806 11.0 1633 86,816 85,183 -13 -15
1 0.305 0.0731 0.9582 0.724 0.5 0.00333 4.94 0.0 1 4.9 0.822 11.2 3 85,183 85,179 -15 -15
500 0.305 0.0731 0.9582 0.724 0.5 0.00333 2469.07 0.0 1 2,469.1 1.878 25.7 8712 85,179 76,468 -15 -24
500 0.305 0.0731 0.9582 0.650 0.5 0.00333 2216.55 0.0 1 2,216.5 2.092 28.6 9704 76,468 66,763 -24 -33
500 0.305 0.0731 0.9582 0.567 0.5 0.00333 1935.25 0.0 1 1,935.3 2.397 32.8 #### 66,763 55,649 -33 -44
499 0.305 0.0731 0.9582 0.473 0.5 0.00333 1609.85 0.0 1 1,609.8 2.875 39.4 #### 55,649 42,341 -44 -58
1 0.305 0.0731 0.9582 0.360 0.5 0.00333 2.45 0.0 1 2.5 3.779 51.7 35 42,341 42,306 -58 -58

The result would be 3.8 m3/s at –58 kPa. This confirms that a 356 mm pipe should be used and also
demonstrates the profound effect of pipe diameter on frictional losses when true gas compression is taken into
account.

File UNSW-UME-(M1-4_VentMon)(R1) Page 19 of 20


School Of Mining Engineering Underground Mine Environment
 2002 University Of New South Wales Module 1.4 Ventilation Monitoring And Mine Services

6.0 REFERENCES

6.1 References Underpinning This Module


Le Roux W.L. 1990 “Le Roux’s Notes On Mine Environmental Control -Ch.9 Compressed Air
Ch10 Water Ch11 Electricity ”.Published by Mine Ventilation Society Of South Africa

6.2 Further References


Deglon P. 1982 “Environmental Engineering In South African Mines – Ch.9 Compressed Air”
Published by Mine Ventilation Society Of South Africa.1982

Hemp R.. 1982 “Environmental Engineering In South African Mines – Ch 6 Pressure Surveys”
Published by Mine Ventilation Society Of South Africa.1982

Holding W.. 1982 “Environmental Engineering In South African Mines – Ch 5 Pressure Measuring
Instruments” Published by Mine Ventilation Society Of South Africa.1982

Ramsden R.. 1982 “Environmental Engineering In South African Mines – Ch4 The measurement of
Water Flow” Published by Mine Ventilation Society Of South Africa.1982

Schmitz B. 1982 “Environmental Engineering In South African Mines – Ch3 The Measurement Of
Airflow” Published by Mine Ventilation Society Of South Africa.1982

Stroh R.M. 1982 “Environmental Engineering In South African Mines – Ch.25 Chilled Water
Reticulation” Published by Mine Ventilation Society Of South Africa.1982

Ven Der Walt N.T. 1982 “Environmental Engineering In South African Mines – Ch.17 Thermometry
And Temperature Measurement” Published by Mine Ventilation Society Of South Africa.1982

7.0 NOMENCLATURE

Symbols

A = cross-sectional area, m2 ρ = density, kg/m3


C = circumference, m R = resistance, Ns2/m8
Cp = thermal capacity, kJ/kg OC SP = static pressure, Pa
D = diameter, m T = temperature, celsius
Dh = hydraulic diameter, m T = temperature, kelvin
g = gravitational acceleration, m/s2 TP = total pressure, Pa
h = height, m µ = absolute or dynamic viscosity, kg/ms
I = electrical current, amps V = velocity m/s , volume m3 or electrical voltage, volts
k = Atkinson’s friction factor, Ns2/m4 VP = velocity pressure, Pa
L = length, m W = power, W kW
n = rotational speed, rpm X = shock loss factor
η = efficiency, % Z = elevation, m
P = pressure, Pa, kPa cos(Φ) = AC power factor
FP = frictional pressure loss, Pa kPa
Q = quantity, m3/s

File UNSW-UME-(M1-4_VentMon)(R1) Page 20 of 20


© 2003 Underground Mine Environment Module 1.4 Ventilation Monitoring Photographs © 2003 Underground Mine Environment Module 1.4 Ventilation Monitoring Photographs

Ventilation And Mine Services Velocity Measuring Equipment

Ventilation System Monitoring Photographs


Pitot Tube

Velometer Electronic Anemometers

Contents
– Velocity Measuring Equipment
– Pressure Measuring Equipment
– Temperature Measuring Equipment

Anemometer In Serious
Need Of Calibration Using An Anemometer

© 2003 Underground Mine Environment Module 1.4 Ventilation Monitoring Photographs © 2003 Underground Mine Environment Module 1.4 Ventilation Monitoring Photographs

Velocity And Flow Measuring Equipment Pressure Differential Measuring Equipment


Inclined/Vertical Manometer

Venturi Meter

Velocity Transducer

Inclined/Vertical Manometer

Orifice Plate

Magnehelic Gauge Electronic Manometer

© 2003 Underground Mine Environment Module 1.4 Ventilation Monitoring Photographs © 2003 Underground Mine Environment Module 1.4 Ventilation Monitoring Photographs

Barometric Pressure Measuring Equipment Temperature Measuring Equipment


Dry Bulb

Wet Bulb

Sling
psychrometer

Single Limb
Chart Barometer Wet Bulb

Whirling
Hygrometers

Assman
Fortin hygrometer
Barometer Mercury In Glass
Electronic Barometer Thermometer

1
© 2003 Underground Mine Environment Module 1.4 Ventilation Monitoring Photographs

Monitoring Systems – Surface Fans

LAST SLIDE RETURN TO START >>>>

Das könnte Ihnen auch gefallen